SóProvas



Prova PUC-PR - 2017 - TJ-PR - Analista Judiciário - Psicologia


ID
2478613
Banca
PUC-PR
Órgão
TJ-PR
Ano
2017
Provas
Disciplina
Português
Assuntos

                     Música ajuda no combate ao câncer

Colocar fones de ouvido e dar play no aparelho de som traz vantagens reais à luta contra a doença

Experts do Instituto Cochrane fizeram uma revisão de estudos para entender qual o papel da música na rotina de quem lida com um câncer. Para isso, eles analisaram 52 pesquisas já publicadas sobre o tema que envolveram mais de 3 700 voluntários. A conclusão mostra que ouvir algumas canções tem efeitos positivos em diversos parâmetros de saúde e de qualidade de vida.

Foram observadas melhoras nos níveis de ansiedade, dor, fadiga, batimentos cardíacos, respiração e pressão arterial. “É natural em quem descobre o problema um grande abalo emocional. As melodias ajudam no reequilíbrio e no preparo para encarar as sessões de quimioterapia ou de radioterapia”, comenta Raul Brabo, coordenador do curso de musicoterapia das Faculdades Metropolitanas Unidas, em São Paulo. O musicoterapeuta, o profissional com formação na área, vai selecionar cantores, estilos e letras que agradam o paciente e fazem sentido naquele contexto.

O trabalho com as pesquisas anteriores, porém, não encontrou repercussões da abordagem no humor, no sistema imune ou na habilidade de comunicação. “Para que o benefício seja maior, as composições selecionadas para o momento precisam ter um significado na vida do indivíduo”, reforça Brabo.

Disponível em: <http://saude.abril.com.br/medicina/musica-ajuda-no-combate-ao-cancer/> . Acesso em: 24/04/2017, às 19h01min (Adaptado). 

O texto é marcado por recursos coesivos que ajudam o autor a atingir seu objetivo. Assim, o último parágrafo do texto

Alternativas
Comentários
  • c) faz uma ressalva à abrangência dos efeitos da música no tratamento do câncer ["O trabalho com as pesquisas anteriores, porém, não encontrou repercussões da abordagem no humor, no sistema imune ou na habilidade de comunicação"] e indica uma condição para a maximização dos resultados [“Para que o benefício seja maior, as composições selecionadas para o momento precisam ter um significado na vida do indivíduo”].

  • Olá, venho compartilhar com todos vocês esté método, que me fez obter um rendimento insano em pouco tempo.

     é do Professor Marlon Souza, Especialista em técnica de estudos e métodos de aprendizagem acelerada.

    https://go.hotmart.com/Q8744867J

    Confiram, eu comprei e vale muitoooo a pena.


ID
2478616
Banca
PUC-PR
Órgão
TJ-PR
Ano
2017
Provas
Disciplina
Português
Assuntos

                     Música ajuda no combate ao câncer

Colocar fones de ouvido e dar play no aparelho de som traz vantagens reais à luta contra a doença

Experts do Instituto Cochrane fizeram uma revisão de estudos para entender qual o papel da música na rotina de quem lida com um câncer. Para isso, eles analisaram 52 pesquisas já publicadas sobre o tema que envolveram mais de 3 700 voluntários. A conclusão mostra que ouvir algumas canções tem efeitos positivos em diversos parâmetros de saúde e de qualidade de vida.

Foram observadas melhoras nos níveis de ansiedade, dor, fadiga, batimentos cardíacos, respiração e pressão arterial. “É natural em quem descobre o problema um grande abalo emocional. As melodias ajudam no reequilíbrio e no preparo para encarar as sessões de quimioterapia ou de radioterapia”, comenta Raul Brabo, coordenador do curso de musicoterapia das Faculdades Metropolitanas Unidas, em São Paulo. O musicoterapeuta, o profissional com formação na área, vai selecionar cantores, estilos e letras que agradam o paciente e fazem sentido naquele contexto.

O trabalho com as pesquisas anteriores, porém, não encontrou repercussões da abordagem no humor, no sistema imune ou na habilidade de comunicação. “Para que o benefício seja maior, as composições selecionadas para o momento precisam ter um significado na vida do indivíduo”, reforça Brabo.

Disponível em: <http://saude.abril.com.br/medicina/musica-ajuda-no-combate-ao-cancer/> . Acesso em: 24/04/2017, às 19h01min (Adaptado). 

Veja a definição do verbo “agradar” no Dicionário Eletrônico Houaiss.

agradar

verbo

transitivo direto, transitivo indireto e intransitivo 1 ser agradável, transmitir satisfação a; dar prazer, contentar pronominal 2 comprazer-se em; experimentar prazer, deleite; sentir-se encantado pronominal 2.1 sentir-se enamorado, tomar-se de amores transitivo direto 3 Regionalismo: Nordeste do Brasil. azer agrados, carinhos; afagar.

Agora releia o seguinte trecho do texto.

(...) vai selecionar cantores, estilos e letras que agradam o paciente (...).

Se tomarmos como base de correção gramatical o que o dicionário afirma sobre o verbo “agradar, o trecho do texto está

Alternativas
Comentários
  • verbo "agradar"

    sujeito coisa > agrada "a" (VTI) 
    sujeito pessoa > agrada (VTD)

     

  • Nos sentidos de “afagar” e “acariciar”, “agradar” será verbo transitivo direto (VTD). Exemplos:

    A mão nunca sai para trabalhar sem antes agradar os filhos.

    O rapaz agradava a namorada.

     

    http://dicasdiariasdeportugues.com.br/regencia-do-verbo-agradar/

  • Isso mesmo, Juliana. Porém isso não ocorre no contexto usado no enunciado. Não entendi por que a alternativa B foi dada como correta, visto que, no contexto, o verbo agradar não está sendo usado com o sentido de "afagar", "acariciar" (VTD, nesses casos), mas sim com o sentido de "transmitir satisfação"; sendo, portanto, VTI. Para mim, a correta seria a C. Alguém poderia comentar? 

  • Não seria VDI, pois o ''agradar'' está no sentido de satisfazer,contentar?

  • Penso que no sentido de "ser agradável", o verbo "agradar" é transitivo indireto. Dessa forma, a estrutura correta seria a que houvesse o emprego da preposição, o que caracteriza a letra C. Porém, a banca considerou justamente o contrário...

    Vamos indicar para comentário do professor, por gentileza!

  • Considero a alternativa errada também.

    Devereia ser correta a letra C.

    Agradar no sentido de satisfazer seria VTI.

    Socorro!!!

  • Letra C com certeza, aff...

     

    2) Agradar é transitivo indireto no sentido de causar agrado a, satisfazer, ser agradável a.  Rege complemento introduzido pela preposição "a".

     

    http://www.soportugues.com.br/secoes/sint/sint68.php

  • Gente, o verbo agradar nessa situação admite tanto a transitividade direta quanto a indireta, pois admite também o sentido de afagar/contentar, portanto é perfeitamente possível e correto utilizar a forma transitiva direta do verbo nesse contexto.

  • Amigos:

    É uma regência verbal importante, explico:

    Agradar
    Transitivo direto, com o sentido de "fazer agrado", "fazer carinho".
    Exemplo: Ela agradou o filho.
                               VTD       OD
    [...]estilos e letras que agradam o paciente[...]
                                            VTD          OD

    Transitivo indireto, com a preposição a, com o sentido de "ser agradável".
    Exemplo: O assunto não agradou ao homem.
                                                  VTI            OI

    Portanto, a alternativa B está correta.

  • Agradar no sentido de acariciar = VTD

    Agradar no sentido de satisfazer = VTI


    Foi dessa forma que aprendi. Assim, o gabarito deveria ser C e não B. 

     

  • Pra min que seria TI pois "Agradar" no sentido de satisfazer é transitivo indireto.

    Vai selecionar músicas que "satisfazem ou causam prazer' ao paciente......

    A mãe agrada o filho. -> agradar significa acariciar.
    A mãe agrada ao filho. -> agradar significa "causar agrado ou prazer", satisfazer.

     

    Fonte: http://www.soportugues.com.br/secoes/sint/sint61.php

  • FUI MUITO SECO NA LETRA C, MAS ACRETIDO QUE SEJA O GAB B PELA FORMA COMO ESTAVA ESCRITO NA FRASE, OU SEJA, SEM A PREPOSIÇÃO, DO CONTRÁRIO, EU NÃO VEJO OUTRA EXPLICAÇÃO PARA O VERBO DESTA QUESTÃO SER VTD.

     

  • Também fui "seca" na letra C.

     

  • AGRADAR

     

    a) No sentido “de acariciar”, “fazer carinho” é verbo transitivo direto.

    A Maria e a Laura agradavam futrica: o cãozinho de estimação.

    b) No sentido de “ser agradável”, exige complemento preposicionado (VTI). A prova não agradou aos candidatos despreparados.

     

    Professor Claiton Natal

  • Questão confusa e mal feita.

  • Pegadinha. O sentido muda se ficar "agradam ao paciente" (que é o que a gente errou, marcando a C).

    No entanto, gramaticalmente não está errado. Só que o sentido aceito ficou como se cantores, estilos e letras fizessem carinho no paciente.

  • Pessoal ! Agradar é uns dos poucos verbos que aceitam ambas as regências para o mesmo significado. Quando significar satisfazer.

     

    A regência, como tudo na língua, a pronúncia, a acentuação, a significação, etc., não é imutável. Cada época tem sua regência, de acordo com o sentimento do povo, o qual varia, conforme as condições novas da vida”, já dizia o filólogo e dicionarista Antenor Nascentes em 1960. E também dissera que agradar ocorre também com Transitividade Direta, sintaxe geralmente impugnada por gramáticos e puristas. Advertiu, contudo: Já foi trans. dir. A velha regência está voltando por analogia com "contentar". 

     

    Em resumo, pode-se usar corretamente o verbo agradar com o mesmo valor semântico como transitivo direto ou transitivo indireto, todos devidamente dicionarizados em:

     

    HOUAISS, Antonio. Dicionário Houaiss da Língua Portuguesa, 1.ed.: Rio de Janeiro: Objetiva, 2009.

    LUFT, Celso Pedro. Dicionário Prático de Regência Verbal - Nova Ortografia, 9. ed. : São Paulo: Ática, 2010, pg 43.

  • A regência verbal estuda a relação que se estabelece entre os verbos e os termos que os complementam (objetos diretos e objetos indiretos) ou caracterizam (adjuntos adverbiais).

    O estudo da regência verbal permite-nos ampliar nossa capacidade expressiva, pois oferece oportunidade de conhecermos as diversas significações que um verbo pode assumir com a simples mudança ou retirada de uma preposição. Observe:

    A mãe agrada o filho. -> 

    agradar significa acariciar.
    A mãe agrada ao filho. -> agradar significa "causar agrado ou prazer", satisfazer.

     

    Logo, conclui-se que "agradar alguém" é diferente de "agradar a alguém".

     

  • Nossam questão mal elaborada... para mim tem duas respostas corretas: b e d...

  • que lixo!!!!!!!! Desde quando música e livro fazem carinho em alguém??????? Não tem como dizer que esta gramaticalmente correto se o sentido exige a outra regência!!!!! Absurdo esse gabarito
  • Se estivesse organizado desta forma, ficaria bem mais fácil analisar a questão:

     

    agradar

    verbo

     

    transitivo direto, transitivo indireto e intransitivo

     

    1 ser agradável, transmitir satisfação a; dar prazer, contentar pronominal

    2 comprazer-se em; experimentar prazer, deleite; sentir-se encantado pronominal  2.1 sentir-se enamorado, tomar-se de amores transitivo direto

    3 Regionalismo: Nordeste do Brasil. azer agrados, carinhos; afagar.

     

    Agora releia o seguinte trecho do texto.

    (...) vai selecionar cantores, estilos e letras que agradam o paciente (...).

    cantores, estilos e letras..agradam o que? O paciente (objeto direto)

     

  • Questão muito mal elabora e confusa, especialmente no trecho que transcreve o significado do Houaiss. Se não colacassem os significados todos na mesma linha, ou se fizessem a pontuação com o uso de vírgulas, seria muito melhor.

     

    Errei essa questão e não concordo que a letra B seja a alternativa correta. 

     

    Segundo a explicação do professor Diogo Arrais:

     

    "1º. Agradar, no sentido de “ser agradável”, “aprazer”, “satisfazer”, é transitivo indireto; exige a preposição:

    “O serviço daquela empresa agradou A todos os clientes.”

     

    2º. Agradar, no sentido de “amimar”, “acarinhar”, é transitivo direto; não exige a preposição:

    “Por ser bom pai, não deixava de agradar o filho benquisto.”

     

    3º. Agradar, no sentido de “enamorar-se”, “simpatizar”, é verbo pronominal; possui o auxílio da preposição “de”. Vejamos um trecho escrito por Raul Pompeia:

    “Agradou-se da criança abandonada, tomando-a para criar; tanto se agradou da moça que lhe propôs casamento.”"

     

    O verbo, no contexto, é usado no sentido de "satisfazer", o que deixaria a letra C correta.

  • Fui na letra C e me ferrei. 

    Segundo consta no meu caderno, o verbo AGRADAR possui 2 regências. 

    Ele pode ser VERBO TRANSITIVO DIRETO, quando empregado no sentido de "Fazer agrados", "Acariciar", "Fazer Carinho".

    EX: O governo procura agradar o povo.

    Ele pode ser VERBO TRANSITIVO INDIRETO, quando empregado no sentido de "Satisfazer", "Contentar".

    EX: A política de preços não agradou aos consumidores.

  • Coloca essas bancas amadoras, não poderia ser diferente o resultado

  • não esqueçam que vocês podem fazer a seguinte pergunta :

    " cantores, estilos e letras " agradam o quê ? ( não precisa ser " a quem ' )...se a palavra agradar tem o significado de agradar a algo também... cantores, estilos e letras satisfazem o paciente...

    " cantores, estilos e letras " agradam " o paciente " ...o objeto se liga ao verbo de forma direta , ou seja...sem preposição..


ID
2478619
Banca
PUC-PR
Órgão
TJ-PR
Ano
2017
Provas
Disciplina
Português
Assuntos

                                Açaí faz bem para a cabeça

       Pesquisadores brasileiros e canadenses testam o potencial do fruto

                contra doenças neuropsiquiátricas, como a bipolaridade

Uma iguaria paraense com sucesso no Brasil todo, o açaí já foi associado ao melhor controle do colesterol e à prevenção do câncer. Agora, pesquisadores da Universidade Federal de Santa Maria, no Rio Grande do Sul, e da Universidade de Toronto, no Canadá, adicionam outra façanha à lista: a possível melhora no quadro de transtorno bipolar.

É que o extrato do fruto reverteu, em laboratório, uma disfunção nas mitocôndrias, organelas que produzem energia para as células – na doença, elas acabam liberando os perigosos radicais livres. “Além disso, houve redução na inflamação”, conta o biomédico  Alencar Kolinski Machado, um dos brasileiros envolvidos no projeto. “Sabemos que indivíduos bipolares têm uma ativação inflamatória crônica”, informa.

De acordo com Machado, é provável que o consumo do fruto (e não só do extrato) já traga vantagens. Um estudo demonstrou, por exemplo, que 120 mililitros do suco por dia promoveram um efeito anti-inflamatório capaz de amenizar a dor. O açaí na tigela cairia igualmente bem, pois contém a polpa do alimento. Basta evitar certos acompanhamentos, como xarope de guaraná e leite condensado. Prefira frutas naturais e um pouco de mel – e não abuse da granola.

Disponível em:  <http://super.abril.com.br/saude/acai-faz-bem-para-a-cabeca/>. Acesso em: 24/04/2017, às 19h11min.  

Textos que trazem relatos de divulgação científica normalmente usam elementos linguísticos que modalizam a informação a fim de que fique claro que os dados são inconclusivos ou ainda que se trata de conjecturas com base em provas e pesquisas. Esse é o caso do seguinte trecho do texto:

Alternativas
Comentários
  • Letra A

     

  • a) (...) é provável [conjectura] que o consumo do fruto (e não só do extrato) já traga vantagens.  

  • Para mim há duas alternativas corretas, pois o enunciado pede elementos linguísticos que modalizam a informação:

    * " a fim de que fique claro que os dados são inconclusivos" - a) (...) é provável que o consumo do fruto (e não só do extrato) já traga vantagens.

    * " ou ainda que se trata de conjecturas com base em provas e pesquisas" - c) É que o extrato do fruto reverteu, em laboratório, uma disfunção nas mitocôndrias (...)

  • obs.: colocar a resposta não ajuda em nada e sim como chegou à tal resposta!

     

    #ficadica

  • Conjectura = ato ou efeito de inferir ou deduzir que algo é provável, com base em presunções, evidências incompletas, pressentimentos; conjetura, hipótese, presunção, suposição.

  • Todas as afirmativas, com exceção da "A", trazem uma certeza ou um fato que ja ocorreu:

    a) (...) é provável que o consumo do fruto (e não só do extrato) já traga vantagens.  (o adjetivo provavel carcteriza algo que tem chances de acontecer, ou seja, não traz uma certeza)

    b) (...) o açaí já foi associado ao melhor controle do colesterol e à prevenção do câncer. (só pelo fato de estar no passado demonstra que é algo que já ocorreu)

    c) É que o extrato do fruto reverteu, em laboratório, uma disfunção nas mitocôndrias (...).  (algo que ja acontenceu também)

    d) (...) na doença, elas acabam liberando os perigosos radicais livres. (é uma afirmação)

    e) Basta evitar certos acompanhamentos, como xarope de guaraná e leite condensado. (verbo no imperativo, indicando ordem)

     

     

     

  • Vendedor hotmart, monetizze companhaia esee nao e o lugar.

ID
2478622
Banca
PUC-PR
Órgão
TJ-PR
Ano
2017
Provas
Disciplina
Português
Assuntos

                                Açaí faz bem para a cabeça

       Pesquisadores brasileiros e canadenses testam o potencial do fruto

                contra doenças neuropsiquiátricas, como a bipolaridade

Uma iguaria paraense com sucesso no Brasil todo, o açaí já foi associado ao melhor controle do colesterol e à prevenção do câncer. Agora, pesquisadores da Universidade Federal de Santa Maria, no Rio Grande do Sul, e da Universidade de Toronto, no Canadá, adicionam outra façanha à lista: a possível melhora no quadro de transtorno bipolar.

É que o extrato do fruto reverteu, em laboratório, uma disfunção nas mitocôndrias, organelas que produzem energia para as células – na doença, elas acabam liberando os perigosos radicais livres. “Além disso, houve redução na inflamação”, conta o biomédico  Alencar Kolinski Machado, um dos brasileiros envolvidos no projeto. “Sabemos que indivíduos bipolares têm uma ativação inflamatória crônica”, informa.

De acordo com Machado, é provável que o consumo do fruto (e não só do extrato) já traga vantagens. Um estudo demonstrou, por exemplo, que 120 mililitros do suco por dia promoveram um efeito anti-inflamatório capaz de amenizar a dor. O açaí na tigela cairia igualmente bem, pois contém a polpa do alimento. Basta evitar certos acompanhamentos, como xarope de guaraná e leite condensado. Prefira frutas naturais e um pouco de mel – e não abuse da granola.

Disponível em:  <http://super.abril.com.br/saude/acai-faz-bem-para-a-cabeca/>. Acesso em: 24/04/2017, às 19h11min.  

Sobre a estrutura sintática desse texto, assinale a alternativa CORRETA.

Alternativas
Comentários
  • Prefira você e abuse você (O "você" se conjuga na 3a pessoa --> Ele) 

    Para encontrar o imperativo afirmativo, utilize, em regra, o presente do subjuntivo: "Maria quer que você prefira bolo a leite" ; "Maria quer que você abuse das frutas"  

    Exceção: 2a pessoa do singular e do plural, onde se usa o presente do indicativo sem o S.

    Tu preferes --> Prefere (tu)

    Tu abusas --> Abusa (tu)

    Logo, o sujeito de ambas as formas trazidas no texto é o "você". 

    As transitividades são diferentes. O verbo preferir é transitivo direito e indireto: Prefiro pão a doce.

    O verbo abusar é transitivo indireto: Abusei dos doces esse fim de semana.

    Qualquer erro, podem comentar a vontade. Bons estudos!!!! 

     

     

     

  • "NÃO ABUSE" Isso não é imperativo afirmativo, mas sim imperativo negativo. Errei por pensar assim. 

  • O que há de errado com a letra A?

  • A) A letra A é um adjunto adverbial de lugar e não um aposto explicativo.

    B) verbo haver no sentido de ocorrer/existir/acontecer/ tempo transcorrido é impessoal. Não varia, não vai para o plural.

    C)  ,conta o  biomédico  Alencar Kolinski Machado, é um aposto explicativo e não restritivo.

    D) correta, que os colegas já explicaram.

    E) o QUE na primeira ocorrência é sujeito e na segunda é obj.direto. ( essa aqui não tenho certeza da primeira ocorrência do QUE, então, quem souber me ajude aí.

    Bjs #força

  • Entendo que o erro da assertiva A é afirmar que o aposto poderia ser separado por " ; ". Não pode!

    Acredito que a separação possa se dar apenas por vírgula ou dois pontos.

  • Qual o erro da letra E? Por que o primeiro "que" seria um sujeito?

  • Alex Large, o primeiro "que" (é provável que o consumo do fruto...) introduz uma Oração Subordinada Substantiva Subjetiva, pois podemos substituir a oração por ISSO é provável, sendo assim, "que o consumo do fruto..." é Sujeito

    O segundo "que" (Um estudo demonstrou, por exemplo, que 120 mililitros do suco...) introduz uma Oração Subordinada Substantiva Objetiva Direta, pois podemos substituir a oração por Um estudo demonstrou ISSO, sendo assim, "que 120 mililitros do suco..." é Objeto Direto.

    Espero que tenha ajudado. Bons estudos pessoal.

  • Livro da Flávia Rita diz:

    " O aposto pode ser isolado por vírgulas, por travessões, por parênteses ou por dois pontos."

    Então acredito que o erro da letra A seja isso... concordo com a elaine pieri.

     

  • Agente focada, não sei onde você tirou que é a letra "a" é um adjunto adverbial.

     

    Aposto

    Pode aparecer antes ou depois do termo ao qual se refere, bem como ser destacado ou não por sinais de pontuação, como vírgula, dois-pontos ou travessão. Pode ainda ser precedido ou não de preposições ou de expressões explicativas (isto é, como,...).

    Exemplos de aposto

    Luís de Camões, importante poeta português, escreveu poemas sobre os descobrimentos portugueses.

    Aquelas duas meninas – a Camila e a Tatiana – ficaram ajudando no fim da festa.

    A professora mais antiga da escola, D. Cristina é respeitada por todos.

    Visitei a cidade de Salvador e adorei!

    Apenas tenho um único objetivo de vida: ser muito feliz!

     

     

    https://www.normaculta.com.br/aposto/

  • LETRA A)O termo “Uma iguaria paraense com sucesso no Brasil todo” é um aposto explicativo que aparece invertido com o termo a que se refere, motivo pelo qual a vírgula que aparece depois dele poderia ser substituída por ponto e vírgula.

    Aposto é um termo que se junta a outro de valor substantivo ou pronominal para explicá-lo ou especificá-lo melhor. Vem separado dos demais termos da oração por vírgula, dois-pontos ou travessão. Nada de ponto e vírgula.

    LETRA E) O conectivo que, nas duas ocorrências destacadas no texto, inicia orações que desempenham a mesma função sintática: são objetos diretos das formas verbais presentes nas suas orações principais.  
    Dica: Troque por ISTO

    ..."é provável que o consumo do fruto (e não só do extrato) já traga vantagens."

    "..é provável ISTO."

     Veja que a oração tem função de sujeito.


    "Um estudo demonstrou, por exemplo, que 120 mililitros do suco por dia promoveram um efeito anti-inflamatório capaz de amenizar a dor."
    "Um estudo demonstrou ISTO"
     Veja que a oração tem função de objeto direto.
     

  • Gente alguém pode me explicar pq o verbo abuse não é do imperativo negativo?
  • LETRA A É UM APOSTO EXPLICATIVO TOPICALIZADO E O ERRO, COMO JÁ MENCIONADO PELOS COLEGAS, É QUE ESSA ESTRUTURA NÃO SUPORTA PONTO E VÍRGULA. PONTO E VÍRGULA SERVE PARA OCASIONAR UMA PAUSA MAIOR QUE A VÍRGULA, DESSA FORMA, REFLETE ÊNFASE E CLAREZA AO QUE ESTÁ SENDO DITO. SERVE PARA ENUMERAÇÕES, SEPARAR ORAÇÕES COORDENADAS.  

  • Concordo com o Messias Aguiar.

    Por que é imperativo afirmativo?

  • Acertei a letra D, justamente pela questão referir-se: a mesma classificação de tipo de sujeito, "mas têm transitividades diferentes."

    E não por 'indicar'  imperativo negativo ou afirmativo.  Revejam a transitividade.

  • reforcando os pedidos, por que é imperativo afirmativo?

  • Letra C: Uma vírgula poderia ter sido usada antes de “Alencar Kolinski Machado”, já que esse termo é um aposto restritivo, função sintática que normalmente aparece separada do restante da oração por algum tipo de pontuação. ERRADO.

     

    Trata-se de aposto ESPECIFICATIVO, o qual não é marcado por sinais de pontuação. 

     

    > Aposto especificativo: Especifica um substantivo de uso genérico. Geralmente é um nome próprio de pessoa ou lugar e não vem isolado por vírgulas.

     

    Fonte: http://www.gabarite.com.br/dica-concurso/126-aposto-explicativo-enumerativo-resumitivo-e-especificativo

     

     

     

  • não entendi essa transitividade falada na alternativa d.

  • Texto: "Prefira frutas naturais e um pouco de mel – e não abuse da granola."

    Questão: As formas verbais “Prefira” e “abuse”, flexionadas na terceira pessoa do singular do imperativo afirmativo, apresentam a mesma classificação de tipo de sujeito, mas têm transitividades diferentes.  

    Minha opinião: questão passível de anulação pois está formulada incorretamente. Os dois verbos estão no Imperativo. O primeiro no Imperativo Afirmativo ( "prefira" ) e o segundo no Imperativo Negativo ( "não abuse" ).

    As transitividades são diferentes (OK - correto): o verbo preferir é VTDI e o verbo abusar é VTI.

  • Prefira: verbo bitransitivo. Pode pedir preposição ou não.

    Abuse: transitivo indireto. Pede preposição. Quem abusa, abusa "de".

     

    Mas esse lance do "imperativo negativo", tá osso. Fui por eliminação das outras.

  • Alguém explica pq é subjetiva e não predicativa, a primeira ocorrência do vacábulo "que"?

  • "Não abuse" está flexionado no imperativo afirmativo? Anulem essa questão por favor.

  • Para esclarecer a dúvida de muitos colegas aqui a respeito do imperativo, segue explicação:

     A frase "não abuse" para ser imperativo negativo deveria ser conjugada da seguinte maneira: não abuses.

    A regra para a conjugação no imperativo negativo é utilizar o verbo no presente do subjuntivo respectivamente como se conjuga.

     

    Diferente, é como se conjuga o imperativo afirmativo; utiliza-se a conjugação do presente do indicativo sem a letra s do final.

    ex. tu abusas (presente do indicativo)

    abusa tu (imperativo afirmativo) .

     

  • banca mulamba!!! considerou imperativo afirmativo e negativo as mesmas coisas!!! vai..........

  • Por que a questão foi anulada????

  • Questão mulamba, foi anulada


ID
2478625
Banca
PUC-PR
Órgão
TJ-PR
Ano
2017
Provas
Disciplina
Português
Assuntos

                                Açaí faz bem para a cabeça

       Pesquisadores brasileiros e canadenses testam o potencial do fruto

                contra doenças neuropsiquiátricas, como a bipolaridade

Uma iguaria paraense com sucesso no Brasil todo, o açaí já foi associado ao melhor controle do colesterol e à prevenção do câncer. Agora, pesquisadores da Universidade Federal de Santa Maria, no Rio Grande do Sul, e da Universidade de Toronto, no Canadá, adicionam outra façanha à lista: a possível melhora no quadro de transtorno bipolar.

É que o extrato do fruto reverteu, em laboratório, uma disfunção nas mitocôndrias, organelas que produzem energia para as células – na doença, elas acabam liberando os perigosos radicais livres. “Além disso, houve redução na inflamação”, conta o biomédico  Alencar Kolinski Machado, um dos brasileiros envolvidos no projeto. “Sabemos que indivíduos bipolares têm uma ativação inflamatória crônica”, informa.

De acordo com Machado, é provável que o consumo do fruto (e não só do extrato) já traga vantagens. Um estudo demonstrou, por exemplo, que 120 mililitros do suco por dia promoveram um efeito anti-inflamatório capaz de amenizar a dor. O açaí na tigela cairia igualmente bem, pois contém a polpa do alimento. Basta evitar certos acompanhamentos, como xarope de guaraná e leite condensado. Prefira frutas naturais e um pouco de mel – e não abuse da granola.

Disponível em:  <http://super.abril.com.br/saude/acai-faz-bem-para-a-cabeca/>. Acesso em: 24/04/2017, às 19h11min.  

Assinale a opção em que todas as palavras a seguir, retiradas do texto, são acentuadas em virtude da mesma regra.

Alternativas
Comentários
  • Gabarito: Letra E
    Contém: oxítona terminada em EM

    Guaraná: oxítona terminada em A

    Canadá: oxítona terminada em A

     

    Obs.: As palavras não precisam terminar necessariamente na mesma letra para estarem na mesma regra. Precisam apenas compartilhar a mesma classificação quanto à tonicidade. 

  • Explicação

    A) A - ÇA - Í (Acetuada por ser HIATO); (Acentuada por ser MONOSSILABO TÔNICO TERMINADO EM -A); TÊM (Não se trata de acento tônico, mas sim ACENTO DIFERENCIAL referente ao plural) - INCORRETA

     

    B) MI - TO - CÔN - DRI - AS (Acentuada por ser PROPAROXÍTONA); CRÔ - NI - CAS (Acentuada por ser PROPAROXÍTONA); PRO - VÁ - VEL (Acentuada por ser PAROXÍTONA TERMINADA EM -L) - INCORRETA

     

    C) POS - SÍ - VEL (Acentuada por ser PAROXÍTONA TERMINADA EM -L); IN - DI - VÍ - DU - OS (Acentuada por ser PROPAROXÍTONA); BI - O - MÉ - DI - COS (Acentuada por ser PROPAROXÍTONA) - INCORRETA

     

    D) CÂN - CER (Acentuada por ser PAROXÍTONA TERMINADA EM -R); A - LÉM (Acentuada por ser OXÍTONA TERMINADA EM -EM); LA - BO - RA - TÓ - RI - O (Acentuada por ser PROPAROXÍTONA) - INCORRETA

     

    E) CON - TÉM (Acentuada por ser OXÍTONA TERMINADA EM -EM); GUA - RA - NÁ (Acentuada por ser OXÍTONA TERMINADA EM -A); CA - NA - DÁ (Acentuada por ser OXÍTONA TERMINADA EM -A) - CORRETA

  • Gabarito E) Todas as palavras são oxítonas.

  • Gabarito: LETRA E

     

    a) ERRADA! açaí (hiato em que a segunda sílaba tônica é i) – já (monosssílabo terminado em a) – têm (acento diferencial). 

     b) ERRADA! mitocôndrias (paroxítona termina em ditongo) – crônica (proparoxítona)– provável (paroxítona terminada em L). 

     c) ERRADA! possível (paroxítona terminada em L) – indivíduos (paroxítona terminada em ditongo)– biomédicos (propoaroxítona).  

     d) ERRADA! câncer (paroxítona terminada em r)– além (oxítona terminada em EM) – laboratório (paroxítona termina em ditongo).

     e)CORRETA! contém – guaraná – Canadá (oxítonas terminadas em EM e A). 

  • VIDE:    https://www.youtube.com/watch?v=hgF5_RC6H8M

     

    DECIO TERROR    26min

     

                           MACETE: CHAMA A PALAVRA EM VOZ ALTA !!!

     

    1-     OXÍ   -     TONAS   =       SÍLABAS MAIS FORTE      A ÚLTIMA  (OXI)    FO -   GA  -  RÉU

    a-    TÉ

     

                   OXÍTONAS:       Sílaba tônica:      ÚLTIMA

     

    Acentuam-se as OXÍTONAS terminadas em:  A (s),  E (s), O (s), EM e ENS

     

    A   (s):

    sofá, sofás

    E   (s):

    Jaca-ré, vocês

                       O   (s):

    paletó, avós

    EM, ENS:

    ninguém, armazéns

     

    ALGU -    ÉM,  TAMBÉM

    Amap -  á          /           Já-ca-re-pa-guá

    Caf  -   é

    Sor - ri

    Cip -   ó

    Uru -  bu

     

    VO- CÊ   -     D Ê  -      Utiliza-se o acento circunflexo nas palavras oxítonas terminadas em:

    -    VOGAIS TÔNICAS fechadas que se grafam -e ou –o e

    -     MONOSSÍLABOS TÔNICOS:

     

                       JÁ, PARÁ, MÁS, DÓ, VÊ, VOVÔ, DÊ

     

    HIATO

    Baú, saída, egoísta

    DestruÍ-la

     

    EXCEÇÃO:       NHA – Rainha, coroinha

     

    .......................

     

     

                        A reforma ortográfica só atingiu as       PAROXÍTONAS.

     

    2-        PAROXÍTONAS     =       SÍLABAS MAIS FORTE A PENÚLTIMA    

     

       JA-NE-LA

     

     

    DITONGO abertos   =    DUAS  VOGAIS  JUNTAS na mesma sílaba

     

    ACENTUAM-SE       AS       PAROXÍTONAS TERMINADAS EM DITONGO  !        VOGAL SEMIVOGAL

     

    ÁGUA, INDIVÍDUOS, PRECÁRIAS, SÉRIE, HISTÓRIA, HOMOGÊNEA, MÉDICO, BROMÉLIA

     

    Lín-gua - É acentuada porque é paroxítona terminada em ditongo.

     

    ÓRFÃO

    A palavra “rubrica” tem apenas uma forma correta para ser escrita, que é a paroxítona.

     

    ZÍ -  PER

    PRÍNCIPE

    Mi-nis--ri-o

    Me-sa

    Ní-ve-l

    Ca-rá-ter

    Livro

    Á-lb-um

    Lá-pis

    PAROXÍTONAS

    Sílaba tônica: penúltima

     

     

    Acentuam-se as paroxítonas TERMINADAS em:   i, is   L, N, R, OS, X, US,

     

    l

    cil

    n

    Pó-len

    r

    caver

    ps

    Bí-ceps

    x

    Tó-rax

    us

    Ví-rus

    i, is

    ri, pis

    om, ons

    iândom, íons

    um, uns

    Ál-bum, ál-buns

    ã(s), ão(s)

    órfã, órfãs, órfão, órfãos

    ditongo oral (seguido ou não de s)

    quei,neis

     

    A maioria das paroxítonas termina em -A, -E, -O, -EM, podendo ou não ser seguidas de "s". Essas paroxítonas, por serem maioria, NÃO são acentuadas graficamente.

    US, UM(S), L, I, R, ÃO(Ã), X, N, PS, ON, e DITONGO (DUAS VOGAIS), são acentuadas.

     

    Observações:

    1)      As paroxítonas terminadas em "n" são acentuadas (hífen), mas as que terminam em "ens", não (HIFENS, JOVENS).

    2)      Não são acentuados os prefixos terminados em "i "e "r" (semi, super).

    3)              Acentuam-se as PAROXÍTONAS terminadas em DITONGOS CRESCENTES: ea(s), oa(s), eo(s), ua(s), ia(s), ue(s), ie(s), uo(s), io(s).

    Exemplos:

    várzea, mágoa, óleo, régua, férias, tênue, cárie, ingênuo, início

  • A) Oxítona - Monossilabo - Monossilabo
    B) Paroxítona - Proparoxítona - Paroxítona
    C) Paroxítona - Paroxítona - Proparoxítona
    D) Paroxítona - Oxítona - Paroxítona
    E) Oxítona - Oxítona - Oxítona (CORRETO)

  • Acentuam-se as oxítonas terminadas em: A - E - O - EM - ENS

    Paroxítonas terminadas em: L  N  R  X  PS  Ã (ÃS)  ÃO (ÃOS)  EI(EIS)   I(IS)   UM(UNS)  US

    TODAS as proparoxítonas são acentuadas.

  • açaí - oxítona terminada em hiato

    - monossílabos

    têm -  monossílabos - acento diferencial - verbo ter

    mitocôndrias - paroxítona terminada em ditongo crescente

    crônica - proparoxítonas (todas são acentuadas)

    provável - paroxítona terminada em L

    possível - paroxítona terminada em L

    indivíduos - paroxítona terminada em ditongo crescente

    biomédicos - proparoxítonas (todas são acentuadas)

    câncer - paroxítona terminada em R

    além - oxítona terminada em EM

    laboratório - paroxítona terminada em ditongo crescente

    contém - oxítona terminada em EM

    guaraná - oxitona terminada em A

    Canadá - oxitona terminada em A

  • LUMPSLEIRUXÃO

  • LETRA E,

    OXÍTONAS TERMINADAS A,E,O COM A TÔNICA FORTE LEVA ACENTO (guaraná – Canadá) 

    OXÍTONAS TERMINAS COM EM OU ENS LEVAM ACENTOS COM ESSAS TÔNICA FORTE (contém) 

    LETRA E contém – guaraná – Canadá. 

  • DESCOMPLICA: seus problemas acabaram.

     

    VIDE:    https://www.youtube.com/watch?v=hgF5_RC6H8M

     

    DECIO TERROR    26min

     

                           MACETE: CHAMA A PALAVRA EM VOZ ALTA !!!

     

    De acordo com a tonicidade, as palavras são classificadas como:

    Oxítonas: são aquelas cuja sílaba tônica recai sobre a ÚLTIMA SÍLABA.

    Ex.: café – coração – cajá – atum – caju – papel

     

    Paroxítonas: são aquelas em que a sílaba tônica evidencia-se na PENÚLTIMA SÍLABA.

     

    Ex.: útil – tórax – táxi – leque – retrato – passível

     

    Proparoxítonas: são aquelas em que a sílaba tônica evidencia-se na ANTEPENÚLTIMA sílaba.

    Ex.: lâmpada –  câmara –  tímpano – médico – ônibus

    Acentuação gráfica

    Regras fundamentais:

    Proparoxítonas: todas são acentuadas. Ex.: analítico, hipérbole, jurídico, cólica.

    Q406972

     

    lá. (antepenúltima)      gri .  (penúltima)         mas (última)        –       

    .    (antepenúltima)       vi.   (penúltima)       das (última)     –                 

       as.tro..   (antepenúltima)      mi. (penúltima)    cas  (última)

     

    de.mo.crá.ti.ca – é.ti.ca – ú.ni.co = todas são proparoxítonas

     

     

     

     

     

    ·         Palavras oxítonas: acentuam-se todas as oxítonas terminadas em "a", "e", "o", "em", seguidas ou não do plural(s). Ex.: Pará – café(s) – cipó(s) – armazém(s)

     

    Essa regra também é aplicada aos seguintes casos:

    → Monossílabos tônicos terminados em "a", "e", "o", seguidos ou não de “s”.

    Ex.: pá – pé – dó –  há

    → Formas verbais terminadas em "a", "e", "o" tônicos seguidas de lo, la, los, las.

    Ex.: respeitá-lo – percebê-lo – compô-lo.
     

    Paroxítonas:  Acentuam-se as palavras paroxítonas terminadas em:

                                                      ROUXINOL

    Acentuam-se as paroxítonas TERMINADAS em:   i, is   L, N, R, OS, X, US,

     

    i, is

    Ex.: táxi – lápis – júri

    us, um, uns

    Ex.: vírus – álbuns – fórum

    l, n, r, x, ps

    Ex.: automóvel – elétron - cadáver – tórax – fórceps

    ã, ãs, ão, ãos 

    Ex.: ímã – ímãs – órfão – órgãos



    Ditongo oral, crescente ou decrescente, seguido ou não de s.

    Ex.: água – pônei – mágoa – jóquei

    Regras especiais:

      Os ditongos de pronúncia aberta "ei", "oi", que antes eram acentuados, perderam o acento com o Novo Acordo. Veja na tabela a seguir alguns exemplos:

  • Letra E, questão de presente para o candidato (risos).

  • Gab. E. Mas acho que o verbo "contém" está acentuado assim não porque é uma oxítona, mas sim porque é um aceno diferencial para diferenciá-lo do contêm( 3º do plural)t

  • Em E não é a mesma regra, porque nem todos os oxítonas são acentuadas. 

  • Tanto contém quanto contêm são oxítonas terminadas em E. Alternativa E.

  • Indivíduos >> Proparoxítonas ou Paroxítonas terminas em Ditongo?  Incrível como cada banca interpreta de uma forma diferente.

     

    A Professora Isabel inclusive, em vídeo sobre o assunto, fala que diversas bancas não respeitam o novo acordo ortográfico (que afirma ser Proparoxítona) e seguem construindo questões com a regra antiga.

     

    Para essa questão, esse pon to naõ faz tanta diferença pois a letra "E" contém apenas Oxítonas, mas é fácil encontrar questões em que você fica preso nesse ponto para determinar qual é a correta.

  • GABARITO: LETRA E

    ACRESCENTANDO:

    Regra de Acentuação para Monossílabas Tônicas:

    Acentuam-se as terminadas em -a(s), -e(s), -o(s).

    Ex.: má(s), trás, pé(s), mês, só(s), pôs…

    Regra de Acentuação para Oxítonas:

    Acentuam-se as terminadas em -a(s), -e(s), -o(s), -em(-ens).

    Ex.: sofá(s), axé(s), bongô(s), vintém(éns)...

     

    Regra de Acentuação para Paroxítonas:

    Acentuam-se as terminadas em ditongo crescente ou decrescente (seguido ou não de s), -ão(s) e -ã(s), tritongo e qualquer outra terminação (l, n, um, r, ns, x, i, is, us, ps), exceto as terminadas em -a(s), -e(s), -o(s), -em(-ens).

    Ex.: história, cáries, jóquei(s); órgão(s), órfã, ímãs; águam; fácil, glúten, fórum, caráter, prótons, tórax, júri, lápis, vírus, fórceps.

     

    Regra de Acentuação para Proparoxítonas:

    Todas são acentuadas .Ex.: álcool, réquiem, máscara, zênite, álibi, plêiade, náufrago, duúnviro, seriíssimo...


    Regra de Acentuação para os Hiatos Tônicos (I e U):

    Acentuam-se com acento agudo as vogais I e U tônicas (segunda vogal do hiato!), isoladas ou seguidas de S na mesma sílaba, quando formam hiatos.

    Ex.: sa-ú-de, sa-í-da, ba-la-ús-tre, fa-ís-ca, ba-ú(s), a-ça-í(s)...

    FONTE: A GRAMÁTICA PARA CONCURSOS PÚBLICOS 3ª EDIÇÃO FERNANDO PESTANA.


ID
2478628
Banca
PUC-PR
Órgão
TJ-PR
Ano
2017
Provas
Disciplina
Português
Assuntos

               Garota é encontrada vivendo entre macacos na Índia

Uma garota de aproximadamente oito anos foi encontrada entre macacos na floresta do santuário selvagem Katarniaghat, na Índia. As autoridades ainda não conseguiram identificar a família da menina, nem mesmo como ela foi parar ali, (1) mas ela já está sendo tratada em um hospital próximo. Apesar de possuir alguns ferimentos pelo corpo e estar subnutrida, (2) ela tem apresentado conduta agressiva, guinchando como um macaco quando se sente irritada.

A criança foi achada na companhia de três macacos enquanto a polícia fazia uma patrulha pela região. Os policiais contam que, no momento em que a encontraram, ela emitiu um grito parecido com o dos animais. "Ela estava com muito medo de nós, não conseguia falar, nem nos ouvir direito", explica um dos inspetores, Ram Avtar Singh.

A polícia, porém, não acredita que ela tenha passado anos ali, já que a floresta faz parte de um santuário local. A teoria levantada é que os pais a tenham abandonado na região devido aos problemas mentais da criança e algum tempo depois ela tenha sido encontrada.

Segundo eles, (3) a menina estava com alguns machucados nas pernas e no cotovelo, porém suas roupas não pareciam muito sujas. Ela foi encaminhada para o hospital local, onde vem sendo tratada nos últimos dois meses. Quando ela se recuperar totalmente, (4) a garota será enviada para o departamento de proteção à criança do governo.

"Quando chegou aqui, ela estava com muito medo de nós, esquivando-se como um animal, como um macaco", explicou o médico Dinesh Singh. "Nós cuidamos dela, demos banho, alimentamo-la e a limpamos. Ela estava desnutrida e pode ter comido o que os animais comiam. Às vezes, ela se irrita e precisamos acalmá-la; tem sido difícil lidar com ela."

A menina já come sozinha, apesar de não pegar os alimentos de um prato — é necessário colocá-los na cama para que ela se sinta à vontade para recolhê-los. De acordo com os médicos, ela também já consegue andar "como uma pessoa normal", sobre as duas pernas, mas já fez algumas tentativas de fuga. A polícia, agora, procura saber quem é a família da menina.

Disponível em: <http://revistagalileu.globo.com/Sociedade/noticia/2017/04/garota-e-encontrada-vivendo-entre-macacos-na-india.html> . Acesso em: 24/04/2017, às 19h21min (adaptado). 

Na construção desse texto, foram dadas algumas informações factuais, com base na descrição feita pelos participantes do episódio, e algumas suposições, a partir de evidências levantadas pela equipe que está tomando conta da garota. Selecione a alternativa a seguir que mostra um fato, e não uma hipótese, de acordo com os detalhes dados no texto.

Alternativas
Comentários
  • GABARITO: LETRA C

    O trecho a seguir responde a questão: "A criança foi achada na companhia de três macacos enquanto a polícia fazia uma patrulha pela região. Os policiais contam que, no momento em que a encontraram, ela emitiu um grito parecido com o dos animais. "Ela estava com muito medo de nós, não conseguia falar, nem nos ouvir direito", explica um dos inspetores, Ram Avtar Singh." Ou seja, isso é um FATO e não uma HIPÓTESE.

  • a) Duração da permanência da garota com os macacos.  

    ERRADA. (hipótese) A polícia, porém, não acredita que ela tenha passado anos ali, já que a floresta faz parte de um santuário local.

     

    b) ERRADA. (hipótese) Motivo pelo qual a garota estava vivendo na floresta. 

    A teoria levantada é que os pais a tenham abandonado na região devido aos problemas mentais da criança e algum tempo depois ela tenha sido encontrada.

     

    c) CERTA. (fato) Comportamento da garota no momento em que foi encontrada.  

    Os policiais contam que, no momento em que a encontraram, ela emitiu um grito parecido com o dos animais. "Ela estava com muito medo de nós, não conseguia falar, nem nos ouvir direito", explica um dos inspetores, Ram Avtar Singh.

     

    d) ERRADA. (hipótese) Alimentação da garota enquanto estava na floresta.  

    Ela estava desnutrida e pode ter comido o que os animais comiam.

     

    e) ERRADA. (hipótese) 

    Dados pessoais exatos referentes à garota.  

    A teoria levantada é que os pais a tenham abandonado na região devido aos problemas mentais da criança e algum tempo depois ela tenha sido encontrada.

     

  • coitada da menina...


ID
2478631
Banca
PUC-PR
Órgão
TJ-PR
Ano
2017
Provas
Disciplina
Português
Assuntos

               Garota é encontrada vivendo entre macacos na Índia

Uma garota de aproximadamente oito anos foi encontrada entre macacos na floresta do santuário selvagem Katarniaghat, na Índia. As autoridades ainda não conseguiram identificar a família da menina, nem mesmo como ela foi parar ali, (1) mas ela já está sendo tratada em um hospital próximo. Apesar de possuir alguns ferimentos pelo corpo e estar subnutrida, (2) ela tem apresentado conduta agressiva, guinchando como um macaco quando se sente irritada.

A criança foi achada na companhia de três macacos enquanto a polícia fazia uma patrulha pela região. Os policiais contam que, no momento em que a encontraram, ela emitiu um grito parecido com o dos animais. "Ela estava com muito medo de nós, não conseguia falar, nem nos ouvir direito", explica um dos inspetores, Ram Avtar Singh.

A polícia, porém, não acredita que ela tenha passado anos ali, já que a floresta faz parte de um santuário local. A teoria levantada é que os pais a tenham abandonado na região devido aos problemas mentais da criança e algum tempo depois ela tenha sido encontrada.

Segundo eles, (3) a menina estava com alguns machucados nas pernas e no cotovelo, porém suas roupas não pareciam muito sujas. Ela foi encaminhada para o hospital local, onde vem sendo tratada nos últimos dois meses. Quando ela se recuperar totalmente, (4) a garota será enviada para o departamento de proteção à criança do governo.

"Quando chegou aqui, ela estava com muito medo de nós, esquivando-se como um animal, como um macaco", explicou o médico Dinesh Singh. "Nós cuidamos dela, demos banho, alimentamo-la e a limpamos. Ela estava desnutrida e pode ter comido o que os animais comiam. Às vezes, ela se irrita e precisamos acalmá-la; tem sido difícil lidar com ela."

A menina já come sozinha, apesar de não pegar os alimentos de um prato — é necessário colocá-los na cama para que ela se sinta à vontade para recolhê-los. De acordo com os médicos, ela também já consegue andar "como uma pessoa normal", sobre as duas pernas, mas já fez algumas tentativas de fuga. A polícia, agora, procura saber quem é a família da menina.

Disponível em: <http://revistagalileu.globo.com/Sociedade/noticia/2017/04/garota-e-encontrada-vivendo-entre-macacos-na-india.html> . Acesso em: 24/04/2017, às 19h21min (adaptado). 

Assinale a alternativa CORRETA sobre os mecanismos de coesão sublinhados no texto, levando em consideração as recomendações da norma-padrão da língua portuguesa.

Alternativas
Comentários
  • A) ONDE = Lugar fixo; AONDE = sentido de movimento. NÃO pode ser substituido. 

    B) PARA QUE expressa finalidade mas NÃO pode ser substuído por AFIM de que.

    Vejamos: AFIM (junto)= é um adjetivo e significa igual, semelhante, parecido. Ex.: Você gosta de sushi, yakissoba ou afins?;

                   A FIM (separado) = FINALIDADE

    C) O conectivo NEM, na primeira ocorrência, expressa sentido de Confirmação;

    D) "APESAR DE" não tem nada a ver com "GRAÇAS A".... Tentem colocar o Graças A na questão e analise.
     

    GABARITO E

  • A conjunção adversativa "MAS" NÃO PODE SER DESLOCADA, MAS PODE SER SUBSTITUÍDA. 

  • Sacanagem dar exemplo com comida

  • tudo bem reconheço que errei, falta de atenção. É muito difícil manter a concentração, sem ressaltar que eu fiquei impressionada com a história da menina e não consegui analisar o texto...

  • NEM

    conjunção coordenativa ADITIVA (não adversativa)

    Conjunções aditivas são conjunções coordenativas que expressam adição.

    As conjunções coordenativas aditivas ligam duas orações em que a segunda oração expressa um acréscimo da ideia iniciada na primeira oração. As principais conjunções aditivas são a conjunção e, usada para indicar uma adição com sentido positivo, e a conjunção nem, usada para indicar uma adição com sentido negativo. Além dessas duas, existem outras locuções conjuntivas aditivas.

    FONTE: https://www.normaculta.com.br/conjuncoes-aditivas/


ID
2478634
Banca
PUC-PR
Órgão
TJ-PR
Ano
2017
Provas
Disciplina
Português
Assuntos

               Garota é encontrada vivendo entre macacos na Índia

Uma garota de aproximadamente oito anos foi encontrada entre macacos na floresta do santuário selvagem Katarniaghat, na Índia. As autoridades ainda não conseguiram identificar a família da menina, nem mesmo como ela foi parar ali, (1) mas ela já está sendo tratada em um hospital próximo. Apesar de possuir alguns ferimentos pelo corpo e estar subnutrida, (2) ela tem apresentado conduta agressiva, guinchando como um macaco quando se sente irritada.

A criança foi achada na companhia de três macacos enquanto a polícia fazia uma patrulha pela região. Os policiais contam que, no momento em que a encontraram, ela emitiu um grito parecido com o dos animais. "Ela estava com muito medo de nós, não conseguia falar, nem nos ouvir direito", explica um dos inspetores, Ram Avtar Singh.

A polícia, porém, não acredita que ela tenha passado anos ali, já que a floresta faz parte de um santuário local. A teoria levantada é que os pais a tenham abandonado na região devido aos problemas mentais da criança e algum tempo depois ela tenha sido encontrada.

Segundo eles, (3) a menina estava com alguns machucados nas pernas e no cotovelo, porém suas roupas não pareciam muito sujas. Ela foi encaminhada para o hospital local, onde vem sendo tratada nos últimos dois meses. Quando ela se recuperar totalmente, (4) a garota será enviada para o departamento de proteção à criança do governo.

"Quando chegou aqui, ela estava com muito medo de nós, esquivando-se como um animal, como um macaco", explicou o médico Dinesh Singh. "Nós cuidamos dela, demos banho, alimentamo-la e a limpamos. Ela estava desnutrida e pode ter comido o que os animais comiam. Às vezes, ela se irrita e precisamos acalmá-la; tem sido difícil lidar com ela."

A menina já come sozinha, apesar de não pegar os alimentos de um prato — é necessário colocá-los na cama para que ela se sinta à vontade para recolhê-los. De acordo com os médicos, ela também já consegue andar "como uma pessoa normal", sobre as duas pernas, mas já fez algumas tentativas de fuga. A polícia, agora, procura saber quem é a família da menina.

Disponível em: <http://revistagalileu.globo.com/Sociedade/noticia/2017/04/garota-e-encontrada-vivendo-entre-macacos-na-india.html> . Acesso em: 24/04/2017, às 19h21min (adaptado). 

A seguir, você encontra quatro motivos para o emprego da vírgula e as quatro ocorrências da vírgula numeradas no texto, de (1) a (4). Relacione o motivo a cada uma delas e selecione a alternativa que completa os parênteses CORRETAMENTE, de cima para baixo.

A. Isolamento de adjunto adverbial.

B. Separação de oração coordenada.

C. Isolamento de oração subordinada adverbial.

D. Separação de oração reduzida.


( ) Vírgula em (1).

( ) Vírgula em (2).

( ) Vírgula em (3).

( ) Vírgula em (4).

Alternativas
Comentários
  • 1- a vírgula separa oração coordenada sindética adversativa (ali, mas ela já está sendo...)

    2- a vírgula separa oração subordinada reduzida de infinitivo (Apesar de possuir alguns ferimentos pelo corpo e estar subnutrida, (2) ela tem apresentado...)

    3- a vírgula isola adjunto adverbial de conformidade (Segundo eles, (3) a menina estava...)

    4- a vírgula isola oração subordinada adverbial temporal (Quando ela se recuperar totalmente, (4) a garota será enviada...)

  • Parabéns Erica Sousa pelo excelente comentário, muito obribado !

     

    GAB     B

     

     

    Q590406       Q817676    Q629834

     

    TEMPORAIS:       QUANDO,    EIS QUE,     MAL = LOGO QUE,   APENAS, SEMPRE QUE, NO MOMENTO QUE

     

    CONCESSÃO:    EMBORA,      A DESPEITO,  MESMO QUE, CONQUANTO,  AINDA QUE, APESAR DE, POSTO QUE

     

    PROPORCIONAL :      ENQUANTO    =             AO PASSO QUE, À MEDIDA QUE

     

    CONFORMATIVA:       COMO     SEGUNDO, CONSOANTE,  CONFORME

     

    CONDICÃO:    DESDE QUE, CONTATO, SE NÃO QUANDO NÃO

     

     

    COMPARATIVO:                    QUE =  DO QUE          Esta casa é mais alta (que = DO QUE) a outra

                                                   ASSIM COMO = COMPARAÇÃO

     

             ADIÇÃO:         NÃO APENAS

                                    TAMBÉM NÃO

                                     OUTROSSIM

                                      MAS = E

     

            PROPORCIONAIS  =      À MEDIDA QUE, AO PASSO, TANTO MAIS

     

    Q478660

                                        ALIÁS      =         ALÉM DISSO  

     

     

     

  • 1B - nem mesmo como ela foi parar ali,

    Separação de oração coordenada sindética aditiva

    2D -  Embora possua alguns ferimentos pelo corpo e esteja subnutrida,

    oração subordinada adverbial concessiva reduzida de infinitivo

    3A - Segundo eles...

    Isolamento de adjunto adverbial de conformidade

    4C - Quando ela se recuperar totalmente, 

    Isolamento de oração subordinada adverbial temporal

    *Tive um pouco de dificuldade em transformar a oração reduzida em desenvolvida, alguém poderia confirmar se desenvolvi a oração corretamente? 

    *Achei esse artigo e me parece que na desenvolvida o professor sempre troca por "embora" =>  http://www.linguativa.com.br/laphp/conteudo/tipoDicionario.php?categoriaPrograma=2024&chamada=47569

    ADVERBIAL CONCESSIVA REDUZIDA DE INFINITIVO

    Apesar de, a despeito de

    Apesar de ser discriminado, conseguiu concluir o curso superior.

    Oração principal: "conseguiu concluir o curso superior".

    Oração subordinada adverbial concessiva reduzida de infinitivo: "Apesar de ser discriminado".

    Período com oração desenvolvida: Embora fosse discriminado, conseguiu concluir o curso superior.

    Desde já, agradeço!

     

     

  • Coloquei na cabeça que " Segundo eles, (3) a menina estava com alguns machucados nas pernas e no cotovelo" era conformidade..

  • Essa questão não está no assunto certo...

  • Não está no assunto certo, como dito pelo colega, fora o fato de tomar uns 15 minutos do candidato =/.

  • Orações Reduzidas

    > O verbo sempre aparecerá em uma de suas formas nominais: gerúndio, particípio, infinitivo;

    > NUNCA são iniciadas por conjunções (no caso das substantiva e adverbiais), nem por pronomes relativos ( no caso das adjetivas);

    > Normalmente podem ser reescritas (desenvolvidas) com esses conectivos;

    > Podem ser iniciadas por preposição ou locução prepositiva.

     

    O entendimento da colega Grá Concurseira está correto, pois a frase desenvolvida fica "Embora (conjunção concessiva) possua alguns ferimentos..."

     

     

  • Esta seria uma questão que tomaria muito meu tempo, com certeza eu a deixaria de lado e se eu estivesse com tempo sobrando depois de terminar minha prova, voltava para resolver.

  • 1) separação de oração coordenada, mais precisamente, adversativa "... ela foi parar alimas ela já está sendo ..."

    2) separação de orações reduzidas, mais precisamente, de particípio " ... e estar subnutrida, ela tem apresentado ..."

    3) isolamento de adjunto adverbial, mais rpecisamente, de conformidade "Segundo eles, a menina ..."

    4) isolamento de oração subordinada adverbial, mais precisamente, temporal "Quando ela se recuperar totalmente, a garota será..."

     

    Quem escolheu a busca não pode recusar a travessia - Guimarães Rosa

    ------------------- 

    Gabarito: B

  • GABARITO: B

  • Gabarito''B''.

    A. Isolamento de adjunto adverbial.

    B. Separação de oração coordenada.

    C. Isolamento de oração subordinada adverbial.

    D. Separação de oração reduzida.

    (B ) Vírgula em (1).

    (D ) Vírgula em (2).

    (A ) Vírgula em (3).

    (C ) Vírgula em (4). 

    Estudar é o caminho para o sucesso.

  • Se eu ao menos entendesse o que a questão quer já seria bom demais rsrsrs

  • Questão chata, mas gratificante. Não sabia se o "Segundo eles," era um adjunto adverbial ou atuava como O.S.A. Conformativa, mas o "Quando ela se recuperar" ajudou a acertar.


ID
2478637
Banca
PUC-PR
Órgão
TJ-PR
Ano
2017
Provas
Disciplina
Português
Assuntos

                  LEI Nº 8.069, DE 13 DE JULHO DE 1990.

                          Das Disposições Preliminares

[...]

Art. 3º A criança e o adolescente gozam de todos os direitos fundamentais inerentes à pessoa humana, sem prejuízo da proteção integral de que trata esta Lei, assegurando-se-lhes, por lei ou por outros meios, todas as oportunidades e facilidades, a fim de lhes facultar o desenvolvimento físico, mental, moral, espiritual e social, em condições de liberdade e de dignidade.

Parágrafo único. Os direitos enunciados nesta Lei aplicam-se a todas as crianças e adolescentes, sem discriminação de nascimento, situação familiar, idade, sexo, raça, etnia ou cor, religião ou crença, deficiência, condição pessoal de desenvolvimento e aprendizagem, condição econômica, ambiente social, região e local de moradia ou outra condição que diferencie as pessoas, as famílias ou a comunidade em que vivem. (incluído pela Lei nº 13.257, de 2016).

Art. 4º É dever da família, da comunidade, da sociedade em geral e do poder público assegurar, com absoluta prioridade, a efetivação dos direitos referentes à vida, à saúde, à alimentação, à educação, ao esporte, ao lazer, à profissionalização, à cultura, à dignidade, ao respeito, à liberdade e à convivência familiar e comunitária.

Parágrafo único. A garantia de prioridade compreende:

A) primazia de receber proteção e socorro em quaisquer circunstâncias;

B) precedência de atendimento nos serviços públicos ou de relevância pública;

C) preferência na formulação e na execução das políticas sociais públicas;

D) destinação privilegiada de recursos públicos nas áreas relacionadas com a proteção à infância e à juventude.

Disponível em: http://www.planalto.gov.br/ccivil_03/leis/L8069.htm Acesso em 20/04/2017.  

Em relação às normas ortográficas da língua portuguesa em vigor, é CORRETO afirmar:

Alternativas
Comentários
  • a) (ERRADO) Mantiveram-se apenas os acentos diferenciais das palavras por/pôr e pode/pôde.

    b) (ERRADO) Todas as proparoxítonas são acentuadas. As palavras destacadas acentuam-se por serem paroxítonas terminadas em ditongo crescente.

    c) Gabarito.

    d) (ERRADO) Não se usa o hífen quando o prefixo termina em vogal e o segundo elemento começa com vogal diferente: aeroespacial.

    e) (ERRADO) Acentuam-se as oxítonas terminadas em éi(s), éu(s) ou ói(s): herói, anéis, céu. Nota: Não se acentuam os ditongos ei e oi das paroxítonas: heroico, proteico, apoio.

  • Regra geral do hífen:

    Só quando a segunda palavra começa com H.

    Ou quando há vogais repetidas.

     

  • a) Nesta atual reforma ortográfica foi abolido o acento circunflexo utilizado no substantivo pelo, deixando de haver acento diferencial em palavras paroxítonas homógrafas de outras não acentuadas.  Errada

    b) As palavras em questão possuem a mesma regra de acentuação: são paroxítonas, mas a alternativa esqueceu de trazer um detalhe: também são terminadas em ditongo. Errada

    c)Sobre o uso do hífen:

    Vogais iguais ficam separadas por hifen, vogais diferentes ficam juntas como em: autoeducação, extraoficial, coeditor e contraexemplo.Correta.

    d)Aeroespacial de acordo com a regra que coloquei acima fica junta sem hífen. 

    Bem-te-vi: Palavras botânicas ou espécies zoológicas são separadas por hífen também em: erva-doce, feijão-verde. 

    Ave-maria: por ser palavra composta, é separada com hífen. Errada

    e) Paroxítonas terminadas em "a" e "o" perderam o acento: ideia, jiboia, feiura

    O  acento gráfico permaneceu nas oxítonas e monossílabos tônicos com ditongos abertos tônicos “éi”, “ói” e “éu”, seguidos ou não de “s”, como em herói. Já nos ditongos "ei", "oi"  não acentuam como em: heroico  Errada

     

     

     

  • Gabarito: LETRA C

    Não se emprega o hífen:

    1. nas palavras em que o prefixo ou falso prefixo termina com vogal e o segundo elemento começa com letra diferente: autoeducação, extraoficial, contaexemplo

    2. nas palavras formadas pleos prefixos co- e re-, mesmo que diante de palavras iniciadas , respectivamente, por o e e: coeditor

  • DUAS MUDANÇAS FORAM:

    1-ATENÇÃO EM ACENTUAÇÃO SÓ MUDOU OS HIATOS PALAVRAS REPETIDAS TERMINADA COM EE E OO NÃO LEVAM ASSENTOS MAIS, EXEMPLOS: VOO, ENJOO,SOBREVOO,VEEM,LEEM,CREEM

    2-DITONGOS EI, OI QUANDO APARECER NA SILABAS TÕNICAS EM PAROXÍTONAS NÃO TEM ACENTOS,EXEMPLOS:
    IDEIA,PLATEIA,ESTREIA,ASSEMBLEIA,HEROICO,JIBOIA,PARANOICO
     

  • Passível de anulação ao meu entender...

     

    Pois a letra C está generalizando com a informação "Em relação à eliminação do emprego do hífen,..." 

     

    O hífen não foi eliminado, ele apenas não se usa mais em algumas palavras... Por isso considero errada essa afirmação também...

    Se estivesse escrito assim: "Em relação à eliminação do emprego do hífen nas palavras..." estaria correto..

  • VIDE:    https://www.youtube.com/watch?v=hgF5_RC6H8M

     

    DECIO TERROR    26min

     

                           MACETE: CHAMA A PALAVRA EM VOZ ALTA !!!

     

    De acordo com a tonicidade, as palavras são classificadas como:

    Oxítonas: são aquelas cuja sílaba tônica recai sobre a ÚLTIMA SÍLABA.

    Ex.: café – coração – cajá – atum – caju – papel

     

    Paroxítonas: são aquelas em que a sílaba tônica evidencia-se na PENÚLTIMA SÍLABA.

     

    Ex.: útil – tórax – táxi – leque – retrato – passível

     

    Proparoxítonas: são aquelas em que a sílaba tônica evidencia-se na ANTEPENÚLTIMA sílaba.

    Ex.: lâmpada –  câmara –  tímpano – médico – ônibus

    Acentuação gráfica

    Regras fundamentais:

    Proparoxítonas: todas são acentuadas. Ex.: analítico, hipérbole, jurídico, cólica.

    Q406972

     

    lá. (antepenúltima)      gri .  (penúltima)         mas (última)        –       

    .    (antepenúltima)       vi.   (penúltima)       das (última)     –                 

       as.tro..   (antepenúltima)      mi. (penúltima)    cas  (última)

     

    de.mo.crá.ti.ca – é.ti.ca – ú.ni.co = todas são proparoxítonas

     

     

     

     

     

    ·         Palavras oxítonas: acentuam-se todas as oxítonas terminadas em "a", "e", "o", "em", seguidas ou não do plural(s). Ex.: Pará – café(s) – cipó(s) – armazém(s)

     

    Essa regra também é aplicada aos seguintes casos:

    → Monossílabos tônicos terminados em "a", "e", "o", seguidos ou não de “s”.

    Ex.: pá – pé – dó –  há

    → Formas verbais terminadas em "a", "e", "o" tônicos seguidas de lo, la, los, las.

    Ex.: respeitá-lo – percebê-lo – compô-lo.
     

    Paroxítonas:  Acentuam-se as palavras paroxítonas terminadas em:

                                                      ROUXINOL

    Acentuam-se as paroxítonas TERMINADAS em:   i, is   L, N, R, OS, X, US,

     

    i, is

    Ex.: táxi – lápis – júri

    us, um, uns

    Ex.: vírus – álbuns – fórum

    l, n, r, x, ps

    Ex.: automóvel – elétron - cadáver – tórax – fórceps

    ã, ãs, ão, ãos 

    Ex.: ímã – ímãs – órfão – órgãos



    Ditongo oral, crescente ou decrescente, seguido ou não de s.

    Ex.: água – pônei – mágoa – jóquei

    Regras especiais:

      Os ditongos de pronúncia aberta "ei", "oi", que antes eram acentuados, perderam o acento com o Novo Acordo.

  • https://www.youtube.com/watch?v=nnNcXb8Lneg

    vídeo curtinho para quem ainda tem dúvidas sobre o novo acordo ortográfico 

  • Boa tarde,

     

    Discordo plenamente do Fabrício, entretando respeito o seu ponto de vista. Para mim a questão está corretisíma, aí cabe ao candidato, também, uma boa interpretação de texto, a banca se referiu à eliminação do hífem nas palavras citadas, letra C está perfeita... agregando um pouco mais sobre o hífem no novo acordo:

     

    De acordo com as novas regras prefixo terminado em vogal seguido de vogal identica terá hífem, lembre da regra que os iguais se repelem

    Micro-organismos.

    Atenção: exceto os prefixos PRE PRO CO RE exemplo: Preenchimento, coocupar (ainda que te tenham vogais identicas ficarão juntos)

     

    Os opostos se atraem, prefixo terminado em vogal, seguido de vogal diferente se juntam, exemplo anti + aéreo = antiaéreo

     

    ***Para agregar (parte 1), prefixos seguidos da letra R ou S dobra-se a letra que segue, exemplo: Auto + retrato = Autorretrato

     

    ***Para agregar (parte 2), prefixos que sempre terão hífem, lembre-se do mnemônico: A EX é VICE "AQUÉM" diga que ela é RECÉM chegada do ALÉM

     

    Bons estudos

  • FEIURA:

    Com a nova ortografia, o “i” e o “u” tônicos precedidos de ditongo, em palavras paroxítonas, perderam o acento.

    Portanto, não há mais acento em palavras como baiuca, cauila, maoismo, maoista, Sauipe, taoismo e feiura.

  • Acredito que o professor se equivocou aos 38 segundos no vídeo, ou é impressão minha?

    http://acordo-ortografico.blogspot.com.br/2013/12/forma-ou-forma.html

    então não foi somente por e pôr?

    fôrma e forma também, não é??????

    pôde e pode também, não é??????

    MACETE PROF MARCELO BERNARDO=> TEM VEM PODE PÔR FÔRMA 

     

    http://www.migalhas.com.br/Gramatigalhas/10,MI127894,31047-Pode+ou+Pode

  • outra coisa, essa prova é da banca PUC-PR

    http://www.pucpr.br/arquivosUpload/5381884891498589103.pdf

    questão 9.

  • A palavra herói continua a ser acentuada. Gabarito C.

  • Comentários da Alternativa a:

     

    Em geral, são eliminados os acentos gráficos que serviam para desambiguar palavras homógrafas:

    antes  pêlo e pelo, mas agora apenas pelo;

    antes  pára e para, mas agora apenas para.

    Mantêm-se, no entanto, as seguintes exceções, continuando a ser distinguidas por meio de acento gráfico as formas:

    por (preposição) e pôr (verbo);

    as formas pode (presente do indicativo) e pôde (pretérito perfeito) do verbo poder;

  • Pegadinha:

    "As palavras ideia, jiboia, heroi e feiura tiveram o acento agudo eliminado após o Novo Acordo Ortográfico"

    - A palavra Herói continua a ser acentuadas: As oxítonas terminadas em ditongo aberto, continua. Somente as paroxítonas terminadas em Ditongo aberto foram excluídas.

  • A regra geral do Hífen dá-se somente em duas situações: segundo elemento iniciado por H e quando as letras do fim do prefixo e no início da palavra forem iguais. 

  • Grá... Concursanda,

     

    segundo o novo acordo ortográfico, o acento diferencial na palavra "forma" é facultativo.

    nas palavras pode e por, é obrigatório.

  • Eu caí nessa pegadinha Rafael Araujo :/

    faltou estudar mais as regras de hífen para matar essa questão

  •   

    Regras para o uso do hifen:

    os iguais de repelem:  antI-Inflamatorio,  micrO-Oninus

    os opostos de atrem: semideus, extraoficial

    H pede hifen:  super-higienico, anti-hemorrágico

  • OS IGUAIS SE SEPARAM

    OS DIFERENTES SE ÚNEM

  • Não mais se acentuarãoi” e “u” tônicos dos hiatos quando, em palavras paroxítonas, forem precedidos de ditongos.


    Exemplos: feiura, feiume, reiuno, Sauipe, Bocaiuva, Guaira,  baiuca, boiuno.

  • Laiane, Guaíra e Guaíba  continuarão com seu tradicional acento. Há um pequeno equívoco ao interpretar a regra que retira o acento que colocávamos em bocaiuva e de baiuca. Não a culpo, pois o Acordo usa o conceito de ditongo de forma muito imprecisa; é necessário ler com muita atenção, para perceber que quando fala de ditongo, está se referindo exclusivamente aos ditongos descrescentes — aqueles que apresentam a semivogal depois da vogal (ai, éi, ei, oi, ói, ui; au, éu, eu, iu, oi).

  • O herói, desta vez, foi o vilão da história!

  • Lembrando que "heroico" é sem acento.

  • Letra c.

    Erro das demais:

    a) o acento foi retirado

    b) são paroxítonas terminadas em ditongo, e nem toda paroxítona é acentuada.

    d) “aeroespacial” não possui hífen.

    e) “herói” possui acento.

     

     

    Questão comentada pelo Prof. Elias Santana 

  • HERÓI===HEROICO

  • a) Segundo o Novo Acordo Ortográfico da língua portuguesa, o acento diferencial de palavras homógrafas como pelo (verbo pelar) e pêlo (substantivo) foi mantido. FALSO

    =======================================

    • Falso, pois o acento diferencial de palavras homógrafas foi abolido;

    • De acordo com o Novo Acordo Ortográfico, somente houve a manutenção do acento diferencial nos seguintes casos:

    pôr (verbo): para distinguir de por (preposição);

    pôde (terceira pessoa do sing. do pretérito perfeito do indic) para diferenciar de pode (terc. pessoa do sing. do presente do indicativo;

    verbos ter, vir e derivados na 3ª pessoa do plural do presente do indicativo (eles têm, eles vêm, eles convêm) para distinguir da 3ª pessoa do singular do presente do indicativo (ele tem, ele vem , ele detém, ele convém);

    • O acento diferencial foi retirado, mas antigamente era utilizado para diferenciar palavras homógrafas (palavras que apresentam a mesma grafia, embora possuam pronúncia e significados diferentes):

    Exemplos:

    apoio (suporte) e apoio (verbo apoiar);

    boto (golfinho) e boto (verbo botar);

    colher (talhes) e colher (apanhar);

    gosto (sabor) e gosto (verbo gostar);

    governo (ministério) e governo (verbo governar);

    dentre outros;

    =======================================

    Fonte: Felipe Luccas, Estratégia; Rosenthal, TEC;

  • b) A acentuação gráfica das palavras deficiênciacomunitáriainfância e precedênciajustifica-se pela mesma regra do Novo Acordo Ortográfico: todas as palavras paroxítonas são acentuadas. FALSO

    ===========

    • Falso, pois a regra em questão é: acentuam-se as paroxítonas terminadas em ditongo;

    • A acentuação gráfica das palavras:

    - comunitária;

    - deficiência;

    - infância;

    - precedência;

    realmente justificam-se pela mesma regra de acentuação e, que, no caso, todas são acentuadas por serem paroxítonas terminadas em ditongos crescentes: ea(s), oa(s), eo(s), ua(s), ia(s), ue(s), ie(s), uo(s), io(s);

    • Note que essas palavras informadas no enunciado (deficiência, comunitária, infância, precedência) estão inseridas na regra dos paroxítonos; são vocábulos paroxítonos terminados em ditongo crescente;

    • O problema na alternativa é que, redigida da forma que está, assevera que toas as palavras paroxítonas seriam acentuadas, o que não é verdade; inclusive a palavra "palavra" é paroxítona, mas não é acentuada!

    ===========

    Fonte: Rosenthal, TEC; Comentários Diversos, TEC; Felipe Luccas, Estratégia;

  • c) Em relação à eliminação do emprego do hífen, as palavras a seguir respeitam o Novo Acordo Ortográfico: autoeducaçãoextraoficialcoeditor e contraexemplo. VERDADEIRO

    ==========================

    • Realmente, as palavras:

    autoeducação;

    extraoficial;

    coeditor;

    contraexemplo;

    estão em acordo com o Novo Acordo Ortográfico pois possuem união de vogais diferentes;

    Co- : não leva hífen mesmo com vogal igual: coobrigado;

    • As palavras que aparecem com os prefixos:

    auto

    extra

    contra

    somente serão redigidas com hífen se o segundo elemento for iniciado por "h" ou pela mesma letra que finalizar o prefixo; exemplos:

    auto-homenagem;

    contra-argumento;

    • Não é o que ocorre com os vocábulos:

    autoeducação (depois de auto não veio nenhum "h" nem "o");

    extraoficial (não veio nem "h" nem "a");

    contraexemplo (depois de auto, não veio nem "h" nem "a");

    ==========================

    Fonte(s): Comentários, TEC; Rosenthal, TEC; Felipe Luccas, Estratégia;

  • d) O Novo Acordo manteve o hífen nas palavras compostas por justaposição cujos elementos constituem uma unidade semântica, mas mantêm uma tonicidade própria, como em: aero-espacialbem-te-viave-maria. FALSO

    ===========================================

    • A grafia correta é:

    Aeroespacial (vogais diferentes);

    Bem-te-vi (espécie zoológica);

    Ave-maria (palavra composta);

    • Nota que o vocábulo:

    aero-espacial

    foi redigido incorretamente, pois não deveria sê-lo com hífen; trata-se de mais um caso de prefixo concluído na letra "o";

    Portanto, somente se usaria hífen caso na sequência houvesse a letra "o" ou letra "h"; além de que a palavra "aeroespacial" não é uma palavra composta, já que há apenas um radical;

    Trata-se de uma derivação e não de uma composição;

    Bem-te-vi : é redigido com hífen, visto que trata de um vocábulo composto designando espécie zoológica;

    • Já o emprego de hífen em : "Ave-maria" : se justifica por ser uma locução substantivada;

    ===========================================

    Fonte: Felipe Luccas, Estratégia; Rosenthal, TEC;

  • GABARITO: LETRA C

    Quanto a questão, gostaria de deixar uma contribuição, vi que muitas pessoas se prenderam na LETRA D, em apenas falar que ela estava incorreta devido ao emprego de hífen em aeroespacial, que, de fato, está incorreto.

    Porém, o pessoal que não está tão familiarizado com o português, pode acabar não extraindo o conceito importante que a alternativa trouxe, imaginando ele estar incorreto também, o que não é verdade.

    A assertiva trouxe um conceito correto, que é bom termos em mente, principalmente quando se trata de bancas como FCC e FGV: de fato, com o novo acordo ortográfico, foi mantido o hífen nas palavras compostas por justaposição cujos elementos, de natureza nominal, adjetival, numeral ou verbal, constituem uma unidade sintagmática e semântica e mantêm o acento próprio.


ID
2478640
Banca
PUC-PR
Órgão
TJ-PR
Ano
2017
Provas
Disciplina
Português
Assuntos

                  LEI Nº 8.069, DE 13 DE JULHO DE 1990.

                          Das Disposições Preliminares

[...]

Art. 3º A criança e o adolescente gozam de todos os direitos fundamentais inerentes à pessoa humana, sem prejuízo da proteção integral de que trata esta Lei, assegurando-se-lhes, por lei ou por outros meios, todas as oportunidades e facilidades, a fim de lhes facultar o desenvolvimento físico, mental, moral, espiritual e social, em condições de liberdade e de dignidade.

Parágrafo único. Os direitos enunciados nesta Lei aplicam-se a todas as crianças e adolescentes, sem discriminação de nascimento, situação familiar, idade, sexo, raça, etnia ou cor, religião ou crença, deficiência, condição pessoal de desenvolvimento e aprendizagem, condição econômica, ambiente social, região e local de moradia ou outra condição que diferencie as pessoas, as famílias ou a comunidade em que vivem. (incluído pela Lei nº 13.257, de 2016).

Art. 4º É dever da família, da comunidade, da sociedade em geral e do poder público assegurar, com absoluta prioridade, a efetivação dos direitos referentes à vida, à saúde, à alimentação, à educação, ao esporte, ao lazer, à profissionalização, à cultura, à dignidade, ao respeito, à liberdade e à convivência familiar e comunitária.

Parágrafo único. A garantia de prioridade compreende:

A) primazia de receber proteção e socorro em quaisquer circunstâncias;

B) precedência de atendimento nos serviços públicos ou de relevância pública;

C) preferência na formulação e na execução das políticas sociais públicas;

D) destinação privilegiada de recursos públicos nas áreas relacionadas com a proteção à infância e à juventude.

Disponível em: http://www.planalto.gov.br/ccivil_03/leis/L8069.htm Acesso em 20/04/2017.  

Em relação à estrutura da frase, aos seus constituintes e aos períodos complexos do texto anterior, é INCORRETO afirmar:

Alternativas
Comentários
  • O verbo gozar, no sentido de "possuir", exige preposição: "gozam de todos os direitos".

    O verbo possuir é transitivo direto: não admite preposição: "possuem todos os direitos".

     

    Portanto, a substituição altera a regência verbal.

    Gabarito: a.

  • Alternativa A : Verbo GOZAR = verbo transitivo indireto( NECESSITA DE PREPOSIÇÃO)/ verbo POSSUIR = verbo transitivo direto ( INDEPENDE DE PREPOSOIÇÃ para ALCANÇAR SEU SENTIDO )

    Alternativa B : Quem se assegura se assegura DE algo , exige preposição para alcançar seu sentido, neste caso é um VERBO TRANSITIVO INDIRETO, logo seu complemento será OBJETO INDIRETO, neste caso utiliza-se o PRONOME OBLÍQUO (LHE) que serve para substituir referencia a PESSOA. 

    Alternativa C : OU é uma conjunção altenativa, bem com ORA .

    Alternativa D : E é uma conjunção ADTIVA bem como NEM.

     

  • Gabarito: LETRA A

     

    O verbo possuir é transitivo direto, ou seja, não necessida de preposição para ser ligado ao seu complemento. Assim, substituindo o verbo gozar pelo verbo possuir mudaríamos, sim, a regência verbal, pois ele passaria de transitivo indireto (verbo gozar) para transitivo direto (verbo possuir).

  • Na alternativa B, se estivesse escrito "assegurando lhes", estaria errado? Qual a função desse SE? Alguém por gentileza, pode tirar me está dúvida ?
  • Flávio, acredito que seja partícula apassivadora "todas as oportunidades e facilidades são asseguradas à criança e ao adolescente".


ID
2478643
Banca
PUC-PR
Órgão
TJ-PR
Ano
2017
Provas
Disciplina
Português
Assuntos

   A família e suas atribuições: uma árvore em constante transformação  

  O Cajueiro é uma planta originária das regiões norte e nordeste do Brasil. Por ser uma planta tropical suporta adversidades e não tem dificuldades em crescer e dar frutos em meio ao clima seco. Na natureza existem dois tipos: o comum, também conhecido como gigante, e o anão, sendo conhecidos por diversos nomes originados da língua tupi como: acaju, acajaíba, acajuíba, caju-comum, cajueiro-comum, dentre outros. As raízes, enterradas no solo, ligam-se à parte aérea pelo caule, que dá sustentação às folhas, consideradas o laboratório químico das plantas verdes. As flores são o órgão reprodutivo e transformam-se em frutos (as castanhas). Estes, quando maduros, caem na terra e, cultivados ou não, eclodem originando novos cajueiros. Sua fisiologia, formada por raízes, caule, folhas, flores e frutos, permite-nos fazer uma comparação com a família.

    Neste sentido, semelhante ao cajueiro, as raízes profundas da família, interligadas ao caule, se justapõem às suas antigas origens que, renovadas ou atualizadas, configuram as diversas formas de organização familiar contemporânea, representadas junto à complexa diversidade de sujeitos e à pluralidade política de suas ações.

GIRÃO, F.N.R.; NOGUEIRA, J.L.R.; TARGINO, K.K.F. A família e suas atribuições: uma árvore em constante transformação. Mossoró: Universidade Estadual do Rio Grande do Norte, 2012.

Disponível em: http://www.uern.br/controledepaginas/edicoes-uern-ebooks/arquivos/1205servico_social_e_crianca_e_adolescente.pdf. Acesso em: 22/04/2017.  

Em relação à interpretação do texto lido, é CORRETO afirmar:

Alternativas
Comentários
  • Quando a opção da letra A diz: "No Texto, ..." e a letra B diz: "Conforme o texto,..." já podemos descartar essas opções pois elas estão pedindo COMPREENSÃO e NÃO Interpretação. O enunciado da questão pede INTERPRETAÇÃO. 

    Logo, fui por eliminação e a alternativa correta é a letra E, bastou-me ler o Título do texto. 

    Força!! 

  • GABARITO: letra E

     

    Complementando:

     

    " Segundo a professora Rafaela Motta¹, quando o comando da questão trabalha a área de compreensão, aquela informação está no texto. Diante disso, você terá alguns enunciados básicos de questões de compreensão. Porém, se a informação estiver além do texto, fora do texto, trata-se de uma questão de interpretação. "

     

    Compreensão de texto – consiste em analisar o que realmente está escrito, ou seja, coletar dados do texto. Os comandos de compreensão (está no texto) são:

    Segundo o texto...

    O autor/narrador do texto diz que...

    O texto informa que...

    No texto...

    Tendo em vista o texto...

    De acordo com o texto...

    O autor sugere ainda...

    O autor afirma que...

    Na opinião do autor do texto...

     

     

    Interpretação de texto – consiste em saber o que se infere (conclui) do que está escrito. Os comandos de Interpretação (está fora (além) do texto) são:

    Depreende-se/infere-se/conclui-se do texto que...

    O texto permite deduzir que...

    É possível subentender-se a partir do texto que...

    Qual a intenção do autor quando afirma que...

    O texto possibilita o entendimento de que...

    Com o apoio do texto, infere-se que...

    O texto encaminha o leitor para...

    Pretende o texto mostrar que o leitor...

    O texto possibilita deduzir-se que...

     

     

    FONTE: http://www.gramaticaparaconcursos.com/2014/03/diferenca-entre-compreensao-de-texto-e.html

  • Errada a Letra E. O texto não tevela uma metáfora, mas sim uma compração( símile). Ver as descrições em Fernado Pestana. Resposta correta, letra C.

     

  • Cistiano, acho que você está equivocado.

  • Nesta questão, apenas complementando o que aqui já foi esclarecido, tem-se o aposto. De acordo com site especializado em língua portuguesa, dentre outras disciplinas: "Aposto é um termo que se junta a outro de valor substantivo ou pronominal para explicá-lo ou especificá-lo melhor. Vem separado dos demais termos da oração por vírgula, dois-pontos ou travessão.". O título do texto apresenta aposto, o que pode ter dificultado na resolução da questão. Veja-se: "A família e suas atribuições: uma árvore em constante transformação". Ou seja, "a família e suas atribuições são uma árvore em constante transformação.". Portanto, veifica-se a ocorrência da metáfora. "A metáfora consiste em utilizar uma palavra ou uma expressão em lugar de outra, sem que haja uma relação real, mas em virtude da circunstância de que o nosso espírito as associa e depreende entre elas certas semelhanças. É importante notar que a metáfora tem um caráter subjetivo momentâneo; se a metáfora se cristalizar, deixará de ser metáfora e passará a ser catacrese (é o que ocorre, por exemplo, com "pé de alface", "perna da mesa", "braço da cadeira"). Obs.: toda metáfora é uma espécie de comparação implícita, em que o elemento comparativo não aparece." [http://www.soportugues.com.br/secoes/estil/estil2.php]

     

    Bons estudos!

     

  • Ótima questão, muito bom o texto.

  • Confundi com comparação, considerei a letra E errada.

  • Cristiano alves você está equivocado, pois não existe o elemento comparativo na frase para dizer que se trata de uma comparação. Metáfora é uma comparação ideológica, não contendo o elemento comparativo. 

    Ex: ele é uma fera no PC.  (Metáfora )

    Ex: ele é ágio no PC COMO uma fera é. (Comparação )

  • Em relação à interpretação do texto lido, é CORRETO afirmar:  

    a) ERRADA. No texto, o cajueiro foi selecionado para representar as famílias das regiões norte e nordeste do Brasil, visto que elas precisam enfrentar as adversidades do clima seco e, mesmo assim, elas não têm dificuldades em crescer e dar frutos.  

    O texto não fala nada em cajueiro representar famílias e nem traz informação que evidenciam essa informação. Veja o trecho: O Cajueiro é uma planta originária das regiões norte e nordeste do Brasil. Por ser uma planta tropical suporta adversidades e não tem dificuldades em crescer e dar frutos em meio ao clima seco.

     

    b) ERRADA. Ao compararem a estrutura da família à da árvore do cajueiro, que tem raízes, caule, folhas, flores e frutos, os autores do texto fazem referência ao modelo padrão de família, constituído por pai, mãe, filhos e seus descendentes. 

    O texto não faz menção ao padrão de família. Sua fisiologia, formada por raízes, caule, folhas, flores e frutos, permite-nos fazer uma comparação com a família.

     

    c) ERRADA. Conforme o texto, “acaju”, “acajaíba” e “acajuíba” são nomes atribuídos ao cajueiro anão; e “gigante”, “caju-comum” e “cajueiro-comum” são nomes atribuídos ao tipo comum dessa árvore. 

    Observe o texto: Na natureza existem dois tipos: o comum, também conhecido como gigante, e o anão, sendo conhecidos por diversos nomes originados da língua tupi como: acaju, acajaíba, acajuíba, caju-comum, cajueiro-comum, dentre outros. 

    Existe, portanto, o cajueiro comum (gigante) e o cajueiro anão, ambos são conhecidos por diversos nomes. Se assim não fosse a palavra "conhecidos", deveria estar no singular para concordar com o sujeito anão.

     

    d) ERRADAO caule liga as raízes às folhas, sustentando estas, e por isso ele é considerado o laboratório químico das plantas verdes.

    De acordo com o texto: As raízes, enterradas no solo, ligam-se à parte aérea pelo caule, que dá sustentação às folhas, consideradas o laboratório químico das plantas verdes.

    Observando a concordância entre o sujeito e o verbo considerado (particípio/passado), no feminino, são as FOLHAS o laboratório das plantas verdes.

     

    e) CERTA. O título revela uma metáfora e tem a função de anunciar uma informação a ser tratada no texto: a família, como uma árvore, tem suas atribuições e está em constante transformação. 

    Segue texto: Sua fisiologia, formada por raízes, caule, folhas, flores e frutos, permite-nos fazer uma comparação com a família.

        Neste sentido, semelhante ao cajueiro, as raízes profundas da família, interligadas ao caule, se justapõem às suas antigas origens que, renovadas ou atualizadas, configuram as diversas formas de organização familiar contemporânea, representadas junto à complexa diversidade de sujeitos e à pluralidade política de suas ações.

     


ID
2478646
Banca
PUC-PR
Órgão
TJ-PR
Ano
2017
Provas
Disciplina
Português
Assuntos

   A família e suas atribuições: uma árvore em constante transformação  

  O Cajueiro é uma planta originária das regiões norte e nordeste do Brasil. Por ser uma planta tropical suporta adversidades e não tem dificuldades em crescer e dar frutos em meio ao clima seco. Na natureza existem dois tipos: o comum, também conhecido como gigante, e o anão, sendo conhecidos por diversos nomes originados da língua tupi como: acaju, acajaíba, acajuíba, caju-comum, cajueiro-comum, dentre outros. As raízes, enterradas no solo, ligam-se à parte aérea pelo caule, que dá sustentação às folhas, consideradas o laboratório químico das plantas verdes. As flores são o órgão reprodutivo e transformam-se em frutos (as castanhas). Estes, quando maduros, caem na terra e, cultivados ou não, eclodem originando novos cajueiros. Sua fisiologia, formada por raízes, caule, folhas, flores e frutos, permite-nos fazer uma comparação com a família.

    Neste sentido, semelhante ao cajueiro, as raízes profundas da família, interligadas ao caule, se justapõem às suas antigas origens que, renovadas ou atualizadas, configuram as diversas formas de organização familiar contemporânea, representadas junto à complexa diversidade de sujeitos e à pluralidade política de suas ações.

GIRÃO, F.N.R.; NOGUEIRA, J.L.R.; TARGINO, K.K.F. A família e suas atribuições: uma árvore em constante transformação. Mossoró: Universidade Estadual do Rio Grande do Norte, 2012.

Disponível em: http://www.uern.br/controledepaginas/edicoes-uern-ebooks/arquivos/1205servico_social_e_crianca_e_adolescente.pdf. Acesso em: 22/04/2017.  

Em relação aos recursos linguísticos empregados no texto, é CORRETO afirmar:

Alternativas
Comentários
  • a) (ERRADO) Que é pronome relativo de "caule".

    b) (ERRADO) Por ser é conjunção causal (equivale a "porque", "já que", "visto que"), e embora é conjunção concessiva (equivale a "mesmo que", "ainda que");

    c) (CERTO) O verbo haver, no sentido de "existir", é impessoal. Não admite flexão no plural.

    d) (ERRADO) Sua retoma Cajueiro.

    e) (ERRADO) Suas refere-se a "famílias".

  • Rapaz, que questão. Exige muita atenção do candidato. "Cajueiro" e não "planta". Quase caio.

  • "Há" no sentido de existir não flexiona.

    A grande pegadinha está na letra "d", pois "sua" refere-se a Cajueiro.

  •  BAGARITO C

    No trecho “Na natureza existem dois tipos”, o termo sublinhado pode ser substituído pela forma verbal “há”, sem alteração do sentido original da frase, que mantém a concordância verbal adequada à língua portuguesa padrão. 


ID
2478649
Banca
PUC-PR
Órgão
TJ-PR
Ano
2017
Provas
Disciplina
Português
Assuntos

Norte-americanos solteiros acabam de eleger os tropeços de gramática como uma das principais razões que arruínam um primeiro encontro ou o flerte com um potencial parceiro. Uma pesquisa realizada com 5.481 adultos com mais de 21 anos foi conduzida pela empresa de marketing on-line MarketTools para definir as qualidades, atitudes e expectativas que ilustram as mudanças culturais nas relações de aproximação entre pessoas solteiras. Encomendada pelo site Match.com, com sede em Dallas, a sondagem buscou definir um quadro dos 100 milhões de norte-americanos solteiros, 1/3 da população, segundo o Censo 2012, dos EUA. O levantamento, divulgado pouco antes do carnaval de 2013, mostrou que, ao julgar um parceiro em potencial, tanto homens como mulheres colocam a gramática no topo da lista de “requisitos obrigatórios” de um relacionamento, com 55% de preferência, atrás apenas do estado geral dos dentes da pessoa (58%).

                                                                     Língua Portuguesa. Ano 8. Nº 89. 

Assinale a opção CORRETA sobre a estrutura de alguns dos períodos presentes no texto.  

Alternativas
Comentários
  • (...) definir as qualidades, atitudes e expectativas que ilustram (...)

     

    Vou explicar de maneira bem resumida. O "que" é pronome relativo. Como sei isso? Basta substituir por "o qual", "a qual" ou suas variantes. Se se mantiver o sentido, ok. Como sabemos, o pronome relativo retoma um termo anterior, nesse caso, o termo "As qualidades" "Atitudes" e "expectativas", tanto é que o verbo logo após ("ilustram") encontra-se no plural exatamente para concordar.

     

    Por fim, visto isso, o pronome relativo atua como sujeito. Para ver isso, basta a substituir o pronome relativo pelo seu referente.

  • Alguém me explica o erro na B, por favor! 

  • Adauto Filho, sou novata nos estudos da língua portuguesa, mas pelo estudo das aulas, eu ACHO que o erro é na função do "que".
    Na oração, é de fato um pronome relativo e inicia uma oração adjetiva (restritiva, diga-se). No entanto, penso que ele não exerce função de objeto direto, mas sim de COMPLEMENTO NOMINAL do substantivo "razões". Como eu disse, não tenho certeza, rs. Se eu estiver errada, me corrijam por favor! 

  • Adauto Filho: o que na primeira ocorrência exerce papel de sujeito veja:

     

    Norte-americanos solteiros acabam de eleger/ oração subordinada substântiva e está faltando o objeto direto nela/  os tropeços/ objeto direto/ de gramática como uma das principais razões/ complemento nominal/ que arruínam; o "que" retoma a locução substântiva "principais razões " na sua oração direta ficaria: arruínaram principais razões, principais razões não está funcionando como objeto direto, mas sim como sujeito do verbo, ou seja, o que se arruínaram? principais razões,. avisem- me se houver algum equívoco.

  • na letra'' B'':se estivesse uma virgula antes do ''que''ela estaria correta????pois, nesse caso ela seria adjetiva explicativa!

    se alguém puder ajudar me agradeço....

  •  b)

    O vocábulo “que”, em sua primeira ocorrência destacada no texto, é um pronome relativo em função de objeto direto e inicia uma oração adjetiva que insere uma informação acessória sobre seu referente, “razões”. ERRADO. O que realmente é um pronome relativo, mas não possui na oração que integra função de objeto direto e sim de sujeito. Vejamos o texto: Norte-americanos solteiros acabam de eleger os tropeços de gramática como uma das principais razões que arruínam um primeiro encontro ou o flerte com um potencial parceiro. -O pronome relativo que retoma principais razões e é sujeito do verbo arruínam . (um primeiro encontro ou flete é o ojeto direto da oração)-Eurismar Penha se houvesse uma vígula antes do pronome relativo , a oração ainda assim continuaria correta , apesas o valor semântico sofreria alterações. A oração deixaria de ser restritiva e se tornaria explicativa.(sempre que há a inclusão de vírgulas antes do pronome relativo o sentido da oração é modificado e na maioria dos casos a alteração mantém a correção gramatical)

  • Gabarito: D
    Letra B
    está errada pois o QUE possui função de sujeito, observe:
    [...]razões que arruínam um primeiro encontro[...]
    O QUE retoma razões. Colocando na ordem direta -->> razões arruínam um primeiro encontro.
                                                                                      suj.       VTD                      OD

     

  • O erro da letra B está em dizer que o que exerce função sintática de objeto direito, quando na realidade esse que exerce função sintática de complemento nominal, pois está em função de razões que é um (Substantivo abstrato)

     

    Bons estudos

  • Pq a E está errada? Pra mim ambos são apostos explicativos, não?

  • Vamos ao que segue...

     

    A - A forma verbal “definir” inicia uma oração subordinada substantiva reduzida que funciona como complemento verbal indireto de “buscou”, na oração anterior, motivo pelo qual poderia ser iniciada por preposição.

    ERRADO - "definir" forma com "buscou" uma locução verbal. Não é complemento..

     

    b -  O vocábulo “que”, em sua primeira ocorrência destacada no texto, é um pronome relativo em função de objeto direto e inicia uma oração adjetiva que insere uma informação acessória sobre seu referente, “razões”.

    ERRADO - O "que" tem função de SUJEITO... 

     

     C -  A oração “ao julgar um parceiro em potencial” é reduzida de infinitivo e veicula valor concessivo, motivo pelo qual poderia ser substituída pela sua versão desenvolvida com o conectivo “quando”. 

    ERRADO - "quando" não tem valor concessivo... Ele esboça valor temporal...

     

    D - O vocábulo “que”, em sua segunda ocorrência destacada no texto, funciona como sujeito da forma verbal “ilustram” e retoma o termo “as qualidades, atitudes e expectativas”, objeto direto da oração anterior.  

    CORRETO- 

     

    E - As orações reduzidas “realizada com 5.481 adultos com mais de 21 anos” e “divulgado pouco antes do carnaval de 2013” têm a mesma classificação, e o emprego de vírgulas ao redor delas é opcional. 

    ERRADA -  creio que o erro está em dizer que as vírgulas são opcionais. São obrigatóriaspara separar um aposto explicativo

     

    Espero ter ajudado..

     

    Abraço

  • Essa prova de português foi muito bem elaborada! Realmente testa os candidatos! Parabéns para a Banca!

  • E - As orações reduzidas “realizada com 5.481 adultos com mais de 21 anos” e “divulgado pouco antes do carnaval de 2013” têm a mesma classificação, e o emprego de vírgulas ao redor delas é opcional. 

    ERRADA -

    A primeira é um aposto especificativo e as vírgulas são proibitivas (não é qualquer pesquisa, pois é aquela realizada..., restringe as pessoas);

    A segunda é um aposto exemplificativo e as vírgulas são opcionais (não restringe nada, apenas explica a forma de divulgação).

  • (...) para definir as qualidades, atitudes e expectativas / que ilustram as mudanças culturais nas relações (...)

     

    É exatamente o que a alternativa D disse.

    O "que" tem valor de sujeito da oração "que ilustram as mudanças culturais nas relações" (o que é que ilustram as mudanças?), pois funciona como pronome relativo, retomando "as qualidades, atitudes e expectativas".

  • Essa não é pra qualquer um....

  • Parabéns para quem acertou essa questão.

     

  • Li a letra A, n entendi nada, mas pensei "para mim isso é locução verbal... hmmm... deixe-me ler as demais";

    Li a letra B, entendi nada, mas pensei comigo "isso não me parece lá muito acessório não... deixe-me ler as demais";

    Li a letra C, entendi nada, mas pensei comigo "n tenho ideia onde essa letra quer que eu encaixe esse 'quando'";

    Li a letra D, tive certeza que tudo estava completamente correto. Voltei ao texto para verificar se a questão pedia a correta ou incorreta (hoje, verifico esse tipo de coisa umas 3 vezes antes de marcar). Pedia a correta e eu não via onde a D poderia estar errada... marquei essa.

  • Na segunda ocorrência, o que é o sujeito do verbo ilustrar, retoma o termo anterior(objeto direto do verbo definir, período subordinado adverbial final). D.


ID
2478652
Banca
PUC-PR
Órgão
TJ-PR
Ano
2017
Provas
Disciplina
Português
Assuntos

Leia o texto a seguir que é parte de um artigo técnico.

A construção e manutenção de um ambiente de trabalho seguro e saudável não é uma tarefa fácil. Ela requer disciplina, dedicação convencimento e capacitação técnica. Os riscos associados ao exercício profissional têm uma ampla variedade de agentes, com atuação permanente no ambiente de trabalho. Antecipar, reconhecer, avaliar e controlar esses riscos é obrigação legal do empregador e compromisso dos profissionais especializados em segurança e saúde. Entretanto, as ações dessas duas partes só alcançam resultados efetivos com a participação ativa dos trabalhadores, pois são eles que atuam diretamente nos sistemas.

MATTOS, Ricardo Pereira. Campos eletromagnéticos. Disponível em: <http://www.ricardomattos.com/artigo.htm> . Acesso em: 15 abril 2017.


De acordo com o texto, “esses riscos”, “dessas duas partes” e “eles” correspondem, respectivamente

Alternativas
Comentários
  • GABARITO: letra E

     

    Os riscos associados ao exercício profissional têm uma ampla variedade de agentes, com atuação permanente no ambiente de trabalho. Antecipar, reconhecer, avaliar e controlar esses riscos é obrigação legal do empregador e compromisso dos profissionais especializados em segurança e saúde. Entretanto, as ações dessas duas partes só alcançam resultados efetivos com a participação ativa dos trabalhadores, pois são eles que atuam diretamente nos sistemas.

  • Típica questão de Coestão referencial. 

    Gabarito: E


ID
2478655
Banca
PUC-PR
Órgão
TJ-PR
Ano
2017
Provas
Disciplina
Português
Assuntos

Leia a seguir o excerto de um artigo científico sobre o trabalho infantil para responder à questão.

Como em muitos países há um número expressivo de crianças e adolescentes que trabalham e estudam, torna-se primordial que se analise não só se o trabalho é responsável pela baixa frequência das crianças na escola, mas também se o trabalho infantil reduz o desempenho escolar. Bezerra, Kassouf e Kuenning (2007) utilizaram os dados do Sistema Nacional de Avaliação da Educação Básica (SAEB) de 2013, que possui informações de testes padrões de língua portuguesa e de matemática aplicados aos alunos do 5ª ano e 9ª ano do ensino fundamental e do 3ª ano do ensino médio, em escolas públicas e privadas do Brasil.

Texto adaptado de: KASSOUF, Ana Lúcia. O que conhecemos sobre o trabalho infantil? Nova Economia: Belo Horizonte V. 17, n. 2, p. 323-350, maio/agosto de 2007. Disponível em: < http://www.scielo.br/pdf/neco/v17n2/v17n2a05.pdf>. Acesso em: 20 abril 2017.


Assinale a alternativa que dá continuidade de modo coerente ao texto.

Alternativas
Comentários
  • A continuidade deve conter algum dado estitístico, visto que os autores avaliaram estudantes de determinadas séries, do ensino fundamental e médio. Assim a assertiva "a" é a unica que versa sobre um resultado obtido.

  • No primeiro período se destacam duas hipóteses  (1) "o trabalho é responsável pela baixa frequência das crianças na escola" (2) "o trabalho infantil reduz o desempenho escolar."

    A assertiva "a" é a única que retoma as hipóteses anteriormente citadas.

  • LETRA A "MENOS ERRADA" 

  • Tenho medo da voz do Arenildo!

     


ID
2478658
Banca
PUC-PR
Órgão
TJ-PR
Ano
2017
Provas
Disciplina
Raciocínio Lógico
Assuntos

Arno, especialista em lógica, perguntou: qual a negação de “hoje é carnaval se, e somente se, for 8 ou 9 de fevereiro”?

A resposta CORRETA é:

Alternativas
Comentários
  • Hoje é Carnaval, se, e somente se, for 8 ou 9 de fevereiro (proposição com o operador lógico bicondicional)

     

    Encontrar a negação de uma proposição significa identificar quando a proposição tem valor lógico FALSO.

     

    Uma proposição com o operador lógico bicondicional tem valor lógico falso quando as proposições tem valores lógicos diferentes, ou seja:

    - A primeira é verdadeira E a segunda é falsa

                                         OU 

    - A primeira é falsa E a segunda é verdadeira

     

    Assim, a negação da proposição "Hoje é Carnaval, se, e somente se, for 8 ou 9 de fevereiro" é:

    - Hoje é Carnaval E não é 8 nem 9 de fevereiro 

                                         OU

    - Hoje não é Carnaval E é 8 ou 9 de fevereiro

  • GAB. C

     

    Trascrevendo para a linguagem simbólica temos:

     

    P ↔ (Q v R)

     

    A negação do "se, e somente se" é

     

    ↔ B = A ^ ~B  v  ~A ^ B 

     

    Portanto, no caso em tela soluciona-se do seguinte modo:

     

    P ^ ~(Q v R)  v ~P ^ (Q v R)

     

    Analisando cada membro:

     

    Hoje é carnaval E NÃO é dia 8 nem dia 9 de fevereiro

                               OU

    Hoje NÃO é carnaval E é dia 8 ou 9 de fevereiro

  • Nega-se o Bicondicional trocando por disjunção exclusiva ou negando uma da proposições.

    Na alternativa C ele demonstrou negando a primeira proposição ou a segunda.

  • Boa noite,

     

    Decore o seguinte, na negação do "se...somente se" (bi-condicional) teremos sempre a disjunção exclusiva (ou...ou) onde duas premissas de mesmo valor retornam um resultado FALSO, logo, para que seja verdadeira apenas uma premissa poderá ser verdadeira isso justifica a presença do OU (disjunção exclusiva)

     

    Bons estudos

  • C: hoje é carnaval 

    F: 8 ou 9 de fevereiro. 

    C <-> F é equivalente a (C -> F) ^ (F -> C).

    Negação: ~(C -> F) v ~(F -> C) = (C ^ ~F) v (F ^ ~ C)

    Logo, gabarito letra c

  • Q felicidade em e entender e acerta a Questão. Mt feliz. Porém se uma dessas caísse na my prova, provavelmente na hora do nervosismo, não conseguiria fazer. 

  • Quem estiver com dificuldades sugiro que faça o curso com o professor Renato aqui no próprio sítio. As explicações dele são simples e objetivas.

  • Apenas com o conhecimento das negações já seria possível resolver essa questão, negação da Bicondicional só tem duas ou Troca pela Disjunção exclusiva( OU OU ) ou Nega o Consequente(¬) a única alternativa que apresenta uma dessas opções é a C , dessa forma resolveria rápido e sobraria tempo para questões com maior grau de dificuldade .

    Bons estudos . 

  • A bicondicional só tem valor verdadeiro quando as duas proposições forem verdadeiras ou as duas proposições forem falsas. Basta uma delas ser verdadeira ou falsa para que a bicondicional seja falsa.

    Portanto basta negar ou uma ou outra proposição para que a bicondiconal seja falsa.

    Algum erro me avisem!

  • (P e ~Q) ou (~P e Q)

  • vídeo da resolução da questão no link:

    https://youtu.be/MNbRuoujMgY

  • SE E SOMENTE SE

    <----->

    VV=V

    FF=V

  • Excelente comentário da professora!! Vale a pena assistir.

  • Se, somente se = Basta  negar as 2 frases e trocar o conectivo por ou...ou .

     

    Simples assim.

  • Pessoal, como que voces identificam com tanta clareza que "for 8 ou 9 de fevereiro" é uma só proposição? Pra mim é óbvio que existem 3 proposições:

     

     

    P: Hoje é carnaval.

    Q: Hoje é 8 de Fevereiro.

    R: Hoje é 9 de Fevereiro.

     

     

    A expressão que eu montei ficou assim: P <-> (Q v R) Resumindo a pergunta: alguém me explica por que aquele "ou" ali nao está dividindo duas proposições?

     

     

     

    Valeu!

  • GABARITO C

     

    A negação do conectivo "se, e somente se" é feito pelo uso do conectivo "ou excludente", --> "ou... ou".  (1 ... ou outro.')

    Voce pode verificar isso pelas tabelas-verdade dos dois: (p <-> q)    e   (ou p ou q.)

    p....q....p <-> q..... ou p ou q
    V....V........V.............F
    V....F........F.............V
    F....V........F.............V
    F....F........V.............F

    Note que a quarta coluna é a negação da terceira. 
     

  • Hoje é carnaval se e somente se for 8 ou 9 de fevereiro

                              <------------------> Bicondicional

    1° vou arrumar a frase  para depois negar

    Se hoje é carnaval, então é 8 ou 9 de fevereiro E Se hoje é 8 ou 9 de fevereiro, então é carnaval ( a frase principal e esta são equivalentes)

    2° vou negar a frase

    Hoje é carnaval e não é 8 ou 9 de fevereiro Ou Hoje é 8 ou 9 de feveiro e não é carnaval.

     

    A banca foi sacana pois ela deu a resposta começando pela segunda frase e ainda invertei a posição da frase.

     

     

  • Only God

  • P: (A= Hoje é Carnaval )  ( B=( Hoje = 8  ou Hoje = 9 ) )

    ~P=   A v B     ( A ^ ~B )  v ( ~A ^ B )

    Portanto nossa resposta DEVE TER  2 termos em disjunção. onde um deles é simples  A  ||  ~A 

     a) Hoje não é carnaval se, e somente se, não for 8 ou 9 de fevereiro.  [ Equivalência e não negação::   A B    ~A ~B ]

     b) Hoje não é carnaval e não é 8 nem 9 de fevereiro. [ o que está escrito é : ~A ^  ~B ] [ lembre que queremos 2 proposições em disjunção ]

     c)  Hoje não é carnaval e é 8 ou 9 de fevereiro OU hoje é carnaval e não é 8 nem 9 de fevereiro.  [ perfeito: o antecedente cobre ~A ^ B; enquanto o consequente cobre A ^ ~B ]

     d) Hoje é carnaval e é 8 de fevereiro. [ tá doido ? ] 

     e) O carnaval não é no mês de fevereiro.  [ brincou ? ]

    Portanto o gabarito é C; a professora fala que a conjunção e a disjunção são operações recíprocas, quando o termo correto entendo ser  que são COMUTATIVAS.

     

     

  • A <---> B = (A --> B)  ^  (B ---> A)

    ~ [ (A --> B)  ^ ( B --> A)]

    ~ (A-->B)  v   ~ (B---> A)


    Regra da amante: mantém a 1ª E nega a 2ª


    A  ^ ~B  v   B  ^ ~A 

    GAB: C

  • Letra "C" , sem decorebas, leis e macetes. Usando somente a tabela verdade considere Verdadeiro 1 e Falso Zero. O enunciado da questão apresenta C = Carnaval e D= Dias, Temos ~(C <->D), um detetor de desiguadades, isso corresponde a ~(1-0-0-1) = (0-1-1-0). Agora considere que P v Q, onde P=~C e D, Q= C e ~D), teremos:

    ~C e D = P              C e ~D = Q               P v Q = Letra "C"  (0-1-1-0), temos conformidade

    1  e  0 = 0                0 e 1 = 0                   0 ^ 0 =  0

    1      1 = 1                0    0 = 0                   1    0 =  1

    0      0 = 0                1    1 = 1                   0    1 =  1

    0      1=  0                1    0 = 0                   0    0 =  0 

  • Negação da bicondicional: sempre a maior frase possivel com conectivo OU

  • Se, somente se = Basta  negar as 2 frases e trocar o conectivo por ou...ou .. E a frase é do tamanho de um trem! 

  • Letra: C

     

    Hoje não é carnaval e é 8 ou 9 de fevereiro ou hoje é carnaval e não é 8 nem 9 de fevereiro.  

  • Chutei e acertei, mas é muito raro de encontrar questões envolvendo bicondicional.

  • Ele pede a negação da BICONDICIONAL (P <--> Q).

    Logo a negação será: (~P ^ Q) v (P ^ ~Q).

    Só é lembrar que na tabela verdade a BICONDICIONAL é negada da seguinte forma. VF v FV.


ID
2478661
Banca
PUC-PR
Órgão
TJ-PR
Ano
2017
Provas
Disciplina
Raciocínio Lógico
Assuntos

Leia o problema a seguir.

Wesley foi apresentado pelo seu amigo Cripa a três amigos: João, Pedro e Arnaldo, que são casados com Marli, Cintia e Paula, mas não foi informado quem é casado com quem. Cripa disse ainda que um é engenheiro, outro é advogado e outro é professor, mas também não informou quem é que faz o quê.

Como Wesley é muito inteligente, Cripa deu algumas pistas para que Wesley descobrisse quem era casado com quem e qual a profissão de cada um. As pistas foram:

O professor é casado com Cintia.

Arnaldo é advogado.

Paula não é casada com Arnaldo.

Pedro não é professor.

De acordo com as pistas, pode-se concluir que

Alternativas
Comentários
  • João : professor casado com Cíntia;

    Pedro: Engenheiro casado com Paula;

    Arnaldo: Advogado casado com Marli.

    Bons estudos!!! Letra E)

     

  • Pista 1: O professor é casado com Cíntia

    Pista 2: Arnaldo é advogado

    A partir dessas afirmações, podemos concluir que:

    CONCLUSÃO 1: Arnaldo não é casado com Cíntia (já que ela é casada com o professor e Arnaldo é advogado).

     

    Pista 3: Paula não é casada com Arnaldo

    Sabemos, de acordo com a CONCLUSÃO 1, que Arnaldo não é casado com Cíntia. 

    Se ele não é casado com Cíntia e nem com Paula, concluímos, por exclusão, que ele é casado com Marli. Portanto:

    CONCLUSÃO 2: Arnaldo é casado com Marli (pois não é casado com Cíntia e nem com Paula)

     

    Pista 3: Paula não é casada com Arnaldo

    Pista 2: Arnaldo é advogado

    A partir dessas afirmações, podemos concluir que:

    CONCLUSÃO 3: Paula não é casada com o advogado (pois não é casada com Arnaldo).

     

    Pista 1: O professor é casado com Cíntia

    Sabemos, de acordo com a CONCLUSÃO 3, que Paula não é casada com o advogado. Por outro lado, Paula também não é casada com o professor, já que, conforme a Pista 1, ele é casado com Cíntia.

    Concluímos, por exclusão, que Paula é casada com o engenheiro. Portanto:

    CONCLUSÃO 4: Paula é casada com o engenheiro (pois não é casada com o professor nem com o advogado).

     

    Pista 4: Pedro não é professor

    Pista 2: Arnaldo é advogado

     Se Pedro não é professor e nem advogado (já que está é a profissão de Arnaldo), concluímos, por exclusão, que ele é engenheiro. Portanto:

    CONCLUSÃO 5: Pedro é engenheiro (já que não é professor e nem advogado)

     

    Pista 2: Arnaldo é advogado

    Sabemos, de acordo com a CONCLUSÃO 5, que Pedro é engenheiro. Por outro lado, conforme a Pista 2, Arnaldo é advogado. Assim, João não desenvolve nenhuma dessas atividades.

    Concluimos, por exclusão, que João é professor, já que ele não é engenheiro e nem advogado. Portanto:

    CONCLUSÃO 6: João é professor (já que não é engenheiro e nem advogado)

     

    Pista 1: O professor é casado com Cíntia

    Sabemos, de acordo com a CONCLUSÃO 6, que João é professor. Logo, ele é casado com Cíntia. Portanto:

    CONCLUSÃO 7: João é casado com Cíntia (já que ele é professor e este profissional é casado com Cíntia)

     

    CONCLUSÃO 2: Arnaldo é casado com Marli

    Sabemos, de acordo com a CONCLUSÃO 7, que João é casado com Cíntia. Por outro lado, conforme a CONCLUSÃO 2, Arnaldo é casado com Marli.

    Concluímos, por exclusão, que Pedro é casado com Paula. Portanto:

    CONCLUSÃO 8: Pedro é casado com Paula (já que João e Cíntia são casados e Arnaldo e Marli também)

     

    Em resumo, temos que

    1) João é professor e casado com Cíntia (CONCLUSÃO 6 + CONCLUSÃO 7)

    2) Pedro é engenheiro e casado com Paula (CONCLUSÃO 5 + CONCLUSÃO 8)

    3) Arnaldo é advogado e casado com Marli (Pista 2 + CONCLUSÃO 2)

  •                                  João                  Pedro                      Arnaldo

    Profissão                 professor            engenheiro             advogado

    Casado                     Cíntia                 Paula                    Marli

     

     

    Gab: E

     

  • Resposta E

    ---------------------------------------------

             JOA  PED  ARN

    CIN  OOO XXX XXX

    PAU XXX OOO XXX

    MAR XXX XXX OOO

    ENG XXX OOO XXX

    ADV  XXX XXX OOO

    PRF OOO XXX XXX

    ---------------------------------------------

    O professor é casado com Cintia. (3)

    Arnaldo é advogado. (1)

    Paula não é casada com Arnaldo. (4)

    Pedro não é professor. (2)

     

    Obrigado Revista Coquetel - Logica

  • GABARITO LETRA "E"

    DEPOIS QUE COMECEI A UTILIZAR O MÉTODA DA TABELA NUNCA MAIS ERREI UMA QUESTÃO DESSA. MUITO LEGAL MESMO!

  • Divino Souza, me ensine essa tabela que vc citou!! Eu não entendi nada rsrs

  • Não sei se existe um método "formal" de resolver isso, mas eu apenas preenchi uma tabela no caderno e resolvi.

    https://imgur.com/a/caT2d

    As linhas pretas são as informações providas pela questão e as linhas azuis são as que você pode deduzir a partir disso.

  • João, Pedro
    Marli, e Paula,
    engenheiro, 

    advogado = Arnaldo,
    professor casada Cintia 
    marli casada Arnaldo

    O professor é casado com Cintia.
    Arnaldo é advogado.
    Paula não é casada com Arnaldo.
    Pedro não é professor.
    De acordo com as pistas, pode-se concluir que 
     

  • QUESTÃO CLASSICA DE ASSOCIAÇÃO LÓGICA.

    LETRA -

  • Como eu fiz:

    O professor é casado com Cintia. 

    Arnaldo é advogado. (não é caso com cintia)

    Paula não é casada com Arnaldo.(logo paulo é casado com marli)

    Pedro não é professor (logo o prof é o joão que é casado com cintia)

    Sendo assim pedro é eng casado com paula.

  • Vem jogando na tabela de trás pra frente que fica muitíssimo fácil.

  • Me perde um pouco mais consegui acerta. E bom ter um rascunho pra n se perder kk

  • Questões desse tipo podem ser resolvidas com a organizaçao da tabela no modelo de um passa tempo coquetel.

                             Marli               Cintia            Paula              /                    Eng         Adv       Prof

    Jõao                    N                    S                     N                                       N             N          S

    Pedro                  N                    N                       S                                     S            N            N

    Arnaldo               S                      N                     N                                      N            S           N

     

    PROF                 N                    N                       S

    ENG                    N                    S                       N

    ADV                     S                    N                     N

     

  • GABARITO E

    Faça uma tabelinha e faz sem pressa pra não botar um nome no lugar errado e errar a questão.

    Não tente fazer decorando nome - profissão etc....pois é assim que o povo erra.

  • questão que ninguem erra, mas faz perder muito tempo, porisso pode prejudicar nas outras. 

  • João com Cintia

    Pedro com Paula

    Arnaldo com Marli

  • O professor é casado com Cintia.
    Arnaldo é advogado.
    Paula não é casada com Arnaldo.
    Pedro não é professor.

    Pelas CERTEZAS, Arnaldo É ADVOGADO ,  Pedro NÃO é PROF logo Pedro é ENGENHEIRO , assim João é PROF
    Já temos as profissões ARNALDO ^ADVPEDRO ^ENG  e JOÃO ^ PROF 
    e  Novas CERTEZAS,  Cintia é casada com o PROF logo  CINTIA ^  JOÃO
    Paula NÃO é casada com  Arnaldo logo Paula é casada com PEDRO,  PAULA ^ PEDRO
    Restando a  ARNALDO ^ MARLI.

  • e-

     

    trab.. - homem - mulher

    O professor é casado com Cintia. Pr - ??? - cin

    Arnaldo é advogado. adv - arn - ???

    Paula não é casada com Arnaldo. adv - mar - arn (por eliminação acima).

    Pedro não é professor. - eng - Pau - Pedro (as outras 2 profissoes ja foram definidas, logo, o que resta sera do Pedro)

  • Gabaríto e

    Marli é casada com o advogado e João é professor que é casado com Cintia.

  • Gab E galera.

    Fiz por Associação lógica,ou seja, montei um simples quadro com as informações e fui marcando X ou O. DÁ MUITO CERTO.

    Ficou : Arnaldo é adv e casado com Marli

    Pedro é Eng e casado com Paula

    João é professor e casado com Cíntia.

    Força!

  • Se Pedro não é professor e arnaldo é Advogado, Pedro só pode ser Engenheiro.

    Se Pedro é Engenheiro e Arnaldo é advogado, João só pode ser Professor.

    Se Cintia é casada com o professor, Cintia é casada com João.

    Se Paula não é casada com arnaldo e nem pode ser com João, Paula é casada com Pedro.

    Se Arnaldo não é casado com cintia e nem com Paula, só pode ser casado com Marli.

    Conclusão: Pedro é Engenheiro e Casado com Paula, Arnaldo é Advogado e casado com Marli, João é Professor e casado com cintia. 

    Gab. E

     

     

  • Associação lógica. Adoro essas questões. Gab. E

  • Um assunto prazeroso, dentro dos limites de um cérebro de humanas, no RLM...

  • Dá pra fazer de cabeça, aprendam a usar abreviaturas tais como: João é Professor = JP


ID
2478664
Banca
PUC-PR
Órgão
TJ-PR
Ano
2017
Provas
Disciplina
Raciocínio Lógico
Assuntos

Considere a seguinte situação:

Tati e Elisandro são dois amigos que trabalham em um departamento de ouvidoria. Elisandro passou três dias contando o número de reclamações que entravam por dia. Tati perguntou quantas reclamações entraram nos três últimos dias e Elisandro respondeu que “o produto do número de reclamações de cada um dos três dias é igual a 72 e a sua soma é igual ao número de anos que a empresa completou ontem”. Tati pensou um pouco e disse: “Tenho uma dúvida e preciso de mais uma dica”. Então ele respondeu que “no dia de maior número de reclamações, ele comprou uma garrafa de vinho no valor de R$ 359,00”.

Com as informações anteriores, é possível concluir que o

Alternativas
Comentários
  • Eu viajei legal nessa questão, porém acertei.

    Fatorei 72 = 2x2x2x3x3 onde 2³ e 3² --> 2³ = 8 e 3² = 9

    8 + 9 = 17 e 3+5+9 = 17

    Pelas alternativas, concluí que os números eram: 8, 3 e 3.

  • Tem que ser medium pra responder esta questão!

    Mas tenho que me render ao comentário da Stéphanie Tokunaga. Parabéns! muito bom o seu raciocínio... Tem palpite pra Mega Sena? rsrsrs...

     

  • Primeira coisa deve-se iguinorar a parte que diz:

     e a sua soma é igual ao número de anos que a empresa completou ontem”. Tati pensou um pouco e disse: “Tenho uma dúvida e preciso de mais uma dica”. Então ele respondeu que “no dia de maior número de reclamações, ele comprou uma garrafa de vinho no valor de R$ 359,00”.

    Segundo teste hipóteses para x.y.z = 72, onde X, Y e Z são números naturais, de acordo com as alternativas

     1 x 8 x 9 = 72 Por essa hipótese letra B e D estariam corretas, devem ser descatadas

    1 x 1 x 72 = 72 Por essa hipótese letra B e E estariam corretas, devem ser descatadas

    A letra C não tem solução natural, 3 números que multiplicado deem 72 e somados deem 23.

    3 x 3 x 8 = 72 é uam solução possível, gabarito letra A

  • Pois é, a informação que ele comprou um vinho de 359 e os anos que a empresa completou ontem só serviram para confundir o candidato.

  • Através da fatoração, poderíamos ter:

    9,4,2/6,6,2/6,4,3/8,3,3/

    também poderíamos ter 72,1,1

    Para mim a questão tem todas essas respostas, ou a resposta tem mais de uma alternativa certa ou não tem resposta. Aguardar o gabarito definitivo...

     

  • Recebi a resolução de um professor e não me convenci. Peço a todos que a indiquem para Comentário do Professor do QC .

  • olha pessoal, acertei, fiz uma exclusão aqui, (c) e (d) não eram, se a (b) estava certa a (e) tambem estaria, e vi uma progressão geometrica no valor 359,00.

    de 3 para 5 (2), de 5 para 9 (4). entao 2 vezes 4=8, conclui que seria o maior dia de reclamacoes, mas para mim questao sem pé nem cabeça.

  • Bom gente, ao tirar as informações do exercício temos os seguintes dados:

    x.y.z=72 (produto das ligações)

    x+y+z= ? (a soma é o dia que a empresa fez aniversario, mas não sabemos quando foi)

    o primeiro passo é fatorar o nº, para saber todos os números que multiplicados dão 72:

    72 / 2

    36 / 2

    18 / 2

    9 / 3

    3/ 3

    1

    Agora nós montamos as combinações possíveis e somamos, pois o exercicio diz que a soma deles é o aniversário da empresa: 

    4 . 2 . 9 = 4+2+9 =15

    2 . 2 . 18 = 2+2+18= 22

    4 . 6 . 3 = 4+6+3= 13

    8 . 3 . 3 = 8+3+3= 14

    6 . 6 . 2 = 6+6+2=14

    Nós não sabemos quantos anos a empresa fez, mas tati trabalhava lá e sabia, e mesmo assim disse que essas informações não eram suficientes...

    Então, as únicas opções possiveis de fatoração acima são: 8, 3, 3 e 6, 6, 2, pois o produto de ambos da 72 e a soma da o mesmo número (14), por isso tati não conseguiu resolver..

    Depois disso, Elisandro deu mais uma informação para que tati descobrisse, e essa informação foi: no dia que a empresa teve MAIS LIGAÇÕES, eu comprei uma garrafa de vinho", ou seja, teve um dia que teve mais ligações. Sendo assim, ficamos com as opções 8, 3 e 3, pois a outra opção o 6 (maior número) se repete 2 vezes... 

    Ou seja, gabarito A: "maior número de reclamações por dia foi 8. "

  • Não é possivel afirmar com precisão nenhuma das respostas.

     

    a) maior número de reclamações por dia foi 8. 

    Não necessariamente. E se tivessemos as seguintes situações:

    1º dia (72), 2º dia (1), 3º dia (1) ou 1º dia (9), 2º dia (8), 3º dia (1).

    Nas duas situações o produto é 72 e há reclamações em numeros maiores que 8.

     

    b) menor número de reclamações por dia foi 1.

    Poderíamos ter: 8*3*3, sendo 8 o maior numero de reclamações por dia.

     

    c) total de reclamações dos três dias foi 23.  

    8*3*3 = 72; 8+3+3 = 14

    72*1*1 = 72; 72+1+1 = 74

    O total pode ser maior ou menor que 23.

     

    d) total de reclamações dos três dias foi 18. 

    Idem alternativa "c".

     

    e) maior número de reclamações por dia foi 72.  

    Poderíamos ter 8*3*3 = 72, sendo 8 o maior número de reclamações.

     

    http://rlmparaconcursos.blogspot.com.br/

     

     

     

  • Não há nenhuma relação lógica entre a compra do vinho de R$ 359,00 e o resto do problema. Atribuir uma coincidência entre a fatoração e o soma dos números que compõem esse valor é apelar para a adivinhação, o que não pode condizer com a disciplina denominada "raciocínio lógico". Entendo que A, B, D ou E podem ser consideradas respostas corretas e totalmente justificáveis para essa questão.

  • Não adianta reclamar, ficar criticando a banca e procurando erro na questão.

    Essa foi uma das melhores questões que já resolvi de Raciocínio Lógico, Excelente questão, bastante difícil e o raciocínio correto que deve ser feito é o descrito pela Stéphanie Tokunaga.

    Muito bom, parabéns!

  • "Não adianta reclamar", temos que aceitar as arbitrariedades da banca, mesmo estando flagrantemente errado? né!?

  • Resolvi a questão como a Stéphanie Tokunaga fez, porém há mais possibilidades:

    1 . 1 . 72 --> 1+1+72 =74

    1 . 2 . 36 --> 1+2+36 =39

    1 . 3 . 24 --> 1+3+24 =28

    1 . 4 . 18 --> 1+4+18 =23

    1 . 6 . 12 --> 1+6+12 =19

    1 . 8 . 9 --> 1+8+9 =18

    2 . 2 . 18 --> 2+2+18 =22

    2 . 3 . 12 --> 2+3+12 =17

    2 . 4 . 9 --> 2+4+9 =15

    2 . 6 . 6 --> 2+6+6 =14

    3 . 3 . 8 --> 3+3+8 =14

    3 . 4 . 6 --> 3+4+6 =13

  • Para resolver a questão só fatorei 72 para chegar aos números possíveis de encontrar o produto 72. Depois, eu fui analisando cada uma das alternativas para achar uma alternativa coerente:

    72 I 2

    36 I 2                           

    18 I 2

    9 I 3

    3 I 3

    1

    a) maior número de reclamações por dia foi 8.  2 x 2 x 2 = 8 (V)

    b) menor número de reclamações por dia foi 1. Pela fatoração, não é possível que o 1 seja resposta  na solução do problema.  (F)

    c) total de reclamações dos três dias foi 23.  O total de reclamações nos três dias é 72. (F)

    d) total de reclamações dos três dias foi 18. O total de reclamações nos três dias é 72 (F)

    e) maior número de reclamações por dia foi 72.  Não é possível, pois o comando da questão já nos dá a pista que houve um dia que teve mais reclamações que os outros dois. (F)

    Foi assim que consegui resolver.

     

    Fé em Deus e prossigamos para o alvo!

  • x=número de reclamações

    3x=72

    x+x+x=?

    3(3x)=72

    9x=72

    x=8

    Logo, o maior número de reclamações é 8.

     

     

     

    c439

  • Gente, não precisa saber o número de aniversário da empresa, mas entender porque essa informação não é suficiente...
    Essa informação não é suficiente pois, somente com ela teriamos duas resposta corretas, observem:

    1 . 1 . 72 - 1+1+72 =74

    1 . 2 . 36 - 1+2+36 =39

    1 . 3 . 24 - 1+3+24 =28

    1 . 4 . 18 -1+4+18 =23

    1 . 6 . 12 - 1+6+12 =19

    1 . 8 . 9 - 1+8+9 =18

    2 . 2 . 18 - 2+2+18 =22

    2 . 3 . 12 - 2+3+12 =17

    2 . 4 . 9 - 2+4+9 =15

    2 . 6 . 6 - 2+6+6 =14 (se repete)

    3 . 3 . 8- 3+3+8 =14 (se repete)

    3 . 4 . 6 - 3+4+6 =13

    Sendo assim ficamos reduzidas a duas opções. Mas sabemos que no dia de maior reclamação um vinho foi comprado, pouco importa o valor do vinho, mas que houve um, e apenas um dia que o número de reclamação foi maior do que os demais, e assim eliminamos a possibilidade 2,6,6 pois temos 2 6 e não um dia com mai reclamação.

    sobra apenas a opção 3,3,8

  • Qual o erro da alternativa B?

    x.y.z = 72

    x = 9

    y = 8

    z = 1

    9x8x1 = 72.

    Ou seja, é possível concluir que o menor número de reclamações/dia foi 1.

    Banca quer fazer questão da Nasa com examinador do brisolão.

  • Rei delas, o problema da 'B' é que se os números fosse os citados ela sabeira só com a informação do dia do aniversário, já que é essa é a única possibilidade em que a soma dos algarismos é 18

  • Fiz de forma simples e achei o mesmo resultado talvez por viajem ou sorte mas pensei assim levando em conta o que disse o enunciado

    o produto do número de reclamações de cada um dos três dias é igual a 72 e a sua soma é igual ao número de anos que a empresa completou ontem”.

    eu dividir 72/3 = 24

     daí eu dividir 24/3 e achei 08

     

  •  a)maior número de reclamações por dia foi 8. 

     b)menor número de reclamações por dia foi 1.

     c)total de reclamações dos três dias foi 23.  

     d)total de reclamações dos três dias foi 18. 

     e)maior número de reclamações por dia foi 72.  

    De início realmente fiquei muito put.. com a banca, porém depois percebi ser bem razoável.
    a) O maior número de reclamações é 8 .  [ A primeira vista descarta-se pois 72.1.1 já seria o suficiente para tal, porém isso é com base matemática, desprezando todas as demais informações, a princípio inúties.

    b) O menor número de reclamações foi 1. [ desprezada por 2.3.12 ]

    c) Total de reclamações foi 23. [ mesmo sem fazer a parte matemática, nada indica que 23 fosse a única resposta ]

    d) Total de reclamações foi 18. [ idem... ]

    e) Maior foi 72. [ o maior poderia ser 72 ]

    Dessa forma percebe-se que tem informações não consideradas importantes.

    1. A soma das reclamações totalizam a idade da empresa! Quanto é ? Quem sabe?  Elisandro, por afirmar !!!
    Tati ficou em dúvida até receber a última "pérola": No dia de maior reclamações, sucedeu algo! pronto baseado nos conhecimentos de Tati a dúvida foi sanada.
    2. Ora, obviamente Tati sabe a idade da empresa, qual seja somando as reclamações( os termos x,y,z ) deu a resposta.
    Portanto as dúividas não estavam nas parcelas, e sim na soma delas. 
    E qual foi a dúvida ? 2 possibilidades somavam esse número.
    2,6,6 ---> 14
    3,3,8 ---> 14
    e assim pode responder  pois o MAIOR é um número único !
    Francamente, é a questão para fim da prova, pelo trabalho que dá, porém não posso deixar de reconhecer que extrai bastante do raciocínio do concurseiro.

  • questão nível ninja hard

  • Considerando os números: 
      1 x 4 x 18 = 72
      1 + 4 + 18 = 23
    a letra C também não estaria correta?

       c) total de reclamações dos três dias foi 23

  • Elisandro respondeu que “o produto do número de reclamações de cada um dos três dias é igual a 72 e a sua soma é igual ao número de anos que a empresa completou ontem”. Tati pensou um pouco e disse: “Tenho uma dúvida e preciso de mais uma dica”. Então ele respondeu que “no dia de maior número de reclamações, ele comprou uma garrafa de vinho no valor de R$ 359,00”.

    Com as informações anteriores, é possível concluir que o  

     a)maior número de reclamações por dia foi 8. 

     b)menor número de reclamações por dia foi 1.

     c)total de reclamações dos três dias foi 23.  

     d)total de reclamações dos três dias foi 18. 

     e)maior número de reclamações por dia foi 72.  

    A RESPOSTA CORRETA E' A "B". A alternativa nos diz que o menor numero de reclamacoes por dia foi "1", e nao que o dia que teve o menor numero de reclamacoes foi de "1" (uma) reclamacao. Esta correto, pois se houvesse algum dia que tivesse ocorrido numero de reclamacoes menor que "1", esse numero seria "0" (zero), dessa forma o produto dos tres dias seria tambem "0" (zero).

  • Para esse calculo, existem mais de uma resposta:

    A) dia1 = 8 - dia2 =3 e dia3 = 3 -> 8*3*3 = 72 (maior número de reclamações por dia foi 8)

    B) dia1 = 1 - dia2=2 e dia3 =36 -> 1*2*36= 72 (menor número de reclamações por dia foi 1)

    E) dia1 =1 - dia2=1 e dia3=72 -> 1*1*72=72 (maior número de reclamações por dia foi 72)

  • Concordo com o comentário da Sheila, até pq fiz da seguinte forma 3x4x6=72 que também o produto informado na questão, mas como não sabemos o resultado da soma não podemos bater o martelo. A unica alternativa correta seria a B, pois caso houvesse um dia que não tivesse nenhuma reclamação esse produto teria de ser ZERO. Me Corrijam se deixei passar algo


ID
2478667
Banca
PUC-PR
Órgão
TJ-PR
Ano
2017
Provas
Disciplina
Raciocínio Lógico
Assuntos

Marcelo é um pequeno criador de gado da raça Nelore. Seu pequeno rebanho tem 50 cabeças e tem provisões (ração) para 20 dias, à razão de três refeições diárias. Rogério, um amigo de longa data, presenteou-o com mais 10 cabeças. Marcelo descobriu que o próximo carregamento com ração vai atrasar. Então tomou a seguinte providência: diminuiu em 1/3 as refeições diárias do seu rebanho. Por quantos dias durará a provisão?

Alternativas
Comentários
  • 50 cabeças de gado comem 3 x por dia = 150 refeições. 

    150 refeições em 20 dias (150 x 20) = 3.000 porções.

    Ele ganha mais 10 cabeças e vai botar o gado para comer 2 x por dia, ou seja, fica com 60 cabeças e 120 refeições.

    Como o número de porções não se altera, por quantos dias as 3.000 refeições que ele tem durarão, considerando que serão 120 refeições por dia?

    3.000/ 120 = 25 dias. Gabarito B)

  • Utilizando regra de três composta:

     

    Cabeças de gado                Dias              Refeições por dia

                50                                20                             3

                60                                 X                              2

     

    Identificando a natureza da relação entre as grandezas:

    - Cabeças de gado X Dias: se, com 50 cabeças de gado, a ração dura 20 dias, com 60 cabeças de gado, essa mesma quantidade de ração irá durar mais ou menos dias? Ela durará menos dias, pois existirão mais cabeças de gado para alimentar com a mesma ração. Assim, quanto mais cabeças de gado, menos dias a ração irá durar. Logo, essas grandezas são inversamente proporcionais.

    - Cabeças de gado X Refeições por dia: se, fazendo 3 refeições por dia, a ração dura 20 dias, fazendo 2 refeições por dia, essa mesma quantidade de ração irá durar mais ou menos dias? Ela durará mais dias, pois será consumida uma quantidade menor de ração em cada dia. Assim, quanto mais refeições por dia, menos dias a ração irá durar. Logo, essas grandezas são inversamente proporcionais.

     

    Portanto, vamos calcular o valor de X na regra de três, considerando que as grandezas são inversamente proporcionais:

     

    (20/x) = (60/50) . (2/3)         -> como as grandezas são inversamente proporcionais, devemos inverter os seus valores ao colocar na equação.

    X = (20.50.3)/(60.2)

    X = 25 dias

     

  • Gabarito: Letra B

     

    Comentários em:

    http://rlmparaconcursos.blogspot.com.br/2017/08/tj-pr-2017.html

     

    Bons estudos!

  • 50 cabeças - 60 refeições

    60 cabeças - x

    x=50 refeições.

    Como o fazendeiro irá reduzir o numero de refeições diarias em 1/3, serão duas refeições por dia, ou seja, o novo rebanho terá racao suficiente para 25 dias.

  • 50 -- 20 -- 3

    60 -- X --2

    ________________________________

    50.20.3 = 60.x.2   [x=25]

  • vídeo com a resolução da questão está no link:

    https://youtu.be/CP7YRfLdvZQ

  • Fiz a questão de forma simples:

    1) se 50 cabeças - para 20 dias (3 refeiçao por dia)

    com acrescimo de mais 10 cabeças eu teria 60 cabeças se fosse ter para as mesmas quantidades de refeiçoes, fiz assim primeiramente:

    50 --------20

    60 -------x

    achei 16 com sobra de 04 somente pra ter uma ideia - teria refeiçoes para mais ou menos 16 dias

    se eu tirar um terço das refeiçoes ou seja agora ficaria com duas refeiçoes por dia, concluo que das 20 dias tirando 1 por dia eu teria a mais 20 refeiçoes que daria pra mais 10 dias se contar que tenho duas refeiçoes por dia

    Então pensei se com as tres refeiçoes eu teria mais ou menos 16 dias como não é uma conta certa, pode ser 15 dias mais 10 dias do terço que reduzir eu terei mais 25 dias 

     


ID
2478670
Banca
PUC-PR
Órgão
TJ-PR
Ano
2017
Provas
Disciplina
Raciocínio Lógico
Assuntos

Ana, Maria e Joana foram ao shopping comprar presentes para seus namorados Jõao, Mário e Paulo. O namorado de Maria ganhou uma gravata, Mário ganhou um cinto e Paulo disse ao namorado de Ana que ganhou uma carteira. Com base nessas informações, é CORRETO afirmar que

Alternativas
Comentários
  • Questões assim, tem que fazer o esquema no papel:

    PAULO-JOANA-CARTEIRA

    MARIO-ANA-CINTO

    JOÃO-MARIA-GRAVATA

  • Gabarito: Letra D

     

    Comentários em:

    http://rlmparaconcursos.blogspot.com.br/2017/08/tj-pr-2017_17.html

     

    Bons estudos!

  • JOÃO,Paulo e Mario

    Mario ganhou o cinto
    Paulo ganhou a carteira
    Só restou o João para ganhar a gravata

  • vídeo com a resolução da questão no link:

    https://youtu.be/9jyHjlxazEI

  • Fica confuso pela ambiguidade gerada no trecho " Paulo disse ao namorado de Ana que ganhou uma carteira" . 

  • COM BASE NO TEXTO ENFERIMOS QUE:

    JOAO                     GRAVATA            MARIA

    MARIO                    CINTO                  MARIO NÃO NAMORA MARIA

    PAULO                   CARTEIRA             PAULO NÃO NAMORA ANA

    COM ESSA ENFERÊNCIAS CHEGAMOS AS SEGUINTES CONCLUSÕES:

    ANA                     MARIO -CINTO

    MARIA                JOAO - GRAVATA

    JOANA                PAULO - CARTEIRA


ID
2478673
Banca
PUC-PR
Órgão
TJ-PR
Ano
2017
Provas
Disciplina
Legislação dos Tribunais de Justiça (TJs)
Assuntos

Segundo Código de Organização e Divisão Judiciária do Estado do Paraná, são considerados órgãos do Poder Judiciário do Estado do Paraná

Alternativas
Comentários
  • Art. 2º. São órgãos do Poder Judiciário do Estado:

    I – o Tribunal de Justiça;

    II – REVOGADO;

    III – os Tribunais do Júri;

    IV – os Juízes de Direito;

    V – os Juízes de Direito Substitutos de entrância final;

    VI – os Juízes Substitutos;

    VII – os Juizados Especiais;

    VIII – os Juízes de Paz.

     

    Quanto aos Juizados Especiais, integram o Poder Judiciário do Paraná apenas os Juizados Especiais estaduais (Juizado Especial Cível e Juizado Especial Criminal).

  • Gabarito A

    CODJ/PR, Art. 2º. São órgãos do Poder Judiciário do Estado:

    I – o Tribunal de Justiça;

    II – REVOGADO;

    III – os Tribunais do Júri;

    IV – os Juízes de Direito;

    V – os Juízes de Direito Substitutos de entrância final;

    VI – os Juízes Substitutos;

    VII – os Juizados Especiais;

    VIII – os Juízes de Paz.

  • Art. 2º. São órgãos do Poder Judiciário do Estado:
    I- o Tribunal de Justiça;
    II - REVOGADO; (pela Lei nº 14.925 de 24/11/2005 – DOE nº 7109 de 25/11/2005)
    III - os Tribunais do Júri;
    IV - os Juízes de Direito;
    V - os Juízes de Direito Substitutos de entrância final;
    VI - os Juízes Substitutos;
    VII - os Juizados Especiais;
    VIII - os Juízes de Paz

    R:A
     

  • CODJ/PR, Art. 2º São órgãos do Poder Judiciário do Estado: I - o Tribunal de Justiça; II - (Revogado); III - os Tribunais do Júri; IV - os Juízes de Direito; V - os Juízes de Direito Substitutos de entrância final; VI - os Juízes Substitutos; VII - os Juizados Especiais; VIII - os Juízes de Paz.

  • A fim de encontrarmos a alternativa correta, iremos analisar cada uma das assertivas a seguir:

    A) Correta - os Tribunais do Júri e os Juízes de Paz. 

    Quanto aos órgãos do Poder Judiciário do Estado, o art. 2º da Lei 14.277/2003 dispõe da seguinte relação: 1) o Tribunal de Justiça; 2) os Tribunais do Júri; 3) os Juízes de Direito; 4) os Juízes de Direito Substitutos de entrância final; 5) os Juízes Substitutos; 6) os Juizados Especiais; e 7) os Juízes de Paz. Pronto, encontramos o nosso gabarito.

    B) Incorreta - as Defensorias Públicas e os Juízes de Direito.

    Quanto aos órgãos do Poder Judiciário do Estado, o art. 2º da Lei 14.277/2003 dispõe da seguinte relação: 1) o Tribunal de Justiça; 2) os Tribunais do Júri; 3) os Juízes de Direito; 4) os Juízes de Direito Substitutos de entrância final; 5) os Juízes Substitutos; 6) os Juizados Especiais; e 7) os Juízes de Paz. Veja que a Defensoria Pública não faz parte desta relação. O art. 134 da CF/88 estabelece que “A Defensoria Pública é instituição permanente, essencial à função jurisdicional do Estado, incumbindo-lhe, como expressão e instrumento do regime democrático, fundamentalmente, a orientação jurídica, a promoção dos direitos humanos e a defesa, em todos os graus, judicial e extrajudicial, dos direitos individuais e coletivos, de forma integral e gratuita, aos necessitados.

    C) Incorreta - os advogados públicos e privados, os Juízes e os Desembargadores.  

    Quanto aos órgãos do Poder Judiciário do Estado, o art. 2º da Lei 14.277/2003 dispõe da seguinte relação: 1) o Tribunal de Justiça; 2) os Tribunais do Júri; 3) os Juízes de Direito; 4) os Juízes de Direito Substitutos de entrância final; 5) os Juízes Substitutos; 6) os Juizados Especiais; e 7) os Juízes de Paz. Veja que os advogados públicos e privados e os Desembargadores não são órgãos do Poder Judiciário. O art. 131 da CF/88 assevera que a Advocacia Pública é uma instituição que representa o ente federativo, judicial e extrajudicialmente, enquanto o art. 133 CF/88 deixa claro que o advogado é indispensável à administração da justiça. Por fim, a norma não prevê que os Desembargadores sejam órgãos, mas, com certeza, atuam nos Tribunais do Judiciário, normalmente em grau recursal.

    D) Incorreta - os juízes federais, os juízes estaduais e os advogados.  

    Quanto aos órgãos do Poder Judiciário do Estado, o art. 2º da Lei 14.277/2003 dispõe da seguinte relação: 1) o Tribunal de Justiça; 2) os Tribunais do Júri; 3) os Juízes de Direito; 4) os Juízes de Direito Substitutos de entrância final; 5) os Juízes Substitutos; 6) os Juizados Especiais; e 7) os Juízes de Paz. Veja que a norma simplesmente menciona “Juízes de Direito", não havendo a necessidade de diferenciar os Juízes federais dos estaduais. Por fim, os advogados não estão incluídos nessa relação. O art. 131 da CF/88 assevera que a Advocacia Pública é uma instituição que representa o ente federativo, judicial e extrajudicialmente, enquanto o art. 133 CF/88 deixa claro que o advogado é indispensável à administração da justiça.

    E) Incorreta - os juizados especiais federais, os juizados especiais estaduais e o Tribunal de Justiça.


    Quanto aos órgãos do Poder Judiciário do Estado, o art. 2º da Lei 14.277/2003 dispõe da seguinte relação: 1) o Tribunal de Justiça; 2) os Tribunais do Júri; 3) os Juízes de Direito; 4) os Juízes de Direito Substitutos de entrância final; 5) os Juízes Substitutos; 6) os Juizados Especiais; e 7) os Juízes de Paz. Veja que a norma simplesmente menciona “os Juizados Especiais", não havendo a necessidade de diferenciar os Juizados Especiais federais dos Juizados Especiais estaduais. 

    Resposta: A



ID
2478676
Banca
PUC-PR
Órgão
TJ-PR
Ano
2017
Provas
Disciplina
Legislação dos Tribunais de Justiça (TJs)
Assuntos

Conforme preceitos descritos no Estatuto dos Servidores Públicos do Poder Judiciário do Estado do Paraná, é absolutamente vedado aos funcionários públicos

Alternativas
Comentários
  • Art. 157. Ao funcionário é proibido:

    XXII - tratar de assuntos particulares na repartição durante o horário de expediente;

  • Esta é a famosa questão chamada mamão com mel .

  • Não precisa nem estudar para acertar essa.

  • Art. 157. Ao funcionário é proibido:

                   XXII - tratar de assuntos particulares na repartição durante o horário de expediente;

  • Das Proibições
    Art. 157. Ao funcionário é proibido:
    I - ausentar-se do serviço durante o expediente, sem prévia autorização do chefe imediato;
    II - retirar qualquer documento ou objeto da repartição sem prévia anuência da autoridade competente;
    III - recusar fé a documentos públicos;
    IV - opor resistência injustificada ao encaminhamento de documento, ao andamento de processo ou à execução de serviço;
    V - promover manifestação de apreço ou desapreço no recinto da repartição;
    VI - cometer a pessoa estranha ao Quadro da repartição, fora dos casos previstos em lei, o desempenho de atribuição que seja
    de sua responsabilidade ou de seu subordinado;
    VII - coagir ou aliciar subordinados no sentido de se filiarem à associação profissional ou sindical, ou a partido político;
    VIII - manter sob sua chefia imediata, em cargo ou função de confiança, cônjuge, companheiro ou parente até o terceiro
    grau;
    IX - valer-se do cargo para lograr proveito pessoal ou para outrem, em detrimento da dignidade da função pública;
    X - participar de gerência ou administração de sociedade privada, personificada ou não personificada, salvo a participação
    em sociedade cooperativa constituída para prestar serviços a seus membros, e exercer o comércio, exceto na qualidade de
    acionista ou cotista;
    XI - atuar como procurador ou intermediário junto a repartições públicas;
    XII - receber propina, comissão, presente ou vantagem de qualquer para o desempenho de suas atribuições;
    XIII - aceitar comissão, emprego ou pensão de estado estrangeiro;
    XIV - praticar usura sob qualquer de suas formas;
    XV - proceder de forma desidiosa;
    XVI - utilizar pessoal ou recursos materiais do Poder Público em serviços ou atividades particulares;
    XVII - cometer a outro funcionário atribuições estranhas ao cargo que ocupa, exceto em situações de emergência e
    transitórias;
    XVIII - exercer quaisquer atividades que sejam incompatíveis com o exercício do cargo ou da função e com o horário de
    trabalho;
    XIX - recusar-se a atualizar seus dados cadastrais quando solicitado;
    XX - referir-se de modo depreciativo em qualquer escrito ou por palavras às autoridades constituídas e aos atos
    administrativos por ela praticados, ressalvada a análise técnica e doutrinária em trabalho de natureza acadêmica;
    XXI - deixar de comparecer ao serviço sem justificativa aceita pela administração;
    XXII - tratar de assuntos particulares na repartição durante o horário de expediente;
    XXIII - empregar materiais e bens do Poder Judiciário ou à disposição deste em serviço ou atividade estranha às funções
    públicas;
    XXIV - manter domicílio ou residência fora da localidade de sua lotação;
    XXV - acumular cargos ou funções, observados os permissivos constitucionais e legais.
     

    :]

  • Meio óbvio galera, não é uma conduta aceitável em nenhum trabalho sério, né? Mas tem sua base no Estatuto também: 

    Art. 157. Ao funcionário é proibido:

    XXII - tratar de assuntos particulares na repartição durante o horário de expediente.

    GABARITO C.

  • Uma dessa não cai na minha prova, porém nem compensa qualquer um acertaria...

  • Art. 157. Ao funcionário é proibido:

    XXII - tratar de assuntos particulares na repartição durante o horário de expediente;

    R:C

  • A questão exigiu conhecimentos sobre o Estatuto dos Servidores Públicos do Poder Judiciário do Estado do Paraná. Nos termos do art. 157, é proibido ao servidor público tratar de assuntos particulares na repartição durante o horário de expediente.

     

    Vejamos:

     

    Art. 157. Ao funcionário é proibido:

    (...)

    XXII - tratar de assuntos particulares na repartição durante o horário de expediente.

     

    Logo, o gabarito é a alternativa C.


ID
2478679
Banca
PUC-PR
Órgão
TJ-PR
Ano
2017
Provas
Disciplina
Direito da Criança e do Adolescente - Estatuto da Criança e do Adolescente (ECA) - Lei nº 8.069 de 1990
Assuntos

Quando for verificada alguma hipótese de violência ou ameaça aos direitos da criança e do adolescente reconhecidos no Estatuto da Criança e do Adolescente (Lei 8.069/90), a autoridade competente poderá determinar algumas medidas expressamente descritas na legislação. Dentre as opções a seguir, assinale a medida que NÃO PODERÁ ser adotada pela autoridade competente.

Alternativas
Comentários
  • Art. 101. Verificada qualquer das hipóteses previstas no art. 98, a autoridade competente poderá determinar, dentre outras, as seguintes medidas:

    I - encaminhamento aos pais ou responsável, mediante termo de responsabilidade;

    II - orientação, apoio e acompanhamento temporários;

    III - matrícula e freqüência obrigatórias em estabelecimento oficial de ensino fundamental;

    IV - inclusão em serviços e programas oficiais ou comunitários de proteção, apoio e promoção da família, da criança e do adolescente;

    V - requisição de tratamento médico, psicológico ou psiquiátrico, em regime hospitalar ou ambulatorial;

    VI - inclusão em programa oficial ou comunitário de auxílio, orientação e tratamento a alcoólatras e toxicômanos;

    VII - acolhimento institucional;

    VIII - inclusão em programa de acolhimento familiar;

    IX - colocação em família substituta.

  •  

            § 1o  O acolhimento institucional e o acolhimento familiar são medidas provisórias e excepcionais, utilizáveis como forma de transição para reintegração familiar ou, não sendo esta possível, para colocação em família substituta, não implicando privação de liberdade.  

  • Abrigo em entidade foi revogado: Art. 101, VII, ECA

  • Inicialmente, é importante destacar que a questão pede a alternativa INCORRETA acerca das medidas que podem ser adotadas pela autoridade competente diante de criança ou adolescente em situação e risco.

    Também é importante conhecermos o teor dos artigos 98 e 101 do ECA (Lei 8.069/90):

    Art. 98. As medidas de proteção à criança e ao adolescente são aplicáveis sempre que os direitos reconhecidos nesta Lei forem ameaçados ou violados:

            I - por ação ou omissão da sociedade ou do Estado;

            II - por falta, omissão ou abuso dos pais ou responsável;

            III - em razão de sua conduta.

    Art. 101. Verificada qualquer das hipóteses previstas no art. 98, a autoridade competente poderá determinar, dentre outras, as seguintes medidas:         I - encaminhamento aos pais ou responsável, mediante termo de responsabilidade;

            II - orientação, apoio e acompanhamento temporários;

            III - matrícula e frequência obrigatórias em estabelecimento oficial de ensino fundamental;

            IV - inclusão em serviços e programas oficiais ou comunitários de proteção, apoio e promoção da família, da criança e do adolescente;            (Redação dada pela Lei nº 13.257, de 2016)

            V - requisição de tratamento médico, psicológico ou psiquiátrico, em regime hospitalar ou ambulatorial;

            VI - inclusão em programa oficial ou comunitário de auxílio, orientação e tratamento a alcoólatras e toxicômanos;

            VII - acolhimento institucional;           (Redação dada pela Lei nº 12.010, de 2009)   Vigência

            VIII - inclusão em programa de acolhimento familiar;         (Redação dada pela Lei nº 12.010, de 2009)   Vigência

            IX - colocação em família substituta.         (Incluído pela Lei nº 12.010, de 2009)   Vigência

            § 1o  O acolhimento institucional e o acolhimento familiar são medidas provisórias e excepcionais, utilizáveis como forma de transição para reintegração familiar ou, não sendo esta possível, para colocação em família substituta, não implicando privação de liberdade.         (Incluído pela Lei nº 12.010, de 2009)   Vigência

            § 2o  Sem prejuízo da tomada de medidas emergenciais para proteção de vítimas de violência ou abuso sexual e das providências a que alude o art. 130 desta Lei, o afastamento da criança ou adolescente do convívio familiar é de competência exclusiva da autoridade judiciária e importará na deflagração, a pedido do Ministério Público ou de quem tenha legítimo interesse, de procedimento judicial contencioso, no qual se garanta aos pais ou ao responsável legal o exercício do contraditório e da ampla defesa.        (Incluído pela Lei nº 12.010, de 2009)   Vigência

            § 3o  Crianças e adolescentes somente poderão ser encaminhados às instituições que executam programas de acolhimento institucional, governamentais ou não, por meio de uma Guia de Acolhimento, expedida pela autoridade judiciária, na qual obrigatoriamente constará, dentre outros:            (Incluído pela Lei nº 12.010, de 2009)   Vigência

            I - sua identificação e a qualificação completa de seus pais ou de seu responsável, se conhecidos;        (Incluído pela Lei nº 12.010, de 2009)   Vigência

            II - o endereço de residência dos pais ou do responsável, com pontos de referência;        (Incluído pela Lei nº 12.010, de 2009)   Vigência

            III - os nomes de parentes ou de terceiros interessados em tê-los sob sua guarda;        (Incluído pela Lei nº 12.010, de 2009)   Vigência

            IV - os motivos da retirada ou da não reintegração ao convívio familiar.        (Incluído pela Lei nº 12.010, de 2009)   Vigência

            § 4o  Imediatamente após o acolhimento da criança ou do adolescente, a entidade responsável pelo programa de acolhimento institucional ou familiar elaborará um plano individual de atendimento, visando à reintegração familiar, ressalvada a existência de ordem escrita e fundamentada em contrário de autoridade judiciária competente, caso em que também deverá contemplar sua colocação em família substituta, observadas as regras e princípios desta Lei.        (Incluído pela Lei nº 12.010, de 2009)   Vigência

            § 5o  O plano individual será elaborado sob a responsabilidade da equipe técnica do respectivo programa de atendimento e levará em consideração a opinião da criança ou do adolescente e a oitiva dos pais ou do responsável.        (Incluído pela Lei nº 12.010, de 2009)   Vigência

            § 6o  Constarão do plano individual, dentre outros:       (Incluído pela Lei nº 12.010, de 2009)   Vigência

            I - os resultados da avaliação interdisciplinar;       (Incluído pela Lei nº 12.010, de 2009)   Vigência

            II - os compromissos assumidos pelos pais ou responsável; e       (Incluído pela Lei nº 12.010, de 2009)   Vigência

            III - a previsão das atividades a serem desenvolvidas com a criança ou com o adolescente acolhido e seus pais ou responsável, com vista na reintegração familiar ou, caso seja esta vedada por expressa e fundamentada determinação judicial, as providências a serem tomadas para sua colocação em família substituta, sob direta supervisão da autoridade judiciária.         (Incluído pela Lei nº 12.010, de 2009)   Vigência

            § 7o  O acolhimento familiar ou institucional ocorrerá no local mais próximo à residência dos pais ou do responsável e, como parte do processo de reintegração familiar, sempre que identificada a necessidade, a família de origem será incluída em programas oficiais de orientação, de apoio e de promoção social, sendo facilitado e estimulado o contato com a criança ou com o adolescente acolhido.        (Incluído pela Lei nº 12.010, de 2009)   Vigência

                § 8o  Verificada a possibilidade de reintegração familiar, o responsável pelo programa de acolhimento familiar ou institucional fará imediata comunicação à autoridade judiciária, que dará vista ao Ministério Público, pelo prazo de 5 (cinco) dias, decidindo em igual prazo.         (Incluído pela Lei nº 12.010, de 2009)   Vigência

            § 9o  Em sendo constatada a impossibilidade de reintegração da criança ou do adolescente à família de origem, após seu encaminhamento a programas oficiais ou comunitários de orientação, apoio e promoção social, será enviado relatório fundamentado ao Ministério Público, no qual conste a descrição pormenorizada das providências tomadas e a expressa recomendação, subscrita pelos técnicos da entidade ou responsáveis pela execução da política municipal de garantia do direito à convivência familiar, para a destituição do poder familiar, ou destituição de tutela ou guarda.        (Incluído pela Lei nº 12.010, de 2009)   Vigência

            § 10.  Recebido o relatório, o Ministério Público terá o prazo de 30 (trinta) dias para o ingresso com a ação de destituição do poder familiar, salvo se entender necessária a realização de estudos complementares ou outras providências que entender indispensáveis ao ajuizamento da demanda.        (Incluído pela Lei nº 12.010, de 2009)   Vigência

            § 11.  A autoridade judiciária manterá, em cada comarca ou foro regional, um cadastro contendo informações atualizadas sobre as crianças e adolescentes em regime de acolhimento familiar e institucional sob sua responsabilidade, com informações pormenorizadas sobre a situação jurídica de cada um, bem como as providências tomadas para sua reintegração familiar ou colocação em família substituta, em qualquer das modalidades previstas no art. 28 desta Lei.          (Incluído pela Lei nº 12.010, de 2009)   Vigência

            § 12.  Terão acesso ao cadastro o Ministério Público, o Conselho Tutelar, o órgão gestor da Assistência Social e os Conselhos Municipais dos Direitos da Criança e do Adolescente e da Assistência Social, aos quais incumbe deliberar sobre a implementação de políticas públicas que permitam reduzir o número de crianças e adolescentes afastados do convívio familiar e abreviar o período de permanência em programa de acolhimento.             (Incluído pela Lei nº 12.010, de 2009)   Vigência


    Feitos esses destaques, analisaremos abaixo cada uma das alternativas.
    ___________________________________________________________________________
    A) Matrícula e frequência obrigatórias em estabelecimento oficial de ensino fundamental. 

    A alternativa A está CORRETA, conforme artigo 101, inciso III, do ECA (Lei 8.069/90) (acima transcrito).
    ___________________________________________________________________________
    C) Inclusão em programa de acolhimento familiar.  

    A alternativa C está CORRETA, conforme artigo 101, inciso VIII, do ECA (Lei 8.069/90) (acima transcrito).
    ___________________________________________________________________________
    D) Colocação em família substituta. 

    A alternativa D está CORRETA, conforme artigo 101, inciso IX, do ECA (Lei 8.069/90) (acima transcrito).
    ___________________________________________________________________________
    E) Requisição de tratamento médico, psicológico ou psiquiátrico, em regime hospitalar ou ambulatorial.

    A alternativa E está CORRETA, conforme artigo 101, inciso V, do ECA (Lei 8.069/90) (acima transcrito).

    ___________________________________________________________________________
    B) Encaminhamento da criança ou do adolescente para abrigo em entidade, com rompimento do vínculo familiar e privação de liberdade.  

    A alternativa B está INCORRETA, pois não consta entre as medidas de proteção previstas no artigo 101 do ECA (Lei 8.069/90) (acima transcrito). Além disso, o §1º do artigo 101 do ECA (acima transcrito) estabelece expressamente que o acolhimento institucional e o acolhimento familiar são medidas provisórias e excepcionais, utilizáveis como forma de transição para reintegração familiar (ou seja, o rompimento do vínculo familiar não é medida almejada pelo ECA) ou, não sendo esta possível, para colocação em família substituta, não implicando privação de liberdade.
    ___________________________________________________________________________
    Resposta: ALTERNATIVA B
  • A  Lei nº 12.010, de 2009 alterou a redação do inc. VII do art. 101 de VII - abrigo em entidade; para   VII - acolhimento institucional;

  • Resposta "B" tendo como fundamentação o Art. 101, § 1°.

  • A Medidas de proteção não tiram a liberdade da criança e do adolescente meu caros ! A questão B) , vetusta alude que haverá privação de liberdade, mas ocorre que este não é o objetivo das medidas de proteção pelo princípio do interesse superior da criança e do adolescente eluciados no artigo 100 do ECA INCISO IV. 

  • Juro que to até agora sem entender essa questão. 

    Só acertei pq maquei vi escrito ABRIGO e sei que abrigo não existe mais. Porém, ainda não entendi bem essa questão.

  • Nazaré Confunsa,

    Acessado a Lei 8.069/90 - por meio do site do planalto - fazendo um Crtl + F - digitando a palavra vínculo - verificará que tal palavra aparecerá 16 vezes.

    Isso na data de hoje, haja vista possíveis atualizações na lei.

    Assim sendo, mata-se a questão pelo simples detalhe da letra B: "com rompimento do vínculo familiar".

    Espero ter ajudado.

    Bons estudos.

     

     

  • Criança não tem privação de liberdade.

  • GAB.: B

    Tenho percebido que as questões gostam de apresentar a colocação em entidade ou abrigo como medida de proteção para confundir com o acolhimento institucional, por isso ATENÇÃO.

  • Galera, resumindo ABRIGO NÃO

  • Não conseguir compreender esta questão, mais após a leitura dos comentário, é claro tudo ficou transparente.

    alternativa B - é correta pois:

    B- Encaminhamento da criança ou do adolescente para abrigo em entidade, com rompimento do vínculo familiar e privação de liberdade.

    CHAVE: criança não pode para ABRIGO e sim acolhimento institucional

    Art. 90. As entidades de atendimento são responsáveis pela manutenção das próprias unidades, assim como pelo planejamento e execução de programas de proteção e sócio-educativos destinados a crianças e adolescentes, em regime de: 

    I - orientação e apoio sócio-familiar;

    II - apoio sócio-educativo em meio aberto;

    III - colocação familiar;

    I V - abrigo;

    IV - acolhimento institucional; 

    V - liberdade assistida;

    V - prestação de serviços à comunidade; 

    VI - semi-liberdade;

    VI - liberdade assistida; 

    VII - internação.

    VII - semiliberdade; e 

    VIII - internação


ID
2478682
Banca
PUC-PR
Órgão
TJ-PR
Ano
2017
Provas
Disciplina
Psicologia
Assuntos

De acordo com as disposições da Lei de Guarda Compartilhada (Lei 13.058/2014), leia as assertivas a seguir e assinale a alternativa CORRETA.

I. Na guarda compartilhada, o tempo de convívio com os filhos deve ser dividido de forma equilibrada com a mãe e com o pai, sempre tendo em vista as condições fáticas e os interesses dos filhos.

II. Quando não houver acordo entre a mãe e o pai quanto à guarda do filho, encontrando-se ambos os genitores aptos a exercer o poder familiar, deverá ser aplicada a guarda unicamente à mãe.

III. Na guarda compartilhada, a cidade considerada base de moradia dos filhos será aquela que melhor atender aos interesses do pai ou da mãe.

IV. Se o juiz verificar que o filho não deve permanecer sob a guarda do pai ou da mãe, deferirá a guarda a pessoa que revele compatibilidade com a natureza da medida, considerados, de preferência, o grau de parentesco e as relações de afinidade e afetividade.

Alternativas
Comentários
  • e) Apenas as assertivas I e IV estão corretas. 

  • Gente, qdo for comentar a resposta não vale nada vc repetir a resposta é sim mostrar os erros das assertivas erradas.Isso ajuda no estudo

  • I. Na guarda compartilhada, o tempo de convívio com os filhos deve ser dividido de forma equilibrada com a mãe e com o pai, sempre tendo em vista as condições fáticas e os interesses dos filhos. CERTA

     

    II. Quando não houver acordo entre a mãe e o pai quanto à guarda do filho, encontrando-se ambos os genitores aptos a exercer o poder familiar, deverá ser aplicada a guarda unicamente à mãe. ( § 2o  Quando não houver acordo entre a mãe e o pai quanto à guarda do filho, encontrando-se ambos os genitores aptos a exercer o poder familiar, será aplicada a guarda compartilhada, salvo se um dos genitores declarar ao magistrado que não deseja a guarda do menor. LEI Nº 13.058, DE 22 DE DEZEMBRO DE 2014. )

     

    III. Na guarda compartilhada, a cidade considerada base de moradia dos filhos será aquela que melhor atender aos interesses do pai ou da mãe. ( § 3º  Na guarda compartilhada, a cidade considerada base de moradia dos filhos será aquela que melhor atender aos interesses dos filhos LEI Nº 13.058, DE 22 DE DEZEMBRO DE 2014. )

     

    IV. Se o juiz verificar que o filho não deve permanecer sob a guarda do pai ou da mãe, deferirá a guarda a pessoa que revele compatibilidade com a natureza da medida, considerados, de preferência, o grau de parentesco e as relações de afinidade e afetividade.  CERTA

  • LEI Nº 13.058/2014

    Somente as assertivas I e IV estão corretas. Vejamos a correção das demais assertivas:

    • II) Quando não houver acordo entre a mãe e o pai quanto à guarda do filho, encontrando-se ambos os genitores aptos a exercer o poder familiar, será aplicada a guarda compartilhada, salvo se um dos genitores declarar ao magistrado que não deseja a guarda do menor (Art 1.584, §2º);

    • III) Na guarda compartilhada, a cidade considerada base de moradia dos filhos será aquela que melhor atender aos interesses dos filhos (Art 1.583, §3º).

    Gabarito: E


ID
2478685
Banca
PUC-PR
Órgão
TJ-PR
Ano
2017
Provas
Disciplina
Direito da Criança e do Adolescente - Estatuto da Criança e do Adolescente (ECA) - Lei nº 8.069 de 1990
Assuntos

A Lei 12.852/2013, que institui o Estatuto da Juventude, estabelece a garantia de alguns direitos às pessoas entre 15 (quinze) e 29 (vinte e nove) anos, consideradas como jovens para efeitos legais. De acordo com as suas disposições, assinale a alternativa que indica um desses direitos expressamente fixados na referida legislação:

Alternativas
Comentários
  • a) Seção VII da lei 12.852 : Do Direito à Comunicação e à Liberdade de Expressão

     

    b)Não existe esta previsão no Estatuto da Juventude. O que é previsto são garantias e direitos aos jovens egressos do sistema prisional, que são afirmados nas diretrizes gerais e na parte da política de segurança pública,conforme explicitado abaixo: 

    É diretriz do Estatuto no art.3º:

    XI - zelar pelos direitos dos jovens com idade entre 18 (dezoito) e 29 (vinte e nove) anos privados de liberdade e egressos do sistema prisional, formulando políticas de educação e trabalho, incluindo estímulos à sua reinserção social e laboral, bem como criando e estimulando oportunidades de estudo e trabalho que favoreçam o cumprimento do regime semiaberto.

    É também diretriz da política de segurança pública prevista no art.38º:

    IV - a priorização de ações voltadas para os jovens em situação de risco, vulnerabilidade social e egressos do sistema penitenciário nacional.

     

    c)Não existe esta previsão no Estatuto.O que é especificado no Estatudo da juventude é o que está previsto no art.32º:

    Art. 32. No sistema de transporte coletivo interestadual, observar-se-á, nos termos da legislação específica:

    I - a reserva de 2 (duas) vagas gratuitas por veículo para jovens de baixa renda;
    II - a reserva de 2 (duas) vagas por veículo com desconto de 50% (cinquenta por cento), no mínimo, no valor das
    passagens, para os jovens de baixa renda, a serem utilizadas após esgotadas as vagas previstas no inciso I.


    Parágrafo único. Os procedimentos e os critérios para o exercício dos direitos previstos nos incisos I e II serão
    definidos em regulamento.

     

    d)Não existe esta previsão no Estatuto. O que está garantido pelo Estatuto da juventude nas diretrizes da política pública de atenção à saúde do jovem no art.20º é:

    I - acesso universal e gratuito ao Sistema Único de Saúde - SUS e a serviços de saúde humanizados e de qualidade, que respeitem as especificidades do jovem.

     

    e)Não existe esta previsão no Estatuto. O que é assegurado pelo art.23º é o seguinte:

    Art. 23. É assegurado aos jovens de até 29 (vinte e nove) anos pertencentes a famílias de baixa renda e aos estudantes, na forma do regulamento, o acesso a salas de cinema, cineclubes, teatros, espetáculos musicais e circenses, eventos educativos, esportivos, de lazer e entretenimento, em todo o território nacional, promovidos por quaisquer entidades e realizados em estabelecimentos públicos ou particulares, mediante pagamento da metade do
    preço do ingresso cobrado do público em geral.

    § 1o Terão direito ao benefício previsto no caput os estudantes regularmente matriculados nos níveis e modalidades de educação e ensino previstos no Título V da Lei no 9.394, de 20 de dezembro de 1996 - Lei de Diretrizes e Bases da Educação Nacional, que comprovem sua condição de discente, mediante apresentação, no momento da aquisição do ingresso e na portaria do local de realização do evento, da Carteira de Identificação Estudantil - CIE.

  • GB A

    PMGO

  • GABARITO -A

    Seção VII

    Do Direito à Comunicação e à Liberdade de Expressão

    Art. 26. O jovem tem direito à comunicação e à livre expressão, à produção de conteúdo, individual e colaborativo, e ao acesso às tecnologias de informação e comunicação.

    Bons estudos!


ID
2478688
Banca
PUC-PR
Órgão
TJ-PR
Ano
2017
Provas
Disciplina
Direito da Criança e do Adolescente - Estatuto da Criança e do Adolescente (ECA) - Lei nº 8.069 de 1990
Assuntos

Sobre o Estatuto da Juventude, leia as assertivas a seguir e assinale a alternativa CORRETA.

I. As diretrizes gerais relativas a políticas públicas de juventude cabem unicamente aos agentes públicos.

II. Um dos princípios estabelecidos na legislação é o da valorização do convívio do jovem com as demais gerações.

III. Uma das políticas públicas de saúde dos jovens tem como diretriz a habilitação dos profissionais de assistência social para a identificação dos problemas relacionados ao uso abusivo e à dependência de álcool, tabaco e outras drogas.

IV. A inclusão digital dos jovens garante a obtenção gratuita de computadores de última geração e de acesso sem custo à internet.

Alternativas
Comentários
  • GABARITO LETRA D, APENAS II E III ESTÃO CORRETAS

    Todas com fundamento na Lei 12.852/13 (Estatuto da Juventude)

     

    I. INCORRETA. “As diretrizes gerais relativas a políticas públicas de juventude cabem unicamente aos agentes públicos”. “DIRETRIZES GERAIS. Art. 3º  Os agentes públicos ou privados envolvidos com políticas públicas de juventude devem observar as seguintes diretrizes: (...)”

     

    II. CORRETA. “Um dos princípios estabelecidos na legislação é o da valorização do convívio do jovem com as demais gerações”.  Art. 2º  O disposto nesta Lei e as políticas públicas de juventude são regidos pelos seguintes PRINCÍPIOS:  (...) VIII - valorização do diálogo e convívio do jovem com as demais gerações”.

     

    III. CORRETA. “Uma das políticas públicas de saúde dos jovens tem como diretriz a habilitação dos profissionais de assistência social para a identificação dos problemas relacionados ao uso abusivo e à dependência de álcool, tabaco e outras drogas”. “Art. 20.  A política pública de atenção à saúde do jovem será desenvolvida em consonância com as seguintes diretrizes: (...) VII - habilitação dos professores e profissionais de saúde e de assistência social para a identificação dos problemas relacionados ao uso abusivo e à dependência de álcool, tabaco e outras drogas e o devido encaminhamento aos serviços assistenciais e de saúde”;

     

    IV. INCORRETA. “A inclusão digital dos jovens garante a obtenção gratuita de computadores de última geração e de acesso sem custo à Internet” . Não existe essa previsão na Lei. Art. 22.  Na consecução dos direitos culturais da juventude, compete ao poder público: (...) VII - promover a inclusão digital dos jovens, por meio do acesso às novas tecnologias da informação e comunicação;”.

  • A questão exige o conhecimento de diversos dispositivos previstos no Estatuto da Juventude, e pede que o candidato assinale a alternativa correta. Vamos aos itens:

    I - incorreto. Os agentes privados, assim como os públicos, também devem observar as diretrizes gerais.

    Seção II - diretrizes gerais. Art. 3º: os agentes públicos ou privados envolvidos com políticas públicas de juventude devem observar as seguintes diretrizes: (...)

    II - correto. Art. 2º, VIII: o disposto nesta lei e as políticas públicas de juventude são regidos pelos seguintes princípios: valorização do diálogo e convívio do jovem com as demais gerações.

    III - correto. Art. 20, VII: a política pública de atenção à saúde do jovem será desenvolvida em consonância com as seguintes diretrizes: habilitação dos professores e profissionais de saúde e de assistência social para a identificação dos problemas relacionados ao uso abusivo e à dependência de álcool, tabaco e outras drogas e o devido encaminhamento aos serviços assistenciais e de saúde.

    IV - incorreto. De fato, o Estatuto assegura ao jovem a inclusão digital. Entretanto, não menciona nada sobre a disponibilização gratuita de computadores de última geração, bem como do acesso sem custo à internet.

    Art. 22, VII: na consecução dos direitos culturais da juventude, compete ao poder público: promover a inclusão digital dos jovens, por meio do acesso às novas tecnologias da informação e comunicação.

    Art. 27, II: a ação do poder público na efetivação do direito do jovem à comunicação e à liberdade de expressão contempla a adoção das seguintes medidas: promover a inclusão digital dos jovens, por meio do acesso às novas tecnologias de informação e comunicação.

    Gabarito: D

  • A opção III está incompleta.


ID
2478691
Banca
PUC-PR
Órgão
TJ-PR
Ano
2017
Provas
Disciplina
Estatuto do Idoso - Lei nº 10.741 de 2003
Assuntos

Analise as assertivas acerca do Estatuto do Idoso e assinale a alternativa CORRETA.

I. Pelo Estatuto do Idoso, o envelhecimento foi fixado como direito personalíssimo.

II. Se o idoso ou seus familiares não possuírem condições econômicas de prover o seu sustento, impõe-se ao Poder Público esse provimento, no âmbito da assistência social.

III. Incumbe ao Poder Público fornecer aos idosos, gratuitamente, medicamentos, especialmente os de uso continuado, assim como próteses, órteses e outros recursos relativos ao tratamento, habilitação ou reabilitação.

IV. É vedado exigir o comparecimento do idoso enfermo perante os órgãos públicos.

Alternativas
Comentários
  • GABARITO: C todas estão corretas

     

    I. Art. 8o O envelhecimento é um direito personalíssimo e a sua proteção um direito social, nos termos desta Lei e da legislação vigente.

     

    II. Art. 14. Se o idoso ou seus familiares não possuírem condições econômicas de prover o seu sustento, impõe-se ao Poder Público esse provimento, no âmbito da assistência social.

     

    III. Art. 15, § 2o Incumbe ao Poder Público fornecer aos idosos, gratuitamente, medicamentos, especialmente os de uso continuado, assim como próteses, órteses e outros recursos relativos ao tratamento, habilitação ou reabilitação.

     

    IV. Art. 15,§ 5o É vedado exigir o comparecimento do idoso enfermo perante os órgãos públicos, hipótese na qual será admitido o seguinte procedimento: (...)

  • A questão trata do Estatuto do Idoso.

    I. Pelo Estatuto do Idoso, o envelhecimento foi fixado como direito personalíssimo.

    Lei nº 10.741 – Estatuto do Idoso:

      Art. 8o O envelhecimento é um direito personalíssimo e a sua proteção um direito social, nos termos desta Lei e da legislação vigente.

    Correta assertiva I.      

    II. Se o idoso ou seus familiares não possuírem condições econômicas de prover o seu sustento, impõe-se ao Poder Público esse provimento, no âmbito da assistência social.

    Lei nº 10.741 – Estatuto do Idoso:

    Art. 14. Se o idoso ou seus familiares não possuírem condições econômicas de prover o seu sustento, impõe-se ao Poder Público esse provimento, no âmbito da assistência social.

    Correta assertiva II.

    III. Incumbe ao Poder Público fornecer aos idosos, gratuitamente, medicamentos, especialmente os de uso continuado, assim como próteses, órteses e outros recursos relativos ao tratamento, habilitação ou reabilitação.

    Lei nº 10.741 – Estatuto do Idoso:

    Art. 15    § 2o Incumbe ao Poder Público fornecer aos idosos, gratuitamente, medicamentos, especialmente os de uso continuado, assim como próteses, órteses e outros recursos relativos ao tratamento, habilitação ou reabilitação.

    Correta assertiva III.   

    IV. É vedado exigir o comparecimento do idoso enfermo perante os órgãos públicos.  

    Lei nº 10.741 – Estatuto do Idoso:

    Art. 15. § 5o É vedado exigir o comparecimento do idoso enfermo perante os órgãos públicos, hipótese na qual será admitido o seguinte procedimento:       (Incluído pela Lei nº 12.896, de 2013)


    Correta assertiva IV.


    A) Apenas as assertivas I e II estão corretas. Incorreta letra “A”.

    B) Apenas as assertivas II e IV estão corretas. Incorreta letra “B”.

    C) Todas as assertivas estão corretas.  Correta letra “C”. Gabarito da questão.

    D) Apenas as assertivas I, II e IV estão corretas. Incorreta letra “D”.


    E) Apenas as assertivas III e IV estão corretas. Incorreta letra “E”.

    Resposta: C

    Gabarito do Professor letra C.
  • Todas alternativas corretas 

  • OBS:

    ÓRTESES: Aparelho destinado a suprir ou corrigir a alteração morfológica de um órgão, de um membro ou de um segmento de um membro, ou a deficiência de uma função. 2. PRÓTESES: Aparelho ou dispositivo destinado a substituir um órgão, de um membro ou parte do membro destruído ou gravemente acometido. 

  • Artigo 15

    § 5 É vedado exigir o comparecimento do idoso enfermo perante os órgãos públicos, hipótese na qual será admitido o seguinte procedimento:

    I - quando de interesse do poder público, o agente promoverá o contato necessário com o idoso em sua residência; ou      

    II - quando de interesse do próprio idoso, este se fará representar por procurador legalmente constituído.       

  • Estatuto do idoso

    Art. 8 O envelhecimento é um direito personalíssimo e a sua proteção um direito social, nos termos desta Lei e da legislação vigente.

    Art. 14. Se o idoso ou seus familiares não possuírem condições econômicas de prover o seu sustento, impõe-se ao Poder Público esse provimento, no âmbito da assistência social.

     Art. 15. § 2 Incumbe ao Poder Público fornecer aos idosos, gratuitamente, medicamentos, especialmente os de uso continuado, assim como próteses, órteses e outros recursos relativos ao tratamento, habilitação ou reabilitação.

    § 5 É vedado exigir o comparecimento do idoso enfermo perante os órgãos públicos, hipótese na qual será admitido o seguinte procedimento:     

    I - quando de interesse do poder público, o agente promoverá o contato necessário com o idoso em sua residência

    II - quando de interesse do próprio idoso, este se fará representar por procurador legalmente constituído. 


ID
2478694
Banca
PUC-PR
Órgão
TJ-PR
Ano
2017
Provas
Disciplina
Estatuto do Idoso - Lei nº 10.741 de 2003
Assuntos

Sobre a Política de Atendimento ao Idoso prevista na Lei 10.741/2003 (Estatuto do Idoso), assinale a alternativa INCORRETA.

Alternativas
Comentários
  • GABARITO: B

     

     

    a) Correta. Art. 50. Constituem obrigações das entidades de atendimento: XIV – fornecer comprovante de depósito dos bens móveis que receberem dos idosos;

     

     

    b) Incorreta.   Art. 50. Constituem obrigações das entidades de atendimento: V – oferecer atendimento personalizado; e XVII – manter no quadro de pessoal profissionais com formação específica.

     

     

    c) Correta.  Art. 48. As entidades de atendimento são responsáveis pela manutenção das próprias unidades, observadas as normas de planejamento e execução emanadas do órgão competente da Política Nacional do Idoso, conforme a Lei no 8.842, de 1994.

     

     

    d) Correta. Art. 49 Parágrafo único. O dirigente de instituição prestadora de atendimento ao idoso responderá civil e criminalmente pelos atos que praticar em detrimento do idoso, sem prejuízo das sanções administrativas.

     

     

    e) Correta. Art. 68  § 4o A multa e a advertência serão impostas ao dirigente da entidade ou ao responsável pelo programa de atendimento.

  • A questão trata do Estatuto do Idoso.


    A) As entidades de atendimento devem fornecer comprovante de depósito dos bens móveis que receberem dos idosos. 

    Lei nº 10.741 – Estatuto do Idoso:

    Art. 50. Constituem obrigações das entidades de atendimento:

    XIV – fornecer comprovante de depósito dos bens móveis que receberem dos idosos;

    As entidades de atendimento devem fornecer comprovante de depósito dos bens móveis que receberem dos idosos. 

    Correta letra “A”. Gabarito da questão.      


    B) As entidades de atendimento ficam dispensadas de oferecer atendimento personalizado ao idoso e de manter no quadro de pessoal profissionais com formação específica.  

    Lei nº 10.741 – Estatuto do Idoso:

    Art. 50. Constituem obrigações das entidades de atendimento:

    V – oferecer atendimento personalizado;

    XVII – manter no quadro de pessoal profissionais com formação específica.

    As entidades de atendimento ficam obrigadas a oferecer atendimento personalizado ao idoso e manter no quadro de pessoal profissionais com formação específica.  

    Incorreta letra “B”. Gabarito da questão.

          
    C) As entidades de atendimento são responsáveis pela manutenção das próprias unidades, observadas as normas de planejamento e execução emanadas do órgão competente da Política Nacional do Idoso. 

    Lei nº 10.741 – Estatuto do Idoso:

    Art. 48. As entidades de atendimento são responsáveis pela manutenção das próprias unidades, observadas as normas de planejamento e execução emanadas do órgão competente da Política Nacional do Idoso, conforme a Lei no 8.842, de 1994.

    Correta letra “C”.


    D) O dirigente de instituição prestadora de atendimento ao idoso responderá civil e criminalmente pelos atos que praticar em detrimento do idoso, sem prejuízo das sanções administrativas. 

    Lei nº 10.741 – Estatuto do Idoso:

    Art. 49.   Parágrafo único. O dirigente de instituição prestadora de atendimento ao idoso responderá civil e criminalmente pelos atos que praticar em detrimento do idoso, sem prejuízo das sanções administrativas.

    O dirigente de instituição prestadora de atendimento ao idoso responderá civil e criminalmente pelos atos que praticar em detrimento do idoso, sem prejuízo das sanções administrativas. 

    Correta letra “D”.

    E) Quando houver apuração judicial de irregularidades em entidade de atendimento ao idoso, a multa e a advertência, quando impostas, serão direcionadas ao dirigente da entidade ou ao responsável pelo programa de atendimento. 

    Lei nº 10.741 – Estatuto do Idoso:

    Art. 68. Apresentada a defesa, o juiz procederá na conformidade do art. 69 ou, se necessário, designará audiência de instrução e julgamento, deliberando sobre a necessidade de produção de outras provas.

    § 4o A multa e a advertência serão impostas ao dirigente da entidade ou ao responsável pelo programa de atendimento.

    Quando houver apuração judicial de irregularidades em entidade de atendimento ao idoso, a multa e a advertência, quando impostas, serão direcionadas ao dirigente da entidade ou ao responsável pelo programa de atendimento. 

    Correta letra “E”.



    Resposta: B

    Gabarito do Professor letra B.


ID
2478697
Banca
PUC-PR
Órgão
TJ-PR
Ano
2017
Provas
Disciplina
Direito da Criança e do Adolescente - Estatuto da Criança e do Adolescente (ECA) - Lei nº 8.069 de 1990
Assuntos

Sobre a Alienação Parental, conforme disposições da Lei 12.318/2010, assinale a alternativa CORRETA.

Alternativas
Comentários
  • Art. 2o  Considera-se ato de alienação parental a interferência na formação psicológica da criança ou do adolescente promovida ou induzida por um dos genitores, pelos avós ou pelos que tenham a criança ou adolescente sob a sua autoridade, guarda ou vigilância para que repudie genitor ou que cause prejuízo ao estabelecimento ou à manutenção de vínculos com este. 

    Parágrafo único.  São formas exemplificativas de alienação parental, além dos atos assim declarados pelo juiz ou constatados por perícia, praticados diretamente ou com auxílio de terceiros:  

    I - realizar campanha de desqualificação da conduta do genitor no exercício da paternidade ou maternidade; 

    II - dificultar o exercício da autoridade parental; 

    III - dificultar contato de criança ou adolescente com genitor; 

    IV - dificultar o exercício do direito regulamentado de convivência familiar; 

    V - omitir deliberadamente a genitor informações pessoais relevantes sobre a criança ou adolescente, inclusive escolares, médicas e alterações de endereço; 

    VI - apresentar falsa denúncia contra genitor, contra familiares deste ou contra avós, para obstar ou dificultar a convivência deles com a criança ou adolescente; 

    VII - mudar o domicílio para local distante, sem justificativa, visando a dificultar a convivência da criança ou adolescente com o outro genitor, com familiares deste ou com avós.

  • POR FAVOR, COLOQUEM O GABARITO PESSOAL.

     

    GABARITO LETRA ( A )!!!!!!!

  • LEI Nº 12.318/2010

    Vejamos a correção das assertivas incorretas:

    • b) a AP prejudica a realização de afeto nas relações com genitor e com o grupo familiar (Art. 3º);
    • c) os atos podem ser praticados diretamente ou com auxílio de terceiros (Art. 2º, § único);
    • d) além dos atos assim declarados pelo juiz ou constatados por perícia (Art. 2º, § único);
    • e) o laudo pericial terá base em ampla avaliação psicológica, compreendendo inclusive o exame da forma como a criança ou adolescente se manifesta acerca de eventual acusação contra genitor (Art. 5º, §1º);

    Gabarito: A

  • #DICA: ALIENAÇÃO PODE SER CONSTATADA DIRETAMENTE PELO JUIZ OU PELA PERÍCIA = LOGO, DEVE-SE INFERIR QUE ELA É DISPENSÁVEL PARA DECISÃO JUDICIAL

  • A questão exige o conhecimento sobre a alienação parental, com previsão na lei nº 12.318/10, e pede que o candidato assinale a alternativa correta. Veja:

    A - correta. Art. 2º, parágrafo único, VII: são formas exemplificativas de alienação parental, além dos atos assim declarados pelo juiz ou constatados por perícia, praticados diretamente ou com auxílio de terceiros: mudar o domicílio para local distante, sem justificativa, visando dificultar a convivência da criança ou adolescente com o outro genitor, com familiares deste ou com avós.

    B - incorreta. A alienação parental também se justifica quando há o prejuízo na relação entre netos e avós, uma vez que eles são considerados como integrantes do grupo familiar.

    Art. 3º: a prática de ato de alienação parental fere direito fundamental da criança ou do adolescente de convivência familiar saudável, prejudica a realização de afeto nas relações com genitor e com o grupo familiar, constitui abuso moral contra a criança ou o adolescente e descumprimento dos deveres inerentes à autoridade parental ou decorrentes de tutela ou guarda.

    C - incorreta. A alienação parental também pode ser promovida com o auxílio de terceiros.

    Art. 2º, parágrafo único: são formas exemplificativas de alienação parental, além dos atos assim declarados pelo juiz ou constatados por perícia, praticados diretamente ou com auxílio de terceiros: (...)

    D - incorreta. Não somente os atos constatados pela perícia são considerados de alienação parental, mas também aqueles declarados pelo juiz.

    Art. 2º, parágrafo único: são formas exemplificativas de alienação parental, além dos atos assim declarados pelo juiz ou constatados por perícia, praticados diretamente ou com auxílio de terceiros: (...)

    E - incorreta. É justamente o contrário: o laudo pericial deve levar em conta a forma como a criança ou adolescente se manifesta acerca de eventual acusação contra genitor.

    Art. 5º, §1º: o laudo pericial terá base em ampla avaliação psicológica ou biopsicossocial, conforme o caso, compreendendo, inclusive, entrevista pessoal com as partes, exame de documentos dos autos, histórico do relacionamento do casal e da separação, cronologia de incidentes, avaliação da personalidade dos envolvidos e exame da forma como a criança ou adolescente se manifesta acerca de eventual acusação contra genitor.

    Gabarito: A


ID
2478700
Banca
PUC-PR
Órgão
TJ-PR
Ano
2017
Provas
Disciplina
Direito da Criança e do Adolescente - Estatuto da Criança e do Adolescente (ECA) - Lei nº 8.069 de 1990
Assuntos

Nos termos do Estatuto da Criança e do Adolescente, assinale a alternativa CORRETA.

Alternativas
Comentários
  • LEI Nº 8.069, DE 13 DE JULHO DE 1990.

    Art. 2º Considera-se criança, para os efeitos desta Lei, a pessoa até doze anos de idade incompletos, e adolescente aquela entre doze e dezoito anos de idade.

            Parágrafo único. Nos casos expressos em lei, aplica-se excepcionalmente este Estatuto às pessoas entre dezoito e vinte e um anos de idade.

    Mas cuidado com o enunciado. Acaso a questão exija texto da CONVENÇÃO SOBRE OS DIREITOS DA CRIANÇA, o entendimento é diferente:

    Artigo 1º - Para os efeitos da presente Convenção, entende-se por criança todo ser humano menor de 18 anos de idade, salvo se, em conformidade com a lei aplicável à criança, a maioridade seja alcançada antes.

  • CRIANÇA - até 12 anos incompletos

    ADOLESCENTE - entre 12 e 18 anos

    JOVEM ADULTO - entre 18 e 21 anos, porém só em casos expressos no ECA

  • A) Considera-se criança a pessoa com até 14 (quatorze) anos de idade incompletos.  

    A alternativa A está INCORRETA, pois, nos termos do artigo 2º, "caput", da Lei 8.069/90, considera-se criança a pessoa com até 12 (doze) anos de idade incompletos:

    Art. 2º Considera-se criança, para os efeitos desta Lei, a pessoa até doze anos de idade incompletos, e adolescente aquela entre doze e dezoito anos de idade. Parágrafo único. Nos casos expressos em lei, aplica-se excepcionalmente este Estatuto às pessoas entre dezoito e vinte e um anos de idade.
    ___________________________________________________________________________
    B) Considera-se adolescente qualquer pessoa que possua até 21 (vinte e um) anos de idade.  

    A alternativa B está INCORRETA, pois, nos termos do artigo 2º, "caput", da Lei 8.069/90, considera-se adolescente a pessoa entre doze e dezoito anos de idade:

    Art. 2º Considera-se criança, para os efeitos desta Lei, a pessoa até doze anos de idade incompletos, e adolescente aquela entre doze e dezoito anos de idade. Parágrafo único. Nos casos expressos em lei, aplica-se excepcionalmente este Estatuto às pessoas entre dezoito e vinte e um anos de idade.
    _____________________________________________________________________________
    C) Considera-se criança a pessoa com no máximo 10 (dez) anos de idade completos.

    A alternativa C está INCORRETA, pois, nos termos do artigo 2º, "caput", da Lei 8.069/90, considera-se criança a pessoa com até 12 (doze) anos de idade incompletos:

    Art. 2º Considera-se criança, para os efeitos desta Lei, a pessoa até doze anos de idade incompletos, e adolescente aquela entre doze e dezoito anos de idade. Parágrafo único. Nos casos expressos em lei, aplica-se excepcionalmente este Estatuto às pessoas entre dezoito e vinte e um anos de idade.
    ____________________________________________________________________________
    E) Considera-se adolescente qualquer pessoa com no máximo 16 (dezesseis) anos de idade.  

    A alternativa E está INCORRETA, pois, nos termos do artigo 2º, "caput", da Lei 8.069/90, considera-se adolescente a pessoa entre doze e dezoito anos de idade:

    Art. 2º Considera-se criança, para os efeitos desta Lei, a pessoa até doze anos de idade incompletos, e adolescente aquela entre doze e dezoito anos de idade. Parágrafo único. Nos casos expressos em lei, aplica-se excepcionalmente este Estatuto às pessoas entre dezoito e vinte e um anos de idade.
    ____________________________________________________________________________
    D) Considera-se adolescente a pessoa que possui entre 12 (doze) e 18 (dezoito) anos de idade. 

    A alternativa D está CORRETA, pois, nos termos do artigo 2º, "caput", da Lei 8.069/90, considera-se adolescente a pessoa entre doze e dezoito anos de idade:

    Art. 2º Considera-se criança, para os efeitos desta Lei, a pessoa até doze anos de idade incompletos, e adolescente aquela entre doze e dezoito anos de idade. Parágrafo único. Nos casos expressos em lei, aplica-se excepcionalmente este Estatuto às pessoas entre dezoito e vinte e um anos de idade.
    ______________________________________________________________________________
    Resposta: ALTERNATIVA D
  • Uma pergunta dessa para analista...

     

  • Art. 2º Considera-se criança, para os efeitos desta Lei, a pessoa até doze anos de idade incompletos, e adolescente aquela entre doze e dezoito anos de idade.

  • Art. 2º Considera-se criança, para os efeitos desta Lei, a pessoa até doze anos de idade incompletos, e adolescente aquela entre doze e dezoito anos de idade.

    O legislador ao definir criança se preocupou em colocar "até 12 (doze) anos incompletos, ou seja, chegou a ser redundante, colocando a palavra "até" e "incompletos", ao passo que ao definir adolescente "aquela entre 12 e 18 anos de idade, deixou a entender que a pessoa com 18 anos de idade ainda é adolescente, carecendo neste último caso, de acrescentar "aquela entre 12 e 18 anos incompletos", pois aos 18 anos adquiriu a maioridade civil e não é mais adolescente.

  • Gabarito letra D


ID
2478703
Banca
PUC-PR
Órgão
TJ-PR
Ano
2017
Provas
Disciplina
Direito Constitucional
Assuntos

Sobre os Direitos Fundamentais previstos na Constituição Federal de 1988, leia as assertivas a seguir e, depois, assinale a alternativa CORRETA.

I. A lei só poderá restringir a publicidade dos atos processuais quando a defesa da intimidade ou o interesse social o exigirem.

II. É assegurado a todos o acesso à informação e resguardado o sigilo da fonte quando necessário ao exercício profissional.

III. É livre a manifestação do pensamento, sendo vedado o anonimato.

IV. Todos têm direito a receber dos órgãos públicos informações de seu interesse particular, ou de interesse coletivo ou geral, que serão prestadas no prazo da lei, sob pena de responsabilidade, ressalvadas aquelas cujo sigilo seja imprescindível à segurança da sociedade e do Estado.

Alternativas
Comentários
  • Gabarito letra c).

     

    CONSTITUIÇÃO FEDERAL, ARTIGO 5°

     

     

    Item "I") LX - a lei só poderá restringir a publicidade dos atos processuais quando a defesa da intimidade ou o interesse social o exigirem.

     

     

    Item "II") XIV - é assegurado a todos o acesso à informação e resguardado o sigilo da fonte, quando necessário ao exercício profissional.

     

     

    Item "III") IV - é livre a manifestação do pensamento, sendo vedado o anonimato.

     

     

    Item "IV") XXXIII - todos têm direito a receber dos órgãos públicos informações de seu interesse particular, ou de interesse coletivo ou geral, que serão prestadas no prazo da lei, sob pena de responsabilidade, ressalvadas aquelas cujo sigilo seja imprescindível à segurança da sociedade e do Estado.

     

     

     

    => Meu Instagram para concursos: https://www.instagram.com/qdconcursos/

  • PESSOAL, TODAS AS ASSERTIVAS ESTÃO CORRETAS.

     

     

    GABARITO ''C''

  • Gabarito: LETRA C

     

    I. CORRETA! A lei só poderá restringir a publicidade dos atos processuais quando a defesa da intimidade ou o interesse social o exigirem.

    Art. 5º Todos são iguais perante a lei, sem distinção de qualquer natureza, garantindo-se aos brasileiros e aos estrangeiros residentes no País a inviolabilidade do direito à vida, à liberdade, à igualdade, à segurança e à propriedade, nos termos seguintes:

     LX - a lei só poderá restringir a publicidade dos atos processuais quando a defesa da intimidade ou o interesse social o exigirem;

     

    II. CORRETA! É assegurado a todos o acesso à informação e resguardado o sigilo da fonte quando necessário ao exercício profissional.

    Art. 5º Todos são iguais perante a lei, sem distinção de qualquer natureza, garantindo-se aos brasileiros e aos estrangeiros residentes no País a inviolabilidade do direito à vida, à liberdade, à igualdade, à segurança e à propriedade, nos termos seguintes:

     XIV - é assegurado a todos o acesso à informação e resguardado o sigilo da fonte, quando necessário ao exercício profissional;

     

    III. CORRETA! É livre a manifestação do pensamento, sendo vedado o anonimato.

    Art. 5º Todos são iguais perante a lei, sem distinção de qualquer natureza, garantindo-se aos brasileiros e aos estrangeiros residentes no País a inviolabilidade do direito à vida, à liberdade, à igualdade, à segurança e à propriedade, nos termos seguintes:

     IV - é livre a manifestação do pensamento, sendo vedado o anonimato;

     

    IV. CORRETA! Todos têm direito a receber dos órgãos públicos informações de seu interesse particular, ou de interesse coletivo ou geral, que serão prestadas no prazo da lei, sob pena de responsabilidade, ressalvadas aquelas cujo sigilo seja imprescindível à segurança da sociedade e do Estado.

    Art. 5º Todos são iguais perante a lei, sem distinção de qualquer natureza, garantindo-se aos brasileiros e aos estrangeiros residentes no País a inviolabilidade do direito à vida, à liberdade, à igualdade, à segurança e à propriedade, nos termos seguintes: 

    XXXIII - todos têm direito a receber dos órgãos públicos informações de seu interesse particular, ou de interesse coletivo ou geral, que serão prestadas no prazo da lei, sob pena de responsabilidade, ressalvadas aquelas cujo sigilo seja imprescindível à segurança da sociedade e do Estado; 

  • A questão aborda a temática dos direitos fundamentais positivados na Constituição Federal. Analisemos as assertivas:

    Assertiva I: está correta. Conforme art. 5º, LX – “a lei só poderá restringir a publicidade dos atos processuais quando a defesa da intimidade ou o interesse social o exigirem".

    Assertiva II: está correta. Conforme art. 5º, XIV – “é assegurado a todos o acesso à informação e resguardado o sigilo da fonte, quando necessário ao exercício profissional".

    Assertiva III: está correta. Conforme art. 5º, IV – “é livre a manifestação do pensamento, sendo vedado o anonimato".

    Assertiva IV: está correta. Conforme art. 5º, XXXIII – “todos têm direito a receber dos órgãos públicos informações de seu interesse particular, ou de interesse coletivo ou geral, que serão prestadas no prazo da lei, sob pena de responsabilidade, ressalvadas aquelas cujo sigilo seja imprescindível à segurança da sociedade e do Estado".

    Portanto, todas as assertivas estão corretas.

    Gabarito do professor: letra c.


ID
2478706
Banca
PUC-PR
Órgão
TJ-PR
Ano
2017
Provas
Disciplina
Direito da Criança e do Adolescente - Estatuto da Criança e do Adolescente (ECA) - Lei nº 8.069 de 1990
Assuntos

Sobre os direitos fundamentais à vida e à saúde da mulher gestante previstos no Estatuto da Criança e do Adolescente, leia as assertivas a seguir e, depois, assinale a alternativa CORRETA.

I. A gestante tem direito a 02 (dois) acompanhantes de sua preferência durante o período do pré-natal, do trabalho de parto e do pós-parto imediato.

II. Caso os profissionais de saúde de referência já tenham vinculado a gestante, no último trimestre da gestação, ao estabelecimento em que será realizado o parto, a mulher não tem o direito de optar por outro local.

III. A obrigação do poder público de proporcionar assistência psicológica à gestante e à mãe se limita ao período pré-natal.

IV. O atendimento pré-natal será realizado por profissionais da atenção primária.

Alternativas
Comentários
  • Art. 8º [...].

     

    § 6º A gestante e a parturiente têm direito a 1 (um) acompanhante de sua preferência durante o período do pré-natal, do trabalho de parto e do pós-parto imediato.

     

    § 2º Os profissionais de saúde de referência da gestante garantirão sua vinculação, no último trimestre da gestação, ao estabelecimento em que será realizado o parto, garantido o direito de opção da mulher.

     

    § 4º Incumbe ao poder público proporcionar assistência psicológica à gestante e à mãe, no período pré e pós-natal, inclusive como forma de prevenir ou minorar as consequências do estado puerperal.

     

    § 1º  O atendimento pré-natal será realizado por profissionais da atenção primária.   

     

    Gabarito: b.

  • Art. 8o  É assegurado a todas as mulheres o acesso aos programas e às políticas de saúde da mulher e de planejamento reprodutivo e, às gestantes, nutrição adequada, atenção humanizada à gravidez, ao parto e ao puerpério e atendimento pré-natal, perinatal e pós-natal integral no âmbito do Sistema Único de Saúde.          (Redação dada pela Lei nº 13.257, de 2016)

    § 1o  O atendimento pré-natal será realizado por profissionais da atenção primária.

    § 2o  Os profissionais de saúde de referência da gestante garantirão sua vinculação, no último trimestre da gestação, ao estabelecimento em que será realizado o parto, garantido o direito de opção da mulher.          (Redação dada pela Lei nº 13.257, de 2016)

    § 3o  Os serviços de saúde onde o parto for realizado assegurarão às mulheres e aos seus filhos recém-nascidos alta hospitalar responsável e contrarreferência na atenção primária, bem como o acesso a outros serviços e a grupos de apoio à amamentação.          (Redação dada pela Lei nº 13.257, de 2016)

            § 4o  Incumbe ao poder público proporcionar assistência psicológica à gestante e à mãe, no período pré e pós-natal, inclusive como forma de prevenir ou minorar as consequências do estado puerperal.       (Incluído pela Lei nº 12.010, de 2009)     Vigência

    § 5o  A assistência referida no § 4o deste artigo deverá ser prestada também a gestantes e mães que manifestem interesse em entregar seus filhos para adoção, bem como a gestantes e mães que se encontrem em situação de privação de liberdade.          (Redação dada pela Lei nº 13.257, de 2016)

    § 6o  A gestante e a parturiente têm direito a 1 (um) acompanhante de sua preferência durante o período do pré-natal, do trabalho de parto e do pós-parto imediato.          (Incluído pela Lei nº 13.257, de 2016)

    § 7o  A gestante deverá receber orientação sobre aleitamento materno, alimentação complementar saudável e crescimento e desenvolvimento infantil, bem como sobre formas de favorecer a criação de vínculos afetivos e de estimular o desenvolvimento integral da criança.          (Incluído pela Lei nº 13.257, de 2016)

    § 8o  A gestante tem direito a acompanhamento saudável durante toda a gestação e a parto natural cuidadoso, estabelecendo-se a aplicação de cesariana e outras intervenções cirúrgicas por motivos médicos.          (Incluído pela Lei nº 13.257, de 2016)

    § 9o  A atenção primária à saúde fará a busca ativa da gestante que não iniciar ou que abandonar as consultas de pré-natal, bem como da puérpera que não comparecer às consultas pós-parto.          (Incluído pela Lei nº 13.257, de 2016)

    § 10.  Incumbe ao poder público garantir, à gestante e à mulher com filho na primeira infância que se encontrem sob custódia em unidade de privação de liberdade, ambiência que atenda às normas sanitárias e assistenciais do Sistema Único de Saúde para o acolhimento do filho, em articulação com o sistema de ensino competente, visando ao desenvolvimento integral da criança.          (Incluído pela Lei nº 13.257, de 2016)

     

  • I. A gestante tem direito a 02 (dois) acompanhantes de sua preferência durante o período do pré-natal, do trabalho de parto e do pós-parto imediato.

    A assertiva I está INCORRETA, pois, nos termos do artigo 8º, §6º, do ECA (Lei 8.069/90), a gestante e a parturiente têm direito a apenas 1 (um) acompanhante de sua preferência durante o período do pré-natal, do trabalho de parto e do pós-parto imediato:

    Art. 8o  É assegurado a todas as mulheres o acesso aos programas e às políticas de saúde da mulher e de planejamento reprodutivo e, às gestantes, nutrição adequada, atenção humanizada à gravidez, ao parto e ao puerpério e atendimento pré-natal, perinatal e pós-natal integral no âmbito do Sistema Único de Saúde.          (Redação dada pela Lei nº 13.257, de 2016)

    § 1o  O atendimento pré-natal será realizado por profissionais da atenção primária.          (Redação dada pela Lei nº 13.257, de 2016)

    § 2o  Os profissionais de saúde de referência da gestante garantirão sua vinculação, no último trimestre da gestação, ao estabelecimento em que será realizado o parto, garantido o direito de opção da mulher.          (Redação dada pela Lei nº 13.257, de 2016)

    § 3o  Os serviços de saúde onde o parto for realizado assegurarão às mulheres e aos seus filhos recém-nascidos alta hospitalar responsável e contrarreferência na atenção primária, bem como o acesso a outros serviços e a grupos de apoio à amamentação.          (Redação dada pela Lei nº 13.257, de 2016)

            § 4o  Incumbe ao poder público proporcionar assistência psicológica à gestante e à mãe, no período pré e pós-natal, inclusive como forma de prevenir ou minorar as consequências do estado puerperal.       (Incluído pela Lei nº 12.010, de 2009)      Vigência

    § 5o  A assistência referida no § 4o deste artigo deverá ser prestada também a gestantes e mães que manifestem interesse em entregar seus filhos para adoção, bem como a gestantes e mães que se encontrem em situação de privação de liberdade.          (Redação dada pela Lei nº 13.257, de 2016)

    § 6o  A gestante e a parturiente têm direito a 1 (um) acompanhante de sua preferência durante o período do pré-natal, do trabalho de parto e do pós-parto imediato.          (Incluído pela Lei nº 13.257, de 2016)

    § 7o  A gestante deverá receber orientação sobre aleitamento materno, alimentação complementar saudável e crescimento e desenvolvimento infantil, bem como sobre formas de favorecer a criação de vínculos afetivos e de estimular o desenvolvimento integral da criança.          (Incluído pela Lei nº 13.257, de 2016)

    § 8o  A gestante tem direito a acompanhamento saudável durante toda a gestação e a parto natural cuidadoso, estabelecendo-se a aplicação de cesariana e outras intervenções cirúrgicas por motivos médicos.          (Incluído pela Lei nº 13.257, de 2016)

    § 9o  A atenção primária à saúde fará a busca ativa da gestante que não iniciar ou que abandonar as consultas de pré-natal, bem como da puérpera que não comparecer às consultas pós-parto.          (Incluído pela Lei nº 13.257, de 2016)

    § 10.  Incumbe ao poder público garantir, à gestante e à mulher com filho na primeira infância que se encontrem sob custódia em unidade de privação de liberdade, ambiência que atenda às normas sanitárias e assistenciais do Sistema Único de Saúde para o acolhimento do filho, em articulação com o sistema de ensino competente, visando ao desenvolvimento integral da criança.          (Incluído pela Lei nº 13.257, de 2016)

    __________________________________________________________________________
    II. Caso os profissionais de saúde de referência já tenham vinculado a gestante, no último trimestre da gestação, ao estabelecimento em que será realizado o parto, a mulher não tem o direito de optar por outro local. 

    A assertiva II está INCORRETA, pois, nos termos do artigo 8º, §2º, do ECA (Lei 8.069/90), mesmo que os profissionais de saúde de referência já tenham vinculado a gestante, no último trimestre da gestação, ao estabelecimento em que será realizado o parto, é garantido o direito de opção da mulher:

    Art. 8o  É assegurado a todas as mulheres o acesso aos programas e às políticas de saúde da mulher e de planejamento reprodutivo e, às gestantes, nutrição adequada, atenção humanizada à gravidez, ao parto e ao puerpério e atendimento pré-natal, perinatal e pós-natal integral no âmbito do Sistema Único de Saúde.          (Redação dada pela Lei nº 13.257, de 2016)

    § 1o  O atendimento pré-natal será realizado por profissionais da atenção primária.          (Redação dada pela Lei nº 13.257, de 2016)

    § 2o  Os profissionais de saúde de referência da gestante garantirão sua vinculação, no último trimestre da gestação, ao estabelecimento em que será realizado o parto, garantido o direito de opção da mulher.          (Redação dada pela Lei nº 13.257, de 2016)

    § 3o  Os serviços de saúde onde o parto for realizado assegurarão às mulheres e aos seus filhos recém-nascidos alta hospitalar responsável e contrarreferência na atenção primária, bem como o acesso a outros serviços e a grupos de apoio à amamentação.          (Redação dada pela Lei nº 13.257, de 2016)

            § 4o  Incumbe ao poder público proporcionar assistência psicológica à gestante e à mãe, no período pré e pós-natal, inclusive como forma de prevenir ou minorar as consequências do estado puerperal.       (Incluído pela Lei nº 12.010, de 2009)      Vigência

    § 5o  A assistência referida no § 4o deste artigo deverá ser prestada também a gestantes e mães que manifestem interesse em entregar seus filhos para adoção, bem como a gestantes e mães que se encontrem em situação de privação de liberdade.          (Redação dada pela Lei nº 13.257, de 2016)

    § 6o  A gestante e a parturiente têm direito a 1 (um) acompanhante de sua preferência durante o período do pré-natal, do trabalho de parto e do pós-parto imediato.          (Incluído pela Lei nº 13.257, de 2016)

    § 7o  A gestante deverá receber orientação sobre aleitamento materno, alimentação complementar saudável e crescimento e desenvolvimento infantil, bem como sobre formas de favorecer a criação de vínculos afetivos e de estimular o desenvolvimento integral da criança.          (Incluído pela Lei nº 13.257, de 2016)

    § 8o  A gestante tem direito a acompanhamento saudável durante toda a gestação e a parto natural cuidadoso, estabelecendo-se a aplicação de cesariana e outras intervenções cirúrgicas por motivos médicos.          (Incluído pela Lei nº 13.257, de 2016)

    § 9o  A atenção primária à saúde fará a busca ativa da gestante que não iniciar ou que abandonar as consultas de pré-natal, bem como da puérpera que não comparecer às consultas pós-parto.          (Incluído pela Lei nº 13.257, de 2016)

    § 10.  Incumbe ao poder público garantir, à gestante e à mulher com filho na primeira infância que se encontrem sob custódia em unidade de privação de liberdade, ambiência que atenda às normas sanitárias e assistenciais do Sistema Único de Saúde para o acolhimento do filho, em articulação com o sistema de ensino competente, visando ao desenvolvimento integral da criança.          (Incluído pela Lei nº 13.257, de 2016)

    ___________________________________________________________________________
    III. A obrigação do poder público de proporcionar assistência psicológica à gestante e à mãe se limita ao período pré-natal.

    A assertiva III está INCORRETA, pois, nos termos do artigo 8º, §4º, do ECA (Lei 8.069/90), a obrigação do poder público de proporcionar assistência psicológica à gestante e à mãe não se limita ao período pré-natal, devendo também ser proporcionada no período pós-natal, inclusive como forma de prevenir ou minorar as consequências do estado puerperal:

    Art. 8o  É assegurado a todas as mulheres o acesso aos programas e às políticas de saúde da mulher e de planejamento reprodutivo e, às gestantes, nutrição adequada, atenção humanizada à gravidez, ao parto e ao puerpério e atendimento pré-natal, perinatal e pós-natal integral no âmbito do Sistema Único de Saúde.          (Redação dada pela Lei nº 13.257, de 2016)

    § 1o  O atendimento pré-natal será realizado por profissionais da atenção primária.          (Redação dada pela Lei nº 13.257, de 2016)

    § 2o  Os profissionais de saúde de referência da gestante garantirão sua vinculação, no último trimestre da gestação, ao estabelecimento em que será realizado o parto, garantido o direito de opção da mulher.          (Redação dada pela Lei nº 13.257, de 2016)

    § 3o  Os serviços de saúde onde o parto for realizado assegurarão às mulheres e aos seus filhos recém-nascidos alta hospitalar responsável e contrarreferência na atenção primária, bem como o acesso a outros serviços e a grupos de apoio à amamentação.          (Redação dada pela Lei nº 13.257, de 2016)

            § 4o  Incumbe ao poder público proporcionar assistência psicológica à gestante e à mãe, no período pré e pós-natal, inclusive como forma de prevenir ou minorar as consequências do estado puerperal.       (Incluído pela Lei nº 12.010, de 2009)      Vigência

    § 5o  A assistência referida no § 4o deste artigo deverá ser prestada também a gestantes e mães que manifestem interesse em entregar seus filhos para adoção, bem como a gestantes e mães que se encontrem em situação de privação de liberdade.          (Redação dada pela Lei nº 13.257, de 2016)

    § 6o  A gestante e a parturiente têm direito a 1 (um) acompanhante de sua preferência durante o período do pré-natal, do trabalho de parto e do pós-parto imediato.          (Incluído pela Lei nº 13.257, de 2016)

    § 7o  A gestante deverá receber orientação sobre aleitamento materno, alimentação complementar saudável e crescimento e desenvolvimento infantil, bem como sobre formas de favorecer a criação de vínculos afetivos e de estimular o desenvolvimento integral da criança.          (Incluído pela Lei nº 13.257, de 2016)

    § 8o  A gestante tem direito a acompanhamento saudável durante toda a gestação e a parto natural cuidadoso, estabelecendo-se a aplicação de cesariana e outras intervenções cirúrgicas por motivos médicos.          (Incluído pela Lei nº 13.257, de 2016)

    § 9o  A atenção primária à saúde fará a busca ativa da gestante que não iniciar ou que abandonar as consultas de pré-natal, bem como da puérpera que não comparecer às consultas pós-parto.          (Incluído pela Lei nº 13.257, de 2016)

    § 10.  Incumbe ao poder público garantir, à gestante e à mulher com filho na primeira infância que se encontrem sob custódia em unidade de privação de liberdade, ambiência que atenda às normas sanitárias e assistenciais do Sistema Único de Saúde para o acolhimento do filho, em articulação com o sistema de ensino competente, visando ao desenvolvimento integral da criança.          (Incluído pela Lei nº 13.257, de 2016)

    ___________________________________________________________________________
    IV. O atendimento pré-natal será realizado por profissionais da atenção primária. 

    A assertiva IV está CORRETA, nos termos do artigo 8º, §1º, do ECA (Lei 8.069/90):

    Art. 8o  É assegurado a todas as mulheres o acesso aos programas e às políticas de saúde da mulher e de planejamento reprodutivo e, às gestantes, nutrição adequada, atenção humanizada à gravidez, ao parto e ao puerpério e atendimento pré-natal, perinatal e pós-natal integral no âmbito do Sistema Único de Saúde.          (Redação dada pela Lei nº 13.257, de 2016)

    § 1o  O atendimento pré-natal será realizado por profissionais da atenção primária.          (Redação dada pela Lei nº 13.257, de 2016)

    § 2o  Os profissionais de saúde de referência da gestante garantirão sua vinculação, no último trimestre da gestação, ao estabelecimento em que será realizado o parto, garantido o direito de opção da mulher.          (Redação dada pela Lei nº 13.257, de 2016)

    § 3o  Os serviços de saúde onde o parto for realizado assegurarão às mulheres e aos seus filhos recém-nascidos alta hospitalar responsável e contrarreferência na atenção primária, bem como o acesso a outros serviços e a grupos de apoio à amamentação.          (Redação dada pela Lei nº 13.257, de 2016)

            § 4o  Incumbe ao poder público proporcionar assistência psicológica à gestante e à mãe, no período pré e pós-natal, inclusive como forma de prevenir ou minorar as consequências do estado puerperal.       (Incluído pela Lei nº 12.010, de 2009)      Vigência

    § 5o  A assistência referida no § 4o deste artigo deverá ser prestada também a gestantes e mães que manifestem interesse em entregar seus filhos para adoção, bem como a gestantes e mães que se encontrem em situação de privação de liberdade.          (Redação dada pela Lei nº 13.257, de 2016)

    § 6o  A gestante e a parturiente têm direito a 1 (um) acompanhante de sua preferência durante o período do pré-natal, do trabalho de parto e do pós-parto imediato.          (Incluído pela Lei nº 13.257, de 2016)

    § 7o  A gestante deverá receber orientação sobre aleitamento materno, alimentação complementar saudável e crescimento e desenvolvimento infantil, bem como sobre formas de favorecer a criação de vínculos afetivos e de estimular o desenvolvimento integral da criança.          (Incluído pela Lei nº 13.257, de 2016)

    § 8o  A gestante tem direito a acompanhamento saudável durante toda a gestação e a parto natural cuidadoso, estabelecendo-se a aplicação de cesariana e outras intervenções cirúrgicas por motivos médicos.          (Incluído pela Lei nº 13.257, de 2016)

    § 9o  A atenção primária à saúde fará a busca ativa da gestante que não iniciar ou que abandonar as consultas de pré-natal, bem como da puérpera que não comparecer às consultas pós-parto.          (Incluído pela Lei nº 13.257, de 2016)

    § 10.  Incumbe ao poder público garantir, à gestante e à mulher com filho na primeira infância que se encontrem sob custódia em unidade de privação de liberdade, ambiência que atenda às normas sanitárias e assistenciais do Sistema Único de Saúde para o acolhimento do filho, em articulação com o sistema de ensino competente, visando ao desenvolvimento integral da criança.          (Incluído pela Lei nº 13.257, de 2016)

    ____________________________________________________________________________
    Estando correta apenas a assertiva IV, deve ser assinalada a alternativa B.
    ____________________________________________________________________________
    RESPOSTA: ALTERNATIVA B
  • Sobre os direitos fundamentais à vida e à saúde da mulher gestante previstos no Estatuto da Criança e do Adolescente, leia as assertivas a seguir e, depois, assinale a alternativa CORRETA.

    I. A gestante tem direito a 1 (UM) 02 (dois) acompanhantes de sua preferência durante o período do pré-natal, do trabalho de parto e do pós-parto imediato.

    II. Caso os profissionais de saúde de referência já tenham vinculado a gestante, no último trimestre da gestação, ao estabelecimento em que será realizado o parto, a mulher TEM SIM não tem o direito de optar por outro local. 

    III. A obrigação do poder público de proporcionar assistência psicológica à gestante e à mãe SERÁ NO  se limita ao período pré-natal E PÓS-NATAL.

    IV. O atendimento pré-natal será realizado por profissionais da atenção primária.  

     

    "No pain no gain in the brain also bro"

  • I. A gestante tem direito a 02 (dois) acompanhantes de sua preferência durante o período do pré-natal, do trabalho de parto e do pós-parto imediato. (E) - art 8 §6º a gestante e a parturiente têm direito a 1(um) acompanhante durante o periodo pré natal, do trabalho de parto e do parto imediato

     

    II. Caso os profissionais de saúde de referência já tenham vinculado a gestante, no último trimestre da gestação, ao estabelecimento em que será realizado o parto, a mulher não tem o direito de optar por outro local.(E) art 8°§2°- (...) garantido o direito de opção da mulher

     

    III. A obrigação do poder público de proporcionar assistência psicológica à gestante e à mãe se limita ao período pré-natal.(E) (...) durante o período pré e pós natal

     

    IV. O atendimento pré-natal será realizado por profissionais da atenção primária.  (C) art 8°§1°

     

    alternativa "B"

  • I - um acompanhante - artigo 8, p. 6, ECA.
    II - pode optar - artigo 8, p. 2. 
    III -  e pós-natal - artigo 8, p. 4. 
    IV- si, artigo 8, p. 1. 

  • GABARITO: B

    Art 8 § 1  O atendimento pré-natal será realizado por profissionais da atenção primária.  

  • I. A gestante tem direito a 02 (dois) acompanhantes de sua preferência durante o período do pré-natal, do trabalho de parto e do pós-parto imediato.

    § 6  A gestante e a parturiente têm direito a 1 (um) acompanhante de sua preferência durante o período do pré-natal, do trabalho de parto e do pós-parto imediato. 

    II. Caso os profissionais de saúde de referência já tenham vinculado a gestante, no último trimestre da gestação, ao estabelecimento em que será realizado o parto, a mulher não tem o direito de optar por outro local.

    § 2 Os profissionais de saúde de referência da gestante garantirão sua vinculação, no último trimestre da gestação, ao estabelecimento em que será realizado o parto, garantido o direito de opção da mulher. 

    III. A obrigação do poder público de proporcionar assistência psicológica à gestante e à mãe se limita ao período pré-natal.

    § 4 Incumbe ao poder público proporcionar assistência psicológica à gestante e à mãe, no período pré e pós-natal, inclusive como forma de prevenir ou minorar as consequências do estado puerperal. 

    IV. O atendimento pré-natal será realizado por profissionais da atenção primária.

    § 1 O atendimento pré-natal será realizado por profissionais da atenção primária. 


ID
2478709
Banca
PUC-PR
Órgão
TJ-PR
Ano
2017
Provas
Disciplina
Direito da Criança e do Adolescente - Estatuto da Criança e do Adolescente (ECA) - Lei nº 8.069 de 1990
Assuntos

A Lei 12.594/2012 (Estatuto da Criança e do Adolescente) instituiu o Sistema Nacional de Atendimento Socioeducativo (SINASE), regulamentando as medidas destinadas a adolescente que pratique ato infracional. Sobre o tema, assinale a alternativa CORRETA.

Alternativas
Comentários
  • ALGUÉM EXPLIQUE ESSA QUESTÃO,POR FAVOR.

  • A questão está bem confusa, mas encontrei na Lei do SINASE (Lei 12.594) a resposta:

    Art. 46.  A medida socioeducativa será declarada extinta: 

    I - pela morte do adolescente; 

    II - pela realização de sua finalidade; 

    III - pela aplicação de pena privativa de liberdade, a ser cumprida em regime fechado ou semiaberto, em execução provisória ou definitiva; 

    IV - pela condição de doença grave, que torne o adolescente incapaz de submeter-se ao cumprimento da medida; e 

    V - nas demais hipóteses previstas em lei. 

  • A) SINASE (Lei 12.594), Art. 46. A medida socioeducativa será declarada extinta:
    I – pela morte do adolescente;
    II – pela realização de sua finalidade;
    III – pela aplicação de pena privativa de liberdade, a ser cumprida em regime fechado ou semiaberto, em execução provisória ou definitiva;
    IV – pela condição de doença grave, que torne o adolescente incapaz de submeter-se ao cumprimento da medida; e
    V – nas demais hipóteses previstas em lei.

    **Entendo que, neste caso, a medida socioeducativa é extinta pela situação de o adolescente tornar-se adulto. Ali fala-se em PENA, e não mais em ato infracional. 

    B) "Retirar do convívio social" é contrário ao objetivo principal do Estatuto. Também, sobre as medidas socioeducativas fazerem isso ou "encarcerarem", lembrar que nem toda medida o faz, dado, por exemplo, a simples advertência, a obrigação de reparar o dano, a prestação de serviços à comunidade e a liberdade assistida. 

    C) SINASE, Título II – DA EXECUÇÃO DAS MEDIDAS SOCIOEDUCATIVAS, CAPÍTULO I – DISPOSIÇÕES GERAIS:
    Art. 35. A execução das medidas socioeducativas reger-se-á pelos seguintes princípios:
    I – legalidade, não podendo o adolescente receber tratamento mais gravoso do que o conferido ao adulto;

    D)
    SINASE Art. 49. São direitos do adolescente submetido ao cumprimento de medida socioeducativa, sem prejuízo de outros previstos em lei:
    I – ser acompanhado por seus pais ou responsável e por seu defensor, em qualquer fase do procedimento administrativo ou judicial;
    ECA Art. 111. I – direito de solicitar a presença de seus pais ou responsável em qualquer fase do procedimento.

    E)
    SINASE Art. 35. A execução das medidas socioeducativas reger-se-á pelos seguintes princípios:
    VI – individualização, considerando-se a idade, capacidades e circunstâncias pessoais do adolescente;
    ECA Art. 112
    § 1º A medida aplicada ao adolescente levará em conta a sua capacidade de cumpri-la, as circunstâncias e a gravidade da infração.
    § 3º Os adolescentes portadores de doença ou deficiência mental receberão tratamento individual e especializado, em local adequado às suas condições.

  • Art. 46

    III - pela aplicação de pena privativa de liberdade, a ser cumprida em regime fechado ou semiaberto, em execução provisória ou definitiva; 

  • ACERTEI POR DEDUÇÃO

  • B) As medidas socieducativas têm por principal objetivo retirar o transgressor do convívio social, promovendo o encarceramento imediato do adolescente infrator, por tempo compatível com a gravidade de sua conduta

    A alternativa B está INCORRETA, pois retirar o transgressor do convívio social não é objetivo das medidas socioeducativas previsto na Lei 12.594/2012. Os objetivos das medidas socioeducativas estão previstos no artigo 1º, §2º, da Lei 12.594/2012:

    Art. 1o  Esta Lei institui o Sistema Nacional de Atendimento Socioeducativo (Sinase) e regulamenta a execução das medidas destinadas a adolescente que pratique ato infracional. 

    § 1o  Entende-se por Sinase o conjunto ordenado de princípios, regras e critérios que envolvem a execução de medidas socioeducativas, incluindo-se nele, por adesão, os sistemas estaduais, distrital e municipais, bem como todos os planos, políticas e programas específicos de atendimento a adolescente em conflito com a lei. 

    § 2o  Entendem-se por medidas socioeducativas as previstas no art. 112 da Lei no 8.069, de 13 de julho de 1990 (Estatuto da Criança e do Adolescente), as quais têm por objetivos: 

    I - a responsabilização do adolescente quanto às consequências lesivas do ato infracional, sempre que possível incentivando a sua reparação; 

    II - a integração social do adolescente e a garantia de seus direitos individuais e sociais, por meio do cumprimento de seu plano individual de atendimento; e 

    III - a desaprovação da conduta infracional, efetivando as disposições da sentença como parâmetro máximo de privação de liberdade ou restrição de direitos, observados os limites previstos em lei. 

    § 3o  Entendem-se por programa de atendimento a organização e o funcionamento, por unidade, das condições necessárias para o cumprimento das medidas socioeducativas. 

    § 4o  Entende-se por unidade a base física necessária para a organização e o funcionamento de programa de atendimento. 

    § 5o  Entendem-se por entidade de atendimento a pessoa jurídica de direito público ou privado que instala e mantém a unidade e os recursos humanos e materiais necessários ao desenvolvimento de programas de atendimento.

    __________________________________________________________________________
    C) Se a conduta do adolescente for considerada extremamente irresponsável, a execução das medidas socioeducativas poderá impor tratamento mais gravoso do que o conferido ao adulto que realizou conduta similar.

    A alternativa C está INCORRETA, conforme artigo 35, inciso I, da Lei 12.594/2012:

    Art. 35.  A execução das medidas socioeducativas reger-se-á pelos seguintes princípios: 

    I - legalidade, não podendo o adolescente receber tratamento mais gravoso do que o conferido ao adulto

    II - excepcionalidade da intervenção judicial e da imposição de medidas, favorecendo-se meios de autocomposição de conflitos; 

    III - prioridade a práticas ou medidas que sejam restaurativas e, sempre que possível, atendam às necessidades das vítimas; 

    IV - proporcionalidade em relação à ofensa cometida; 

    V - brevidade da medida em resposta ao ato cometido, em especial o respeito ao que dispõe o art. 122 da Lei no 8.069, de 13 de julho de 1990 (Estatuto da Criança e do Adolescente)

    VI - individualização, considerando-se a idade, capacidades e circunstâncias pessoais do adolescente; 

    VII - mínima intervenção, restrita ao necessário para a realização dos objetivos da medida; 

    VIII - não discriminação do adolescente, notadamente em razão de etnia, gênero, nacionalidade, classe social, orientação religiosa, política ou sexual, ou associação ou pertencimento a qualquer minoria ou status; e 

    IX - fortalecimento dos vínculos familiares e comunitários no processo socioeducativo.

    __________________________________________________________________________
    D) Nos procedimentos administrativos ou judiciais relativos a medidas socioeducativas, o adolescente não poderá ser acompanhado por seus pais ou por responsável, mas apenas por seu defensor.

    A alternativa D está INCORRETA, pois, conforme artigo 49, inciso I, da Lei 12.594/2012, é direito do adolescente ser acompanhado por seus pais ou responsável e por seu defensor, em qualquer fase do procedimento administrativo ou judicial:

    Art. 49.  São direitos do adolescente submetido ao cumprimento de medida socioeducativa, sem prejuízo de outros previstos em lei: 

    I - ser acompanhado por seus pais ou responsável e por seu defensor, em qualquer fase do procedimento administrativo ou judicial

    II - ser incluído em programa de meio aberto quando inexistir vaga para o cumprimento de medida de privação da liberdade, exceto nos casos de ato infracional cometido mediante grave ameaça ou violência à pessoa, quando o adolescente deverá ser internado em Unidade mais próxima de seu local de residência; 

    III - ser respeitado em sua personalidade, intimidade, liberdade de pensamento e religião e em todos os direitos não expressamente limitados na sentença; 

    IV - peticionar, por escrito ou verbalmente, diretamente a qualquer autoridade ou órgão público, devendo, obrigatoriamente, ser respondido em até 15 (quinze) dias; 

    V - ser informado, inclusive por escrito, das normas de organização e funcionamento do programa de atendimento e também das previsões de natureza disciplinar; 

    VI - receber, sempre que solicitar, informações sobre a evolução de seu plano individual, participando, obrigatoriamente, de sua elaboração e, se for o caso, reavaliação; 

    VII - receber assistência integral à sua saúde, conforme o disposto no art. 60 desta Lei; e 

    VIII - ter atendimento garantido em creche e pré-escola aos filhos de 0 (zero) a 5 (cinco) anos. 

    § 1o  As garantias processuais destinadas a adolescente autor de ato infracional previstas na Lei no 8.069, de 13 de julho de 1990 (Estatuto da Criança e do Adolescente), aplicam-se integralmente na execução das medidas socioeducativas, inclusive no âmbito administrativo. 

    § 2o  A oferta irregular de programas de atendimento socioeducativo em meio aberto não poderá ser invocada como motivo para aplicação ou manutenção de medida de privação da liberdade

    __________________________________________________________________________
    E) Pelo princípio da impessoalidade, a execução das medidas socioeducativas não pode levar em consideração a idade, as capacidades ou as circunstâncias pessoais do adolescente, mas tão somente a gravidade de sua conduta.  

    A alternativa E está INCORRETA, pois, nos termos do artigo 35 da Lei 12.594/2012, o princípio da impessoalidade não rege a execução das medidas socioeducativas. Ao contrário: a execução das medidas socioeducativas é regida pelo princípio da individualização, de acordo com o qual devem ser consideradas a idade, capacidades e circunstâncias pessoais do adolescente (inciso VI):

    Art. 35.  A execução das medidas socioeducativas reger-se-á pelos seguintes princípios: 

    I - legalidade, não podendo o adolescente receber tratamento mais gravoso do que o conferido ao adulto; 

    II - excepcionalidade da intervenção judicial e da imposição de medidas, favorecendo-se meios de autocomposição de conflitos; 

    III - prioridade a práticas ou medidas que sejam restaurativas e, sempre que possível, atendam às necessidades das vítimas; 

    IV - proporcionalidade em relação à ofensa cometida; 

    V - brevidade da medida em resposta ao ato cometido, em especial o respeito ao que dispõe o art. 122 da Lei no 8.069, de 13 de julho de 1990 (Estatuto da Criança e do Adolescente)

    VI - individualização, considerando-se a idade, capacidades e circunstâncias pessoais do adolescente

    VII - mínima intervenção, restrita ao necessário para a realização dos objetivos da medida; 

    VIII - não discriminação do adolescente, notadamente em razão de etnia, gênero, nacionalidade, classe social, orientação religiosa, política ou sexual, ou associação ou pertencimento a qualquer minoria ou status; e 

    IX - fortalecimento dos vínculos familiares e comunitários no processo socioeducativo.

    __________________________________________________________________________
    A) Uma das maneiras de extinção de medida socioeducativa se dá pela aplicação de pena privativa de liberdade, a ser cumprida em regime fechado ou semiaberto, em execução provisória ou definitiva.

    A alternativa A está CORRETA, conforme artigo 46, inciso III, da Lei 12.594/2012:

    Art. 46.  A medida socioeducativa será declarada extinta: 

    I - pela morte do adolescente; 

    II - pela realização de sua finalidade; 

    III - pela aplicação de pena privativa de liberdade, a ser cumprida em regime fechado ou semiaberto, em execução provisória ou definitiva

    IV - pela condição de doença grave, que torne o adolescente incapaz de submeter-se ao cumprimento da medida; e 

    V - nas demais hipóteses previstas em lei. 

    § 1o  No caso de o maior de 18 (dezoito) anos, em cumprimento de medida socioeducativa, responder a processo-crime, caberá à autoridade judiciária decidir sobre eventual extinção da execução, cientificando da decisão o juízo criminal competente. 

    § 2o  Em qualquer caso, o tempo de prisão cautelar não convertida em pena privativa de liberdade deve ser descontado do prazo de cumprimento da medida socioeducativa.

    ___________________________________________________________________________
    Resposta: ALTERNATIVA A
  • o ECA pode ser aplicado até os 21 anos, assim se a pessoa fixa maior de 18 anos e por outro fato recebe pena privativa de liberdade ocorre a questão 

  • LEI Nº 12.594/2012

    Art. 46, III - pela aplicação de pena privativa de liberdade, a ser cumprida em regime fechado ou semiaberto, em execução provisória ou definitiva; 

    b) a integração social do adolescente e a garantia de seus direitos (Art. 1º, §2º, inciso II);

    c) não pode o adolescente receber tratamento mais gravoso do que o conferido ao adulto (Art. 35, inciso I);

    d) ser acompanhado é um direito do adolescente (Art. 49, inciso I);

    e) pelo princípio da individualização, considerando-se a idade, capacidades e circunstâncias pessoais do adolescente (Art. 35, inciso VI);

    Quem escolheu a busca não pode recusar a travessia - Guimarães Rosa

    Gabarito: A

  • SINASE

    Art. 46. A medida socioeducativa será declarada extinta:

    I - pela morte do adolescente;

    II - pela realização de sua finalidade;

    III - pela aplicação de pena privativa de liberdade, a ser cumprida em regime fechado ou semiaberto, em execução provisória ou definitiva;

    IV - pela condição de doença grave, que torne o adolescente incapaz de submeter-se ao cumprimento da medida; e

    V - nas demais hipóteses previstas em lei.


ID
2478712
Banca
PUC-PR
Órgão
TJ-PR
Ano
2017
Provas
Disciplina
Direito Processual Penal
Assuntos

Conhecida como Lei Maria da Penha, a Lei 11.340/2006 criou mecanismos para coibir a violência doméstica e familiar contra a mulher. Sobre o tema, assinale a alternativa CORRETA.

Alternativas
Comentários
  • Letra C

     

  • Resposta C

    Art. 9o  A assistência à mulher em situação de violência doméstica e familiar será prestada de forma articulada e conforme os princípios e as diretrizes previstos na Lei Orgânica da Assistência Social, no Sistema Único de Saúde, no Sistema Único de Segurança Pública, entre outras normas e políticas públicas de proteção, e emergencialmente quando for o caso.

    § 2o  O juiz assegurará à mulher em situação de violência doméstica e familiar, para preservar sua integridade física e psicológica:

    II - manutenção do vínculo trabalhista, quando necessário o afastamento do local de trabalho, por até seis meses.

  • Todos arts. da Lei 11.340:

     

    A - Art. 12.  Em todos os casos de violência doméstica e familiar contra a mulher, feito o registro da ocorrência, deverá a autoridade policial adotar, de imediato, os seguintes procedimentos, sem prejuízo daqueles previstos no Código de Processo Penal:

    V - ouvir o agressor e as testemunhas;

     

    B - Art. 5o  Para os efeitos desta Lei, configura violência doméstica e familiar contra a mulher qualquer ação ou omissão baseada no gênero que lhe cause morte, lesão, sofrimento físico, sexual ou psicológico e dano moral ou patrimonial:

     

    C - CORRETA - É hipótese de suspensão do contrato de trabalho (não há pagamento de salários):

    Art. 9o, § 2o  O juiz assegurará à mulher em situação de violência doméstica e familiar, para preservar sua integridade física e psicológica:

    II - manutenção do vínculo trabalhista, quando necessário o afastamento do local de trabalho, por até seis meses.

     

    D - Art. 2o  Toda mulher, independentemente de classe, raça, etnia, orientação sexual, renda, cultura, nível educacional, idade e religião, goza dos direitos fundamentais inerentes à pessoa humana, sendo-lhe asseguradas as oportunidades e facilidades para viver sem violência, preservar sua saúde física e mental e seu aperfeiçoamento moral, intelectual e social.

    Art. 5o, Parágrafo único.  As relações pessoais enunciadas neste artigo independem de orientação sexual.

     

    E - Art. 5o, III - em qualquer relação íntima de afeto, na qual o agressor conviva ou tenha convivido com a ofendida, independentemente de coabitação.

  • manutenção do vínculo trabalhista, quando necessário o afastamento do local de trabalho, por até seis meses.

     

  • Correta, C

    Observações:

    Violência Doméstica contra a mulher, pode ser:

    - Física;
    - Psicológica;
    - Moral;
    - Patrimonial.

    Sujeitos Ativos e Passivos da Lei Maria da Penha:

    - Homem contra Mulher;

    - Mulher contra Mulher;

    - Mãe contra Filha.

    Entretanto, não aplica-se a lei maria da penha na relação homoafetiva entre Homens, visto que o sujeito passivo da Lei Maria da Penha é sempre a Mulher. Vale resaltar que, o que pode acontecer é ser aplicado alguma das medidas protetivas da referida lei nestas relações. 

  • DA ASSISTÊNCIA À MULHER EM SITUAÇÃO DE VIOLÊNCIA DOMÉSTICA E FAMILIAR

    Art. 9o  A assistência à mulher em situação de violência doméstica e familiar será prestada de forma articulada e conforme os princípios e as diretrizes previstos na Lei Orgânica da Assistência Social, no Sistema Único de Saúde, no Sistema Único de Segurança Pública, entre outras normas e políticas públicas de proteção, e emergencialmente quando for o caso.

    § 1o  O juiz determinará, por prazo certo, a inclusão da mulher em situação de violência doméstica e familiar no cadastro de programas assistenciais do governo federal, estadual e municipal.

    § 2o  O juiz assegurará à mulher em situação de violência doméstica e familiar, para preservar sua integridade física e psicológica:

    I - acesso prioritário à remoção quando servidora pública, integrante da administração direta ou indireta;

    II - manutenção do vínculo trabalhista, quando necessário o afastamento do local de trabalho, por até seis meses.

    § 3o  A assistência à mulher em situação de violência doméstica e familiar compreenderá o acesso aos benefícios decorrentes do desenvolvimento científico e tecnológico, incluindo os serviços de contracepção de emergência, a profilaxia das Doenças Sexualmente Transmissíveis (DST) e da Síndrome da Imunodeficiência Adquirida (AIDS) e outros procedimentos médicos necessários e cabíveis nos casos de violência sexual.

  • A) Para evitar represálias, em casos de violência doméstica e familiar contra a mulher, feito o registro da ocorrência, a autoridade policial está expressamente proibida de ouvir o agressor e as testemunhas.  

    A alternativa A está INCORRETA, pois, conforme artigo 12, inciso V, da Lei 11.340/2006, a autoridade policial DEVERÁ ouvir o agressor e as testemunhas:

    Art. 12.  Em todos os casos de violência doméstica e familiar contra a mulher, feito o registro da ocorrência, deverá a autoridade policial adotar, de imediato, os seguintes procedimentos, sem prejuízo daqueles previstos no Código de Processo Penal:

    I - ouvir a ofendida, lavrar o boletim de ocorrência e tomar a representação a termo, se apresentada;

    II - colher todas as provas que servirem para o esclarecimento do fato e de suas circunstâncias;

    III - remeter, no prazo de 48 (quarenta e oito) horas, expediente apartado ao juiz com o pedido da ofendida, para a concessão de medidas protetivas de urgência;

    IV - determinar que se proceda ao exame de corpo de delito da ofendida e requisitar outros exames periciais necessários;

    V - ouvir o agressor e as testemunhas;

    VI - ordenar a identificação do agressor e fazer juntar aos autos sua folha de antecedentes criminais, indicando a existência de mandado de prisão ou registro de outras ocorrências policiais contra ele;

    VII - remeter, no prazo legal, os autos do inquérito policial ao juiz e ao Ministério Público.

    § 1o  O pedido da ofendida será tomado a termo pela autoridade policial e deverá conter:

    I - qualificação da ofendida e do agressor;

    II - nome e idade dos dependentes;

    III - descrição sucinta do fato e das medidas protetivas solicitadas pela ofendida.

    § 2o  A autoridade policial deverá anexar ao documento referido no § 1o o boletim de ocorrência e cópia de todos os documentos disponíveis em posse da ofendida.

    § 3o  Serão admitidos como meios de prova os laudos ou prontuários médicos fornecidos por hospitais e postos de saúde.

    _______________________________________________________________________________
    B) A violência doméstica prevista na Lei Maria da Penha é unicamente a violência física, na qual o homem faz uso da força para subjugar a esposa.

    A alternativa B está INCORRETA, pois, nos termos do artigo 5º, "caput", da Lei 11.340/2006, a violência doméstica prevista na Lei Maria da Penha NÃO é unicamente a violência física, mas também a violência sexual ou psicológica e o dano moral ou patrimonial:

    Art. 5o  Para os efeitos desta Lei, configura violência doméstica e familiar contra a mulher qualquer ação ou omissão baseada no gênero que lhe cause morte, lesão, sofrimento físico, sexual ou psicológico e dano moral ou patrimonial: (Vide Lei complementar nº 150, de 2015)

    I - no âmbito da unidade doméstica, compreendida como o espaço de convívio permanente de pessoas, com ou sem vínculo familiar, inclusive as esporadicamente agregadas;

    II - no âmbito da família, compreendida como a comunidade formada por indivíduos que são ou se consideram aparentados, unidos por laços naturais, por afinidade ou por vontade expressa;

    III - em qualquer relação íntima de afeto, na qual o agressor conviva ou tenha convivido com a ofendida, independentemente de coabitação.

    Parágrafo único.  As relações pessoais enunciadas neste artigo independem de orientação sexual.

    _______________________________________________________________________________
    D) As relações pessoais que podem configurar atos de violência doméstica são necessariamente aquelas derivadas da relação entre homem e mulher, não se podendo aplicá-las a eventuais relações homoafetivas entre duas mulheres.  

    A alternativa D está INCORRETA, conforme vem decidindo a jurisprudência:

    RECURSO EM SENTIDO ESTRITO. VIOLÊNCIA DOMÉSTICA. UNIÃO HOMOAFETIVA. POSSIBILIDADE. ANÁLISE DO CASO CONCRETO. RELAÇÃO ÍNTIMA PRETÉRITA. AUSÊNCIA DE PERTINÊNCIA. MOTIVAÇÃO DESVINCULADA DO GÊNERO. INEXISTÊNCIA DE CONDIÇÃO DE HIPOSSUFICIÊNCIA OU VULNERABILIDADE. INAPLICABILIDADE DA LEI MARIA DA PENHA. DECISÃO MANTIDA. RECURSO DESPROVIDO.
    1. Os dispositivos da Lei Maria da Penha são também aplicáveis às mulheres que, no uso de sua liberdade sexual, mantêm relacionamentos homoafetivos. É dizer: a lei não desampara a mulher pelo fato de sua relação íntima estabelecer-se com pessoa do mesmo sexo, sendo certo que conclusão diversa seria absolutamente inconstitucional.
    2. A Lei Maria da Penha define com clareza o sujeito passivo da violência doméstica, que será sempre a mulher. Contudo, o sujeito ativo poderá ser tanto o homem quanto a mulher, devendo a análise do caso concreto atentar-se à existência ou não de motivação de gênero e utilização da relação doméstica, familiar ou de afetividade como escopo para a prática da violência, fatores que serão determinantes para concluir-se pela (in)aplicabilidade da referida norma.
    3. Não se verifica a permanência de qualquer vínculo íntimo entre a ofendida e a recorrida: o transcurso de significativo lapso temporal entre o término do relacionamento (2008) e a data da suposta ameaça (2013), bem como a prova da existência de sério relacionamento afetivo posterior, obstam eventual presunção de que a violência tenha sido decorrente da relação de afeto mantida, no passado, entre a vítima e a agressora.
    4. A motivação da suposta ameaça teria sido um desentendimento entre agressora e ofendida, fundado no receio por parte da agressora de que a vítima estivesse colaborando para que os credores encontrassem o seu endereço, motivo que não guarda qualquer pertinência com a relação homoafetiva mantida e encerrada anos atrás.
    5. Não se constata que a vítima estivesse em condição de vulnerabilidade ou hipossuficiência frente à recorrida.

    6. Recurso desprovido.
    (TJDFT - Acórdão n.777193, 20130710404924RSE, Relator: SILVÂNIO BARBOSA DOS SANTOS 2ª Turma Criminal, Data de Julgamento: 03/04/2014, Publicado no DJE: 09/04/2014. Pág.: 386)

    ________________________________________________________________________________
    E) A configuração de atos de violência doméstica depende necessariamente de haver coabitação entre cônjuges.  

    A alternativa E está INCORRETA, pois, nos termos do artigo 5º, inciso III, parte final, da Lei 11.340/2006, a configuração de atos de violência doméstica NÃO depende necessariamente de haver coabitação entre cônjuges:

    Art. 5o  Para os efeitos desta Lei, configura violência doméstica e familiar contra a mulher qualquer ação ou omissão baseada no gênero que lhe cause morte, lesão, sofrimento físico, sexual ou psicológico e dano moral ou patrimonial: (Vide Lei complementar nº 150, de 2015)

    I - no âmbito da unidade doméstica, compreendida como o espaço de convívio permanente de pessoas, com ou sem vínculo familiar, inclusive as esporadicamente agregadas;

    II - no âmbito da família, compreendida como a comunidade formada por indivíduos que são ou se consideram aparentados, unidos por laços naturais, por afinidade ou por vontade expressa;

    III - em qualquer relação íntima de afeto, na qual o agressor conviva ou tenha convivido com a ofendida, independentemente de coabitação.

    Parágrafo único.  As relações pessoais enunciadas neste artigo independem de orientação sexual.

    ______________________________________________________________________________
    C) Para preservar a integridade física e psicológica da mulher em situação de violência doméstica, o juiz poderá assegurar, quando necessário, o afastamento da mulher do local de trabalho, por até seis meses.

    A alternativa C está CORRETA, conforme artigo 9º, §2º, inciso II, da Lei 11.340/2006:

    Art. 9o  A assistência à mulher em situação de violência doméstica e familiar será prestada de forma articulada e conforme os princípios e as diretrizes previstos na Lei Orgânica da Assistência Social, no Sistema Único de Saúde, no Sistema Único de Segurança Pública, entre outras normas e políticas públicas de proteção, e emergencialmente quando for o caso.

    § 1o  O juiz determinará, por prazo certo, a inclusão da mulher em situação de violência doméstica e familiar no cadastro de programas assistenciais do governo federal, estadual e municipal.

    § 2o  O juiz assegurará à mulher em situação de violência doméstica e familiar, para preservar sua integridade física e psicológica:

    I - acesso prioritário à remoção quando servidora pública, integrante da administração direta ou indireta;

    II - manutenção do vínculo trabalhista, quando necessário o afastamento do local de trabalho, por até seis meses.

    § 3o  A assistência à mulher em situação de violência doméstica e familiar compreenderá o acesso aos benefícios decorrentes do desenvolvimento científico e tecnológico, incluindo os serviços de contracepção de emergência, a profilaxia das Doenças Sexualmente Transmissíveis (DST) e da Síndrome da Imunodeficiência Adquirida (AIDS) e outros procedimentos médicos necessários e cabíveis nos casos de violência sexual.
    _______________________________________________________________________________
    Resposta: ALTERNATIVA C
  • GABARITO: C - Art. 9°, § 2°, II - manutenção do vínculo trabalhista, quando necessário o afastamento do local de trabalho, por até seis meses.

    A) ERRADA - Art. 12.  Em todos os casos de violência doméstica e familiar contra a mulher, feito o registro da ocorrência, deverá a autoridade policial adotar, de imediato, os seguintes procedimentos, sem prejuízo daqueles previstos no Código de Processo Penal:

    V - ouvir o agressor e as testemunhas;

    B) ERRADA -  Art. 7°, I - violência física; II - Psicológica; III - Sexual; IV - Patrimonial; V - Moral

    D) ERRADA - Art. 5°, III, Parágrafo único - As relações pessoais enunciadas neste artigo independem de orientação sexual.

    E) ERRADA - Art. 5°, III - em qualquer relação íntima de afeto, na qual o agressor conviva ou tenha convivido com a ofendida, independentemente de coabitação.

     

     

    Fonte: LEI Nº 11.340, DE 7 DE AGOSTO DE 2006

     

    Bons estudos!

     

  • a) Para evitar represálias, em casos de violência doméstica e familiar contra a mulher, feito o registro da ocorrência, a autoridade policial está expressamente proibida de ouvir o agressor e as testemunhas.  

     

    b) A violência doméstica prevista na Lei Maria da Penha é unicamente a violência física, na qual o homem faz uso da força para subjugar a esposa.

     

    c) Para preservar a integridade física e psicológica da mulher em situação de violência doméstica, o juiz poderá assegurar, quando necessário, o afastamento da mulher do local de trabalho, por até seis meses. 

     

    d) As relações pessoais que podem configurar atos de violência doméstica são necessariamente aquelas derivadas da relação entre homem e mulher, não se podendo aplicá-las a eventuais relações homoafetivas entre duas mulheres.

     

    e) A configuração de atos de violência doméstica depende necessariamente de haver coabitação entre cônjuges.  

     

  • Art. 9º, § 2º, II, Lei 11340/2006

    FÉ, FORÇA, FOCO E FACA NA CAVEIRA.

     

  • Algumas questões estão considerando transsexuais como sujeito passivo da lei. Muito cuidado, pois há alguns julgados em favor disso.

  • Capitulo II da Lei 11340/06

    § 2o  O juiz assegurará à mulher em situação de violência doméstica e familiar, para preservar sua integridade física e psicológica:

    I - acesso prioritário à remoção quando servidora pública, integrante da administração direta ou indireta;

    II - manutenção do vínculo trabalhista, quando necessário o afastamento do local de trabalho, por até seis meses.

  • Sobre a assertiva "E": 

    A Súmula 600/STJ:“ Para configuração da violência doméstica e familiar prevista no artigo 5º da Lei 11.340/2006, Lei Maria da Penha, não se exige a coabitação entre autor e vítima”.

  • MANUTENÇÃO DO VÍNCULO TRABALHISTA QUANDO NECESSÁRIO O AFASTAMENTO DO LOCAL DE TRABALHO , POR ATÉ 6 MESES.

    FORÇA É HORA!!!

    SERTÃO BRASIL.xx

  •                                                                CAPÍTULO II

    DA ASSISTÊNCIA Á MULHER EM SITUAÇÃO DE VIOLÊNCIA DOMÉSTICA E FAMILIAR 

    II - manutenção do vínculo trabalhista, quando necessário o afastamento do local de trabalho, por até SEIS MESES.

     

    Vá e Vença !!!!!!!!!

     

     

     

  • A assertiva C se refere a lei trabalhista, onde ficara até seis meses sem remuneração. OBS. NÃO A SERVIDOR. 

  • Gab C

     

    Art. 9o  A assistência à mulher em situação de violência doméstica e familiar será prestada de forma articulada e conforme os princípios e as diretrizes previstos na Lei Orgânica da Assistência Social, no Sistema Único de Saúde, no Sistema Único de Segurança Pública, entre outras normas e políticas públicas de proteção, e emergencialmente quando for o caso.

    § 1o  O juiz determinará, por prazo certo, a inclusão da mulher em situação de violência doméstica e familiar no cadastro de programas assistenciais do governo federal, estadual e municipal.

    § 2o  O juiz assegurará à mulher em situação de violência doméstica e familiar, para preservar sua integridade física e psicológica:

    I - acesso prioritário à remoção quando servidora pública, integrante da administração direta ou indireta;

    II - manutenção do vínculo trabalhista, quando necessário o afastamento do local de trabalho, por até seis meses

  • Importante ressaltar que essa possibilidade de manutenção dos vínculos trabalhistas, em caso de necessidade de afastamento do trabalho, NÃO configura caso de medida protetiva de urgência a ser tomada pelo juiz. Essa questão foi cobrada em concurso recente!

  • LEI Nº 11.340/2006

     

    Art. 9º, §2º – O juiz assegurará à mulher em situação de violência doméstica e familiar, para preservar sua integridade física e psicológica:

    II - manutenção do vínculo trabalhista, quando necessário o afastamento do local de trabalho, por até seis meses;

     

    a) em todos os casos a autoridade policial deverá ouvir o agressor e as testemunhas (Art. 12, inciso V);

    b) são formas de violência previstas na referida Lei a sexual, moral, patrominial, física e psicológica (Art. 7º);

    d)  as relações pessoais independem de orientação sexual (Art. 5º, § único);

    e) independentemente de coabitação (Art. 5º, inciso III);

     

    Quem escolheu a busca não pode recusar a travessia - Guimarães Rosa

    ------------------- 

    Gabarito: C

  • § 2º O juiz assegurará à mulher em situação de violência doméstica e familiar, para preservar sua integridade física e psicológica:

    I - acesso prioritário à remoção quando servidora pública, integrante da administração direta ou indireta;

    II - manutenção do vínculo trabalhista, quando necessário o afastamento do local de trabalho, por até seis meses.

     III - encaminhamento à assistência judiciária, quando for o caso, inclusive para eventual ajuizamento da ação de separação judicial, de divórcio, de anulação de casamento ou de dissolução de união estável perante o juízo competente.  

  • Vale ressaltar que a manutenção do vinculo trabalhista não configura medida protetiva de urgência.


ID
2478715
Banca
PUC-PR
Órgão
TJ-PR
Ano
2017
Provas
Disciplina
Direitos Humanos
Assuntos

Sobre a Lei 10.216/2001, que dispõe sobre a proteção e os direitos das pessoas portadoras de transtornos mentais e redireciona o modelo assistencial em saúde mental, leia as assertivas a seguir e, depois, assinale a alternativa CORRETA.

I. A internação da pessoa com transtorno mental, em qualquer de suas modalidades, deverá ser indicada independentemente da utilização prévia de recursos extra-hospitalares.

II. Os direitos e a proteção das pessoas acometidas de transtorno mental são assegurados apenas àqueles considerados de baixa renda ou economicamente frágeis, na acepção da lei.

III. A internação psiquiátrica somente será realizada mediante laudo médico circunstanciado que caracterize os seus motivos.

IV. O término da internação voluntária dar-se-á por solicitação escrita do paciente ou por determinação do médico assistente.

Alternativas
Comentários
  • GABARITO: D III e IV corretas

     

    LEI nº 10.216/2001

     

    I. A internação da pessoa com transtorno mental, em qualquer de suas modalidades, deverá ser indicada independentemente da utilização prévia de recursos extra-hospitalares. Errada.

    Art. 4o A internação, em qualquer de suas modalidades, só será indicada quando os recursos extra-hospitalares se mostrarem insuficientes.

     

     

    II. Os direitos e a proteção das pessoas acometidas de transtorno mental são assegurados apenas àqueles considerados de baixa renda ou economicamente frágeis, na acepção da lei. Errada.

    Art. 1o Os direitos e a proteção das pessoas acometidas de transtorno mental, de que trata esta Lei, são assegurados sem qualquer forma de discriminação quanto à raça, cor, sexo, orientação sexual, religião, opção política, nacionalidade, idade, família, recursos econômicos e ao grau de gravidade ou tempo de evolução de seu transtorno, ou qualquer outra.

     

     

    III. A internação psiquiátrica somente será realizada mediante laudo médico circunstanciado que caracterize os seus motivos. Correta

    Art. 6o A internação psiquiátrica somente será realizada mediante laudo médico circunstanciado que caracterize os seus motivos.


     

    IV. O término da internação voluntária dar-se-á por solicitação escrita do paciente ou por determinação do médico assistente. Correta

    Art. 7o Parágrafo único. O término da internação voluntária dar-se-á por solicitação escrita do paciente ou por determinação do médico assistente.

  • A alternativa I está errada porque o art. 4º da Lei n. 10.216/10 indica que a internação somente poderá ser indicada quando os recursos extra-hospitalares se mostrarem insuficientes - ou seja, ela é uma medida excepcional e que será utilizada como o "último recurso", quando outras medidas não se mostrarem suficientes para assegurar um tratamento adequado ao paciente.
    A alternativa II também está errada, pois o art. 1º da Lei n. 10.216/10 indica que os direitos e proteções serão assegurados a todas as pessoas acometidas por transtornos mentais, sem qualquer forma de discriminação.
    A alternativa III, por sua vez, está correta, pois transcreve o caput do art. 6º da Lei n. 10.216/10, que determina que "a internação psiquiátrica somente será realizada mediante laudo médico circunstanciado que caracterize os seus motivos".
    Por fim, a alternativa IV também está correta, pois o art. 7º, par. único prevê que, em se tratando de internação voluntária, o seu término pode se dar por solicitação escrita do paciente ou por determinação do médico assistente. 


    A resposta correta é a letra D - apenas as assertivas III e IV estão corretas. 
  • I e II é um paradoxo a própria proteção dos Direitos Humanos!

  • Art. 6o A internação psiquiátrica somente será realizada mediante laudo médico circunstanciado que caracterize os seus motivos.

    Parágrafo único. São considerados os seguintes tipos de internação psiquiátrica:

    I - internação voluntária: aquela que se dá com o consentimento do usuário;

    II - internação involuntária: aquela que se dá sem o consentimento do usuário e a pedido de terceiro; e

    III - internação compulsória: aquela determinada pela Justiça.


ID
2478718
Banca
PUC-PR
Órgão
TJ-PR
Ano
2017
Provas
Disciplina
Direito da Criança e do Adolescente - Estatuto da Criança e do Adolescente (ECA) - Lei nº 8.069 de 1990
Assuntos

Acerca dos dispositivos legais previstos na Lei da Primeira Infância (Lei 13.257/2016), leia as assertivas a seguir e, depois, assinale a alternativa CORRETA.

I. As políticas para a primeira infância serão articuladas com as instituições de formação profissional, visando à adequação dos cursos às características e necessidades das crianças e à formação de profissionais qualificados, para possibilitar a expansão com qualidade dos diversos serviços.

II. Como a lei da primeira infância é destinada a crianças de até seis anos de idade, não é possível reconhecê-las como produtoras ativas de cultura.

III. Por serem consideradas juridicamente incapazes, as crianças não podem participar ativamente na formulação das políticas públicas de inclusão social.

IV. As políticas públicas terão, necessariamente, componentes de monitoramento e coleta sistemática de dados, avaliação periódica dos elementos que constituem a oferta dos serviços à criança e divulgação dos seus resultados.

Alternativas
Comentários
  • De acordo com a lei nº 13.257, de 8 de março de 2016:

     

    I)Art. 9. As políticas para a primeira infância serão articuladas com as instituições de formação profissional, visando
    à adequação dos cursos às características e necessidades das crianças e à formação de profissionais qualificados, para
    possibilitar a expansão com qualidade dos diversos serviços.

     

    II)Art. 15. As políticas públicas criarão condições e meios para que, desde a primeira infância, a criança tenha
    acesso à produção cultural e seja reconhecida como produtora de cultura.

     

    III)Art.4.Parágrafo único. A participação da criança na formulação das políticas e das ações que lhe dizem respeito tem o
    objetivo de promover sua inclusão social como cidadã e dar-se-á de acordo com a especificidade de sua idade, devendo
    ser realizada por profissionais qualificados em processos de escuta adequados às diferentes formas de expressão
    infantil.

     

    IV) Art. 11. As políticas públicas terão, necessariamente, componentes de monitoramento e coleta sistemática de
    dados, avaliação periódica dos elementos que constituem a oferta dos serviços à criança e divulgação dos seus
    resultados.

     

    Gabarito Letra E

  • kkkkk formação profissional pra crianças de 6 anos.

  • Nivel: Fácil

  • gostei

  • A questão exige conhecimento acerca da Lei n. 13.257/2016 (Lei da Primeira Infância) e pede ao candidato que julgue os itens a seguir. Vejamos:

    I. As políticas para a primeira infância serão articuladas com as instituições de formação profissional, visando à adequação dos cursos às características e necessidades das crianças e à formação de profissionais qualificados, para possibilitar a expansão com qualidade dos diversos serviços.

    Correto. Trata-se de cópia literal do art. 9º, da Lei da Primeira Infância: Art. 9º As políticas para a primeira infância serão articuladas com as instituições de formação profissional, visando à adequação dos cursos às características e necessidades das crianças e à formação de profissionais qualificados, para possibilitar a expansão com qualidade dos diversos serviços.

    II. Como a lei da primeira infância é destinada a crianças de até seis anos de idade, não é possível reconhecê-las como produtoras ativas de cultura.

    Errado. As crianças são, sim, produtoras de cultura, nos termos do art. 15 da Lei em análise: Art. 15. As políticas públicas criarão condições e meios para que, desde a primeira infância, a criança tenha acesso à produção cultural e seja reconhecida como produtora de cultura.

    III. Por serem consideradas juridicamente incapazes, as crianças não podem participar ativamente na formulação das políticas públicas de inclusão social.

    Errado. De fato, as crianças são juridicamente incapazes, todavia, podem participar, sim, na formulação das políticas públicas de inclusão social, nos termos do art. 4º, II, da Lei n. 13.257/2016: Art. 4º As políticas públicas voltadas ao atendimento dos direitos da criança na primeira infância serão elaboradas e executadas de forma a: II - incluir a participação da criança na definição das ações que lhe digam respeito, em conformidade com suas características etárias e de desenvolvimento;

    IV. As políticas públicas terão, necessariamente, componentes de monitoramento e coleta sistemática de dados, avaliação periódica dos elementos que constituem a oferta dos serviços à criança e divulgação dos seus resultados.

    Correto. Inteligência do art. 11 da Lei n. 13.257/2016: Art. 11. As políticas públicas terão, necessariamente, componentes de monitoramento e coleta sistemática de dados, avaliação periódica dos elementos que constituem a oferta dos serviços à criança e divulgação dos seus resultados.

    Portanto, apenas os itens I e IV estão corretos.

    Gabarito: E


ID
2478721
Banca
PUC-PR
Órgão
TJ-PR
Ano
2017
Provas
Disciplina
Legislação dos Tribunais de Justiça (TJs)
Assuntos

Segundo o Regimento Interno do Tribunal de Justiça do Paraná, responde pelo poder de polícia do Tribunal, como regra geral

Alternativas
Comentários
  • Art. 23. O Presidente responde pelo poder de polícia do Tribunal, podendo requisitar o auxílio de outras autoridades, quando necessário.

  • complementando...

    art. 23, § único: O poder de polícia nas sessões e nas audiências compete a quem presidi-las.

  • Letra B

    CAPÍTULO V DO PODER DE POLÍCIA DO TRIBUNAL

    Art. 23. O Presidente responde pelo poder de polícia do Tribunal, podendo requisitar o auxílio de outras autoridades, quando necessário. 

    Parágrafo único. O poder de polícia nas sessões e nas audiências compete a quem presidi-las.

  • RI TJ-PR: Art. 23. O Presidente responde pelo poder de polícia do Tribunal, podendo requisitar o auxílio de outras autoridades, quando necessário. 

    GABARITO B. 

  •  

      Art. 23. O Presidente responde pelo poder de polícia
    do Tribunal, podendo requisitar o auxílio de outras
    autoridades, quando necessário.
    Parágrafo único. O poder de polícia nas sessões e nas
    audiências compete a quem presidi-las.

    R: B
     

  • CAPÍTULO V

    DO PODER DE POLÍCIA DO TRIBUNAL

    Art. 23. O Presidente responde pelo poder de polícia do Tribunal, podendo requisitar o auxílio de outras autoridades, quando necessário. 

    Parágrafo único. O poder de polícia nas sessões e nas audiências compete a quem presidi-las.

  • A questão exigiu conhecimentos sobre o exercício do poder de polícia do Tribunal, que nos termos do art. 23 do Regimento Interno é exercido pelo Presidente.

     

    Vejamos:

     

    Art. 23. O Presidente responde pelo poder de polícia do Tribunal, podendo requisitar o auxílio de outras autoridades, quando necessário.

     

    Logo, o gabarito é a alternativa B.


ID
2478724
Banca
PUC-PR
Órgão
TJ-PR
Ano
2017
Provas
Disciplina
Legislação Federal
Assuntos

A Política Nacional Antidrogas, introduzida pelo Decreto 4.345/2002, estabelece objetivos e diretrizes para o desenvolvimento de estratégias na prevenção, tratamento, recuperação e reinserção social, redução de danos sociais e à saúde, repressão ao tráfico e estudos, pesquisas e avaliações decorrentes do uso indevido de drogas. NÃO está em consonância com os seus pressupostos básicos

Alternativas
Comentários
  •         2.3.  Evitar a discriminação de indivíduos pelo fato de serem usuários ou dependentes de drogas. (A)

            2.6.  Priorizar a prevenção do uso indevido de drogas, por ser a intervenção mais eficaz e de menor custo para a sociedade. (B)

            2.8.  Reconhecer a "lavagem de dinheiro" como a principal vulnerabilidade a ser alvo das ações repressivas, visando ao desmantelamento do crime organizado, em particular do relacionado com as drogas. (D)

            2.16.  Experimentar de forma pragmática e sem preconceitos novos meios de reduzir danos, com fundamento em resultados científicos comprovados. (E)

     

            2.2.  Reconhecer as diferenças entre o usuário, a pessoa em uso indevido, o dependente e o traficante de drogas, tratando-os de forma diferenciada. (C)

  • Para aqueles que estão resolvendo 1uestão não é sobre a Lei de Drogas, reparem que a questão foi classificada erroneamente, pois na verdade o questionamento é sobre o Decreto 4.345/2002. 

  • Punir vai de encontro aos objetivos estratégicos de uma política pública. Pela lógica dá pra responder.

  • Uma das principais novidades introduzidas pela Lei n° 11.343/06 diz respeito à mudança da política criminal em relação ao usuário de drogas. Se, à época da vigência do art. 16 da Lei n° 6.368/76, o usuário de drogas estava sujeito a uma pena de detenção, de 6 (seis) meses a 2 (dois) anos, e pagamento de 20 (vinte) a 50 (cinquenta) dias-multa, com o advento da Lei n° 11.343/06, o preceito secundário do art. 28 passou a cominar as seguintes penas: advertência sobre os efeitos das drogas, prestação de serviços à comunidade e medida educativa de comparecimento a programa ou curso educativo.

    Logo, afastou a pena privativa de liberdade.

  • DE ENCONTRO - DISCORDA!

    AO ENCONTRO - CONCORDA!

  • PPMG.

    É uma grande oportunidade.

    É hora de revisar, revisar e revisar.

    Pra isso, temos 6 simulados inéditos, baseados na SELECON.

    Corre e fortaleça seus estudos, RUMO A APROVAÇÃO. RUMO A PPMG

    Segue link:

    https://sun.eduzz.com/1082953?a=48670029


ID
2478727
Banca
PUC-PR
Órgão
TJ-PR
Ano
2017
Provas
Disciplina
Legislação dos Tribunais de Justiça (TJs)
Assuntos

De acordo com as disposições do Estatuto dos Servidores Públicos do Poder Judiciário do Estado do Paraná, leis as assertivas a seguir e, depois, assinale a alternativa CORRETA.

I. A jornada de trabalho dos servidores do Poder Judiciário é de 8 (oito) horas diárias e de 40 (quarenta) horas semanais, facultada a fixação de 7 (sete) horas ininterruptas.

II. Licenciado para tratamento de saúde, por acidente no exercício de suas atribuições ou por doença profissional, o funcionário recebe integralmente o vencimento ou a remuneração com as vantagens inerentes ao cargo.

III. Ao entrar em exercício, o funcionário nomeado para cargo de provimento efetivo ficará sujeito a estágio probatório por período máximo de 12 (doze) meses.

IV. O funcionário público estável não poderá perder o cargo em virtude de corte de despesas com pessoal.

Alternativas
Comentários
  • Letra E (I,II - corretas)

  • Art. 40. A jornada de trabalho dos servidores do Poder Judiciário é de 8 (oito) horas diárias e de 40 (quarenta) horas semanais, facultada a fixação de 7 (sete) horas initerruptas.

     

    Art. 113. Licenciado para tratamento de saúde, por acidente no exercício de suas atribuições ou por doença profissional, o funcionário recebe integralmente o vencimento ou a remuneração com as vantagens inerentes ao cargo.

     

    Art. 19. Ao entrar em exercício, o funcionário nomeado para cargo de provimento efetivo ficará sujeito a estágio probatório por período de 36 (trinta e seis) meses [...]

     

    Art. 25. O funcionário estável somente perderá o cargo em virtude de:

    Isentença judicial transitada em julgado;

    IIdecisão em processo administrativo disciplinar;

    IIIdecisão derivada de processo de avaliação periódica de desempenho, na forma da lei complementar federal, assegurada a ampla defesa;

    IVpara corte de despesas com pessoal conforme disposto na Constituição e legislação federal.

  • I. A jornada de trabalho dos servidores do Poder Judiciário é de 8 (oito) horas diárias e de 40 (quarenta) horas semanais, facultada a fixação de 7 (sete) horas ininterruptas.  CERTO

    Art. 40. A jornada de trabalho dos servidores do Poder Judiciário é de 8 (oito) horas diárias e de 40 (quarenta) horas semanais, facultada a fixação de 7 (sete) horas initerruptas.

    II. Licenciado para tratamento de saúde, por acidente no exercício de suas atribuições ou por doença profissional, o funcionário recebe integralmente o vencimento ou a remuneração com as vantagens inerentes ao cargo. CERTO

    Art. 113. Licenciado para tratamento de saúde, por acidente no exercício de suas atribuições ou por doença profissional, o funcionário recebe integralmente o vencimento ou a remuneração com as vantagens inerentes ao cargo.

    III. Ao entrar em exercício, o funcionário nomeado para cargo de provimento efetivo ficará sujeito a estágio probatório por período máximo de 12 (doze) meses. ERRADO

    Art. 19. Ao entrar em exercício, o funcionário nomeado para cargo de provimento efetivo ficará sujeito a estágio probatório por período de 36 (trinta e seis) meses.

    IV. O funcionário público estável não poderá perder o cargo em virtude de corte de despesas com pessoal.  ERRADO

    Art. 25. O funcionário estável somente perderá o cargo em virtude de:

    IV - para corte de despesas com pessoal conforme disposto na Constituição e legislação federal.

  • I. A jornada de trabalho dos servidores do Poder Judiciário é de 8 (oito) horas diárias e de 40 (quarenta) horas semanais, facultada a fixação de 7 (sete) horas ininterruptas.

    II. Licenciado para tratamento de saúde, por acidente no exercício de suas atribuições ou por doença profissional, o funcionário recebe integralmente o vencimento ou a remuneração com as vantagens inerentes ao cargo.

    III. Ao entrar em exercício, o funcionário nomeado para cargo de provimento efetivo ficará sujeito a estágio probatório por período máximo de 36 meses. (3 ANOS)

    IV. O funcionário público estável poderá perder o cargo em virtude de corte de despesas com pessoal.

    GABARITO E.

  • Obrigada QC por liberar o deslike :D

  • A fim de encontrarmos a alternativa correta, iremos analisar cada uma das assertivas a seguir:

    I. Correta - A jornada de trabalho dos servidores do Poder Judiciário é de 8 (oito) horas diárias e de 40 (quarenta) horas semanais, facultada a fixação de 7 (sete) horas ininterruptas.


    Justamente isso! E o fundamento se encontra no Art. 40 da Lei Estadual nº 16.024/2008. Memorize isso. Sobre a jornada de trabalho, são oito horas por dia e quarenta horas por semana. Não caia na pegadinha quando a questão afirma que são quarenta e quatro horas por semana. Ok? O Estatuto dos Servidores nada tem a ver com a Consolidação das Leis do Trabalho.


    II. Correta - Licenciado para tratamento de saúde, por acidente no exercício de suas atribuições ou por doença profissional, o funcionário recebe integralmente o vencimento ou a remuneração com as vantagens inerentes ao cargo.


    É isso aí! E o fundamento se encontra no Art. 113 da Lei Estadual nº 16.024/2008. Não caia em eventual pegadinha da banca no sentido de que o servidor receberá parcialmente a sua remuneração, quando licenciado por motivo de acidente no exercício de suas atribuições. A remuneração é integral. Ok?


    III. Incorreta - Ao entrar em exercício, o funcionário nomeado para cargo de provimento efetivo ficará sujeito a estágio probatório por período máximo de 12 (doze) meses.


    O Art. 19 da Lei Estadual nº 16.024/2008 afirma que “Ao entrar em exercício, (...) estágio probatório por período de 36 (trinta e seis) meses, durante o qual a sua aptidão e capacidade serão objetos de avaliação para o desempenho do cargo (...). Pessoal, essa é clássica nas provas. O tempo de estágio probatório é o mesmo em que o servidor se torna estável: trinta e seis meses.


    IV. Incorreta - O funcionário público estável não poderá perder o cargo em virtude de corte de despesas com pessoal.  


    O Art. 25 da Lei Estadual nº 16.024/2008 afirma que “O funcionário estável somente perderá o cargo em virtude de:  I - sentença judicial transitada em julgado;  II - decisão em processo administrativo disciplinar;  III - decisão derivada de processo de avaliação periódica de desempenho, na forma da lei complementar federal, assegurada a ampla defesa; IV - para corte de despesas com pessoal conforme disposto na Constituição e legislação federal ". Galera, memorize esse dispositivo. Ele cai muito em provas.


    Sendo assim, apenas as assertivas I e II estão corretas. 



    Resposta: E


ID
2478730
Banca
PUC-PR
Órgão
TJ-PR
Ano
2017
Provas
Disciplina
Direito Processual Civil - Novo Código de Processo Civil - CPC 2015
Assuntos

Sobre os conceitos gerais de Conciliação, de Mediação de Conflitos e de Justiça Restaurativa, assinale a alternativa CORRETA.

Alternativas
Comentários
  • GABARITO: D

     

    A)  Art. 3o, § 3o A conciliação, a mediação e outros métodos de solução consensual de conflitos deverão ser estimulados por juízes, advogados, defensores públicos e membros do Ministério Público, inclusive no curso do processo judicial [processo judicial, é genero do qual processo civil é espécie]. 

     

    B)  Art. 3o, § 3o A conciliação, a mediação e outros métodos de solução consensual de conflitos deverão ser estimulados por juízes, advogados, defensores públicos e membros do Ministério Público, inclusive no curso do processo judicial. 

     

    C) A questão descreveu aspectos da JUSTIÇA RETRIBUTIVA, onde a concentração do foco punitivo volta-se ao infrator e ha o predomínio de penas privativas de liberdade. 

     

    D) Art. 165, § 3o O mediador, que atuará preferencialmente nos casos em que houver vínculo anterior entre as partes, auxiliará aos interessados a compreender as questões e os interesses em conflito, de modo que eles possam, pelo restabelecimento da comunicação, identificar, por si próprios, soluções consensuais que gerem benefícios mútuos.

     

    E) A justiça restaurativa não é exclusiva a crimes considerados leves, sendo podendo está, a principio, ser aplicada a qualquer delito. 

  • O que significa Justiça Restaurativa?
    Costumo dizer que Justiça Restaurativa é uma prática que está buscando um conceito. Em linhas gerais poderíamos dizer que se trata de um processo colaborativo voltado para resolução de um conflito caracterizado como crime, que envolve a participação maior do infrator e da vítima. Surgiu no exterior, na cultura anglo-saxã. As primeiras experiências vieram do Canadá e da Nova Zelândia e ganharam relevância em várias partes do mundo. Aqui no Brasil ainda estamos em caráter experimental, mas já está em prática há dez anos. Na prática existem algumas metodologias voltadas para esse processo. A mediação vítima-ofensor consiste basicamente em colocá-los em um mesmo ambiente guardado de segurança jurídica e física, com o objetivo de que se busque ali acordo que implique a resolução de outras dimensões do problema que não apenas a punição, como, por exemplo, a reparação de danos emocionais

    Quem realiza a Justiça Restaurativa?
    Não é o juiz que realiza a prática, e sim o mediador que faz o encontro entre vítima e ofensor e eventualmente as pessoas que as apoiam. Apoiar o ofensor não significa apoiar o crime, e sim apoiá-lo no plano de reparação de danos. Nesse ambiente se faz a busca de uma solução que seja aceitável. Não necessariamente o mediador precisa ter formação jurídica, pode ser por exemplo uma assistente social.

    A Justiça Restaurativa só pode ser aplicada em crimes considerados mais leves?
    Não, pode também ser aplicada aos mais graves. No Brasil temos trabalhado ainda, na maioria das vezes, com os crimes mais leves, porque ainda não temos estrutura apropriada para os crimes mais graves. Em outros países até preferem os crimes mais graves, porque os resultados são mais bem percebidos. A diversidade de crimes e de possibilidades a serem encontradas para sua resolução é muito grande. Vamos supor que, após um sequestro relâmpago, a vítima costuma desenvolver um temor a partir daquele episódio, associando seu agressor a todos que se pareçam com ele, criando um “fantasma” em sua vida, um estereótipo. Independentemente do processo judicial contra o criminoso, como se retoma a segurança emocional dessa pessoa que foi vítima? Provavelmente se o ofensor tiver a oportunidade de dizer, por exemplo, porque a vítima foi escolhida, isso pode resolver essa insegurança que ela vai carregar para o resto da vida. 

     

    http://www.cnj.jus.br/noticias/cnj/62272-justica-restaurativa-o-que-e-e-como-funciona

  • Mas a Justiça Restaurativa implica o não cumprimento da pena tradicional?

    Não, as duas coisas podem ser e frequentemente são concomitantes. O mediador não estabelece redução da pena, ele faz o acordo de reparação de danos. Pode ser feito antes do julgamento, mas a Justiça Restaurativa é um conceito muito aberto. Há experiências na fase de cumprimento da pena, na fase de progressão de regime etc. Mas nos crimes de pequeno potencial ofensivo, de acordo com artigo 74 da Lei n. 9.099, de 1995, o acordo pode inclusive excluir o processo legal. Já quando falamos de infrações cometidas pelo público infantojuvenil há outras possibilidades como a remissão ou a não judicialização do conflito após o encontro restaurativo e o estabelecimento de um plano de recuperação para que o adolescente não precise de internação, desde que o resultado gere segurança para a vítima e reorganização para o infrator. Em São Paulo e no Rio Grande do Sul, por exemplo, há juízes com larga experiência na Justiça Restaurativa com adolescentes, por meio de um processo circular e desritualizado, mais lúdico.

    Qual é a diferença da Justiça Restaurativa e da conciliação?

    Em comum, podemos dizer que não são processos dogmáticos. No entanto, a conciliação é mais voltada para resolver questões de interesse econômico. Os conciliadores se permitem conduzir um pouco o processo para resultados mais efetivos; a conciliação acontece com hora marcada na pauta do tribunal. Já na mediação realizada pela Justiça Restaurativa não é possível estabelecer quando vai acabar, pode demorar dias, meses, até se construir uma solução. Na medida em que você tem um conflito de maior gravidade, que traz uma direção maior de problemas afetados, é preciso dedicar mais tempo. A vítima tem espaço para sugerir o tipo de reparação. O crime gera uma assimetria de poderes: o infrator tem um poder maior sobre a vítima, e a mediação que fazemos busca reequilibrar esses poderes, mas não invertê-los. Os envolvidos podem ir com advogados, embora ao advogado seja reservado um papel muito mais de defesa da voluntariedade de participação e dos limites do acordo, para que este represente uma resposta proporcional àquela ofensa.
     

  • O senhor poderia nos contar um caso interessante aqui do TJDFT?

    Há um caso recente que ocorreu em uma zona rural aqui do Distrito Federal, que era relativamente simples: dois vizinhos que brigavam em relação aos limites da terra ajuizaram um processo que foi resolvido na vara cível, confirmado no tribunal, mas depois continuaram a brigar pelos limites das águas de uma mina. Aquele conflito terminou desenvolvendo para a morte de alguns animais de uma das chácaras, feita supostamente por um dos vizinhos, além de ameaças, e decidimos encaminhá-lo para a Justiça Restaurativa. A solução foi muito interessante. A equipe entendeu por chamar para participar a Agência Nacional de Águas (ANA) e a ONG ambiental WWF, que trouxe como sugestão um programa chamado apadrinhamento de minas. Então aqueles dois confrontantes terminaram fazendo um acordo de proteção pela mina e ficaram plenamente satisfeitos com a solução. Tratava-se de um conflito que já estava na Justiça há mais de dez anos e que, embora com a solução já transitada em julgado, as coisas estavam se encaminhando para um desfecho trágico. Ou seja, a Justiça tradicional resolveu apenas um espectro do problema, o jurídico, mas as demais questões em aberto continuaram se acumulando, até que foi feito esse acordo criativo pelo Programa Justiça Restaurativa do TJDFT.

    Então a Justiça Restaurativa não retira o direito da pessoa recorrer à Justiça tradicional?

    A intervenção restaurativa é suplementar: de par com o processo oferecemos um ambiente para resolver demais problemas relacionados com o conflito. Nada impede que você tenha uma iniciativa, como com adolescentes infratores, que exclua o processo. Primeiro buscamos uma persuasão, depois dissuasão e só depois mecanismos de interdição, que seria a internação. Persuasão significa abrir o ambiente para uma negociação direta entre as partes. Se isso não for alcançado, usamos mecanismos dissuasórios, que seriam um misto de acordo com possibilidades de uma resposta punitiva e, se isso tudo não funcionar, daí sim partimos para outros mecanismos.

    Qual é o maior benefício da Justiça Restaurativa?

    Em muitos casos, essas iniciativas alcançam a pacificação das relações sociais de forma mais efetiva do que uma decisão judicial. 

  • As mediações e a "justiça restaurativa" são ótimas novidades.  No entanto, estamos num estágio em que nem mesmo os agentes do judiciário cumprem plenamente suas responsabilidades, e fica então a impressão de que todos esses movimentos de soluções extrajudiciais refletem na verdade um desejo desses agentes se alijarem de suas obrigações.

  • Concordo com Adriana.

  • Muito bom Maria. Parabéns 

  • Alternativa A) A conciliação, a mediação e os outros métodos de solução consensual de conflitos devem ser estimulados durante todas as fases do processo e não somente durante as audiências de conciliação e de instrução. Acerca do tema, explica a doutrina: "A frustração do resultado da autocomposição na audiência de conciliação ou de mediação (art. 334, NCPC) não impede outras tentativas posteriores de composição do litígio por iniciativa das partes - para se preservar a livre autonomia dos interessados - ou do Poder Judiciário. O NCPC recomenda a autocomposição em outros momentos, como, por exemplo, quando da instalação da audiência de instrução e julgamento (Art. 359 do NCPC) e nas ações de família (art. 694, parágrafo único, do NCPC). Nada impede que a conciliação ou a mediação seja realizada, após o proferimento de sentença de mérito, quando os autos estejam nos Tribunais, caso as partes, expressamente, manifestarem o desejo de autocomposição ou quando o órgão judicial vislumbrar a possibilidade de resolução consensual da controvérsia. Estando os autos no Tribunal, incumbe ao relator estimular e homologar a autocomposição entre as partes (art. 932, I, do NCPC). As partes podem requerer a suspensão do processo judicial art. 313, I, do NCPC) para submeterem o conflito aos meios alternativos de solução de controvérsias" (CAMBI, Eduardo. In: WAMBIER, Teresa Arruda Alvim; e outros. Breves comentários ao novo Código de Processo Civil. 2 ed. São Paulo: Revista dos Tribunais, 2016, p. 948). Afirmativa incorreta.
    Alternativa B) Vide comentário sobre a alternativa A. Não há nenhuma regra que exija que a conciliação seja intermediada pelo juiz. Ela pode ser dirigida por um conciliador ou até mesmo por um terceiro que consiga fazer as partes chegarem a um acordo. Acerca do tema, dispõe o art. 1º, §3º, do CPC/15: "A conciliação, a mediação e outros métodos de solução consensual de conflitos deverão ser estimulados por juízes, advogados, defensores públicos e membros do Ministério Público, inclusive no curso do processo judicial". Afirmativa incorreta.
    Alternativa C) A Justiça Restaurativa está pautada numa técnica de solução de conflitos que tem por base a escuta da vítima e de seu ofensor a fim de obter, por meio dela, uma espécie de acordo de reparação de danos. Não há que se falar em punição mais severa do ofensor em prol da restauração da dignidade do ofendido. O que se busca, com essa forma alternativa de solução de conflitos, é a minimização do dano emocional sofrido pela vítima por meio de uma punição ao ofensor considerada por ela adequada para reparar ou minimizar o dano sofrido. Afirmativa incorreta.
    Alternativa D) A lei processual refere-se a duas principais técnicas alternativas de solução de conflitos: a mediação e a conciliação. Acerca da mediação, dispõe o art. 165, §3º, do CPC/15: "O mediador, que atuará preferencialmente nos casos em que houver vínculo anterior entre as partes, auxiliará aos interessados a compreender as questões e os interesses em conflito, de modo que eles possam, pelo restabelecimento da comunicação, identificar, por si próprios, soluções consensuais que gerem benefícios mútuos". Conforme se nota, embora, na mediação, seja necessária a presença de um terceiro para auxiliar a solução do conflito, ele apenas dirigirá o diálogo a fim de incentivar as partes a realizarem, elas próprias, um acordo. Afirmativa correta.
    Alternativa E) Ao contrário do que se afirma, a técnica da Justiça Restaurativa pode ser aplicada tanto na ocorrência de delitos leves quanto graves. Afirmativa incorreta.

    Gabarito do professor: Letra D.

  • Não concordo com a reclamação do colega PM DGO, se não for do seu interessa, é só não ler.

    Eu, particularmente, não estudo só para saber a resposta da pergunta, quero agregar e desenvolver conhecimento, então, contribuições assim podem ajudar os interessados.

  • Gab. D

    CPC/2015

    Art. 165.  Os tribunais criarão centros judiciários de solução consensual de conflitos, responsáveis pela realização de sessões e audiências de conciliação e mediação e pelo desenvolvimento de programas destinados a auxiliar, orientar e estimular a autocomposição.

    § 3o O mediador, que atuará preferencialmente nos casos em que houver vínculo anterior entre as partes, auxiliará aos interessados a compreender as questões e os interesses em conflito, de modo que eles possam, pelo restabelecimento da comunicação, identificar, por si próprios, soluções consensuais que gerem benefícios mútuos.

     

  • Eu também Michele, procuro estuadar tudo, tanto as erradas quanto as corretas.

  • MEDIADOR - restabelece o diálogo

    Função do mediador: CPC, art. 165, §3º: § 3o O mediador, que atuará preferencialmente nos casos em que houver vínculo anterior entre as partes, auxiliará aos interessados a compreender as questões e os interesses em conflito, de modo que eles possam, pelo restabelecimento da comunicação, identificar, por si próprios, soluções consensuais que gerem benefícios mútuos.

    Na mediação, visa-se recuperar o diálogo entre as partes. Por isso mesmo, são elas que decidem. As técnicas de abordagem do mediador tentam primeiramente restaurar o diálogo para que posteriormente o conflito em si possa ser tratado. Só depois pode se chegar à solução. Na mediação não é necessário interferência, ambas partes chegam a um acordo sozinhas, se mantém autoras de suas próprias soluções. Conflitos familiares e de vizinhança, por exemplo, muitas vezes são resolvidos apenas com o estabelecimento da comunicação respeitosa entre os envolvidos.

     

    CO-N-CILIADOR - sugere a solução  > N-ÃO C-ONHECE

    CPC, art. 165, §2º: O conciliador, que atuará preferencialmente nos casos em que não houver vínculo anterior entre as partes, poderá sugerir soluções para o litígio, sendo vedada a utilização de qualquer tipo de constrangimento ou intimidação para que as partes conciliem.

    conciliação pode ser mais indicada quando há uma identificação evidente do problema, quando este problema é verdadeiramente a razão do conflito - não é a falta de comunicação que impede o resultado positivo. Diferentemente do mediador, o conciliador tem a prerrogativa de sugerir uma solução.Essa polarização pede uma intervenção do conciliador no sentido de um acordo justo para ambas as partes e no estabelecimento de como esse acordo será cumprido. Causas trabalhistas costumam ser um objeto onde a conciliação atua com eficiência.

     

    Art. 1o, Lei 13.140 Esta Lei dispõe sobre a mediação como meio de solução de controvérsias entre particulares e sobre a autocomposição de conflitos no âmbito da administração pública. 

    Parágrafo único.  Considera-se mediação a atividade técnica exercida por terceiro imparcial sem poder decisório, que, escolhido ou aceito pelas partes, as auxilia e estimula a identificar ou desenvolver soluções consensuais para a controvérsia. 

     

    Art. 2o A mediação será orientada pelos seguintes princípios: 

    I - imparcialidade do mediador; 

    II - isonomia entre as partes; 

    III - oralidade; 

    IV - informalidade; 

    V - autonomia da vontade das partes; 

    VI - busca do consenso; 

    VII - confidencialidade; 

    VIII - boa-fé. 

     

  • Resposta: D

    Em funcionamento há cerca de 10 anos no Brasil, a prática da Justiça Restaurativa tem se expandido pelo país. Conhecida como uma técnica de solução de conflitos que prima pela criatividade e sensibilidade na escuta das vítimas e dos ofensores, a prática tem iniciativas cada vez mais diversificadas e já coleciona resultados positivos.

    Em São Paulo, a Justiça Restaurativa tem sido utilizada em dezenas de escolas públicas e privadas, auxiliando na prevenção e na diminuição do agravamento de conflitos. No Rio Grande do Sul, juízes aplicam o método para auxiliar nas medidas socioeducativas cumpridas por adolescentes em conflito com a lei, conseguindo recuperar para a sociedade jovens que estavam cada vez mais entregues ao caminho do crime. No Distrito Federal, o Programa Justiça Restaurativa é utilizado em crimes de pequeno e médio potencial ofensivo, além dos casos de violência doméstica. Na Bahia e no Maranhão, o método tem solucionado os crimes de pequeno potencial ofensivo, sem a necessidade de prosseguir com processos judiciais.

  • Michelle Mikoski, sem educaçona !!!! kkkkkkkk...

  • Alternativa A) A conciliação, a mediação e os outros métodos de solução consensual de conflitos devem ser estimulados durante todas as fases do processo e não somente durante as audiências de conciliação e de instrução. Acerca do tema, explica a doutrina: "A frustração do resultado da autocomposição na audiência de conciliação ou de mediação (art. 334, NCPC) não impede outras tentativas posteriores de composição do litígio por iniciativa das partes - para se preservar a livre autonomia dos interessados - ou do Poder Judiciário. O NCPC recomenda a autocomposição em outros momentos, como, por exemplo, quando da instalação da audiência de instrução e julgamento (Art. 359 do NCPC) e nas ações de família (art. 694, parágrafo único, do NCPC). Nada impede que a conciliação ou a mediação seja realizada, após o proferimento de sentença de mérito, quando os autos estejam nos Tribunais, caso as partes, expressamente, manifestarem o desejo de autocomposição ou quando o órgão judicial vislumbrar a possibilidade de resolução consensual da controvérsia. Estando os autos no Tribunal, incumbe ao relator estimular e homologar a autocomposição entre as partes (art. 932, I, do NCPC). As partes podem requerer a suspensão do processo judicial art. 313, I, do NCPC) para submeterem o conflito aos meios alternativos de solução de controvérsias" (CAMBI, Eduardo. In: WAMBIER, Teresa Arruda Alvim; e outros. Breves comentários ao novo Código de Processo Civil. 2 ed. São Paulo: Revista dos Tribunais, 2016, p. 948). Afirmativa incorreta.

    Alternativa B) Vide comentário sobre a alternativa A. Não há nenhuma regra que exija que a conciliação seja intermediada pelo juiz. Ela pode ser dirigida por um conciliador ou até mesmo por um terceiro que consiga fazer as partes chegarem a um acordo. Acerca do tema, dispõe o art. 1º, §3º, do CPC/15: "A conciliação, a mediação e outros métodos de solução consensual de conflitos deverão ser estimulados por juízes, advogados, defensores públicos e membros do Ministério Público, inclusive no curso do processo judicial". Afirmativa incorreta.

  • Alternativa C) A Justiça Restaurativa está pautada numa técnica de solução de conflitos que tem por base a escuta da vítima e de seu ofensor a fim de obter, por meio dela, uma espécie de acordo de reparação de danos. Não há que se falar em punição mais severa do ofensor em prol da restauração da dignidade do ofendido. O que se busca, com essa forma alternativa de solução de conflitos, é a minimização do dano emocional sofrido pela vítima por meio de uma punição ao ofensor considerada por ela adequada para reparar ou minimizar o dano sofrido. Afirmativa incorreta.

    Alternativa D) A lei processual refere-se a duas principais técnicas alternativas de solução de conflitos: a mediação e a conciliação. Acerca da mediação, dispõe o art. 165, §3º, do CPC/15: "O mediador, que atuará preferencialmente nos casos em que houver vínculo anterior entre as partes, auxiliará aos interessados a compreender as questões e os interesses em conflito, de modo que eles possam, pelo restabelecimento da comunicação, identificar, por si próprios, soluções consensuais que gerem benefícios mútuos". Conforme se nota, embora, na mediação, seja necessária a presença de um terceiro para auxiliar a solução do conflito, ele apenas dirigirá o diálogo a fim de incentivar as partes a realizarem, elas próprias, um acordo. Afirmativa correta.

    Alternativa E) Ao contrário do que se afirma, a técnica da Justiça Restaurativa pode ser aplicada tanto na ocorrência de delitos leves quanto graves. Afirmativa incorreta.

    Gabarito: Letra D.

  • Sobre os conceitos gerais de Conciliação, de Mediação de Conflitos e de Justiça Restaurativa, é correto afirmar que:  A Mediação é uma forma de solução de conflitos na qual uma terceira pessoa, neutra e imparcial, facilita o diálogo entre as partes, para que elas construam, com autonomia e solidariedade, a melhor solução para o problema.

  • gente, apenas um comentário pertinente a letra D

    Em 2021, essa realidade prescrita na teoria tá bem distante da realidade, colegas advogados saberão do que estou falando.


ID
2478733
Banca
PUC-PR
Órgão
TJ-PR
Ano
2017
Provas
Disciplina
Psicologia
Assuntos

Leia a citação a seguir.

“A violência ou abuso físico intrafamiliar está relacionado ao uso de força física contra a criança ou adolescente por parte de seus cuidadores, sejam pais adotivos ou biológicos ou ainda outros, que devam zelar por seu bem-estar e integridade física e emocional.” (Cecconello, De Antoni e Koller, 2003, apud De Antoni e Koller in: Habigzang et al., 2012, p. 43.)


A partir do contexto da violência intrafamiliar, avalie as afirmações a seguir.

I. Pais abusivos demonstram menos preocupação com os filhos e mais desconforto frente a experiências negativas de outros, incluindo seus próprios filhos.

II. A falta de habilidade dos pais, bem como a precariedade de estratégias para lidar com os desafios advindos do desenvolvimento dos filhos, podem gerar ainda mais conflitos.

III. Pais com alto potencial para o abuso físico demonstram limitações empáticas.

IV. Pais abusivos expressam menos compaixão e solidariedade, além de demonstrarem sentimento de culpa pelos erros e atitudes tomadas para com seus filhos.

V. O fato de os filhos serem vistos pelos pais como causadores de conflitos pode estar associado ao comportamento desafiador da criança e do adolescente.

Estão CORRETAS somente as afirmações

Alternativas
Comentários
  • LETRA B

    I, II, III e V.

    I – Pais abusivos demonstram menos preocupação com os filhos e mais desconforto frente a experiências negativas de outros, incluindo seus próprios filhos.

    II – A falta de habilidade dos pais, bem como a precariedade de estratégias para lidar com os desafios advindos do desenvolvimento dos filhos, podem gerar ainda mais conflitos.

    III – Pais com alto potencial para o abuso físico demonstram limitações empáticas.

    IV – Pais abusivos expressam menos compaixão e solidariedade, além de demonstrarem sentimento de culpa pelos erros e atitudes tomadas para com seus filhos.

    V – O fato de os filhos serem vistos pelos pais como causadores de conflitos pode estar associado ao comportamento desafiador da criança e do adolescente.


ID
2478736
Banca
PUC-PR
Órgão
TJ-PR
Ano
2017
Provas
Disciplina
Psicologia
Assuntos

Considere as informações da situação a seguir.

Ana está lidando com a ansiedade e depressão com muita frequência. Como consequência, cortou relações interpessoais até com amigos mais íntimos e com as pessoas de sua família. O tempo todo está irritada e frustrada. Abandonou as aulas na faculdade e tem dificuldade para dormir e se alimentar. Acredita que tudo o que faz não tem nenhum valor e é incapaz de experimentar prazer.

Os sintomas descritos na situação hipotética correspondem a

Alternativas
Comentários
  • Como consequência, cortou relações interpessoais até com amigos mais íntimos e com as pessoas de sua família. O tempo todo está irritada e frustrada. Abandonou as aulas na faculdade e tem dificuldade para dormir e se alimentar (funções físicas perturbadas). Acredita que tudo o que faz não tem nenhum valor (sintomas de menos valia) e é incapaz de experimentar prazer (anedonia)


ID
2478739
Banca
PUC-PR
Órgão
TJ-PR
Ano
2017
Provas
Disciplina
Psicologia
Assuntos

Sobre os conhecimentos de avaliação psicológica, assinale a alternativa CORRETA.

Alternativas
Comentários
  • Questão  fundamentada nas  5 etapas da avaliação psicológica ou psicodiagnóstica, segundo Cunha: 

    1ª- levantamente e esclareciemnto dos motivos ( manifesto e latentes)... . Hipótes iniciais e objetivos dos exames ( demanda, hipótese);

    2º- reflexão sobre o material coletado na etapa anteriror... Planejar e selecionar os instrumentos  a serem  utilizados na avaliação;

    3º- realização da estratégia diagnóstica  planejada;( aplicação de teste e/ou técnica )

    4º- estudo do material coletado; 

    5º-  entrevista de devolução; 

     

  • Quais os passos mínimos para se fazer avaliação psicológica?

    O processo de avaliação psicológica apresenta alguns passos essenciais para que seja possível alcançar os resultados esperados, a saber:

    ¬ Levantamento dos objetivos da avaliação e particularidades do indivíduo ou grupo a ser avaliado. Tal processo permite a escolha dos instrumentos/estratégias mais adequados para a realização da avaliação psicológica;

    ¬ Coleta de informações pelos meios escolhidos (entrevistas, dinâmicas, observações e testes projetivos e/ou psicométricos, etc). É importante salientar que a integração dessas informações devem ser suficientemente amplas para dar conta dos objetivos pretendidos pelo processo de avaliação. Não é recomendada a utilização de uma só técnica ou um só instrumento para a avaliação;

    ¬ Integração das informações e desenvolvimento das hipóteses iniciais. Diante destas, o psicólogo pode constatar a necessidade de utilizar outros instrumentos/estratégias de modo a refinar ou elaborar novas hipóteses;

    ¬ Indicação das respostas à situação que motivou o processo de avaliação e comunicação cuidadosa dos resultados, com atenção aos procedimentos éticos implícitos e considerando as eventuais limitações da avaliação. Nesse processo, os procedimentos variam de acordo com o contexto e propósito da avaliação.

    Psicologia Pra Passar em Concursos 

  • Quais os passos mínimos para se fazer avaliação psicológica? O processo de avaliação psicológica apresenta alguns passos essenciais para que seja possível alcançar os resultados esperados, a saber:

    ¬ Levantamento dos objetivos da avaliação e particularidades do indivíduo ou grupo a ser avaliado. Tal processo permite a escolha dos instrumentos/estratégias mais adequados para a realização da avaliação psicológica;

    ¬ Coleta de informações pelos meios escolhidos (entrevistas, dinâmicas, observações e testes projetivos e/ou psicométricos, etc). É importante salientar que a integração dessas informações devem ser suficientemente amplas para dar conta dos objetivos pretendidos pelo processo de avaliação. Não é recomendada a utilização de uma só técnica ou um só instrumento para a avaliação;

    ¬ Integração das informações e desenvolvimento das hipóteses iniciais. Diante destas, o psicólogo pode constatar a necessidade de utilizar outros instrumentos/estratégias de modo a refinar ou elaborar novas hipóteses;

    ¬ Indicação das respostas à situação que motivou o processo de avaliação e comunicação cuidadosa dos resultados, com atenção aos procedimentos éticos implícitos e considerando as eventuais limitações da avaliação. Nesse processo, os procedimentos variam de acordo com o contexto e propósito da avaliação.

    Fonte: Cartilha sobre Avaliação Psicológica CFP, 2007.

  • a) Essa afirmativa está relacionada à dimensão ética da avaliação psicológica;

    b) A elaboração de um contrato de prestação de serviços é obrigatória;

    d) A elaboração e a entrega de um documento com os resultados do processo de avaliação psicológica são obrigatórias;

    e) O acompanhamento acerca dos instrumentos considerados favoráveis para uso pelo CFP é de responsabilidade do psicólogo;

     

    Quem escolheu a busca não pode recusar a travessia - Guimarães Rosa

    ------------------- 

    Gabarito: C


ID
2478742
Banca
PUC-PR
Órgão
TJ-PR
Ano
2017
Provas
Disciplina
Psicologia
Assuntos

A maior parte dos casos de abuso sexual ocorre no seio das famílias, apesar das proibições biológicas e culturais do incesto.

Sendo assim, analise as seguintes proposições:

I. A família é uma instituição caracterizada como “sagrada” pela Religião e como “a base da sociedade” pelo Direito.

II. Para a Psicologia, a família é uma instituição na qual as relações se estabelecem independentemente do escopo social.

III. As relações familiares podem tanto promover o desenvolvimento saudável quanto desencadear desajustes, violências e psicopatologias.

IV. O abuso sexual infantil pode ocorrer em qualquer família e não somente naqueles consideradas “desestruturadas”.

V. A falta de comunicação é uma característica importante na dinâmica das famílias abusivas.

No que tange aos padrões, características e dinâmicas familiares no abuso sexual infantil, é CORRETO o que se afirma apenas em

Alternativas
Comentários
  • A única assertiva incorreta é a II:

     

    II. Para a Psicologia, a família é uma instituição na qual as relações se estabelecem independentemente do escopo social.

     

    Contudo, para a psicologia, sobretudo a social, as relações se estabelecem mediante a interação social, ou seja, há uma interdependência entre os indivíduos, as instituições às quais eles pertencem e o meio social.

     

    ------------------- 

    Gabarito: B


ID
2478745
Banca
PUC-PR
Órgão
TJ-PR
Ano
2017
Provas
Disciplina
Psicologia
Assuntos

Com base nos conceitos de contingência e metacontingência do Behaviorismo Radical, assinale a alternativa CORRETA.

Alternativas
Comentários
  • RESPOSTA "A"

    Vejamos,

    "Segundo Skinner (1969), a formulação adequada da relação organismo-ambiente é especificada pela contingência, que dita a organização da relação entre eventos ambientais e eventos comportamentais. Isso quer dizer que a contingência é uma �ferramenta� que possibilita a compreensão e modificação do comportamento. Portanto, quando o analista do comportamento olha para o fluxo comportamental ele vê uma relação entre estímulos antecedentes, resposta e conseqüências.

    Nesse ponto, é preciso fazer uma ressalva. Como estamos buscando uma definição de comportamento, que deve ser �maior� que o conceito de comportamento operante, não podemos nos deter na análise das contingências de reforço. Devemos, portanto, continuar a admitir que a contingência tríplice é o modo de organização da relação entre eventos comportamentais e ambientais na Análise do Comportamento, desde que ela não seja identificada com a contingência de reforço. Assim, quando falamos de processos comportamentais estamos falando de contingências, que embora incluam contingências de reforço, não se reduzem a elas.

    Sendo assim, os eventos que participam de um comportamento (independente de se tratar de um operante, liberado ou reflexo) se organizam na forma �estímulo-resposta-conseqüência�. No caso do operante, por exemplo, é a contingência de reforço quem dita o modo como os eventos se relacionam – uma resposta é emitida na presença de estímulos e produz alterações no campo estimulacional (remove estímulos presentes ou produz novos estímulos). Além de ditar a forma de organização entre eventos, as contingências, ou processos comportamentais, são uma espécie de �ponte� entre eventos e estados comportamentais, indicando de que modo podemos partir de eventos para construir estados".

     

    Uma proposta de definição de comportamento no behaviorismo radical

    http://pepsic.bvsalud.org/scielo.php?script=sci_arttext&pid=S1517-55452008000100002

  • Portanto,  a trípilce contigência seria : 

    As condições ambientais em que  organismo se encontra.

    A reação desse indivíduo a essas condições.(resposta)

    As consequências que essa reação lhe traz e os efeitos que essas consequências produzem.

  • Quando falamos em Tríplice Contingência, estamos falando de uma relação de interdependência entre estímulos e respostas, relação esta na qual um estímulo consequente a uma classe de respostas altera a probabilidade de emissão desta mesma classe de respostas no futuro em uma situação semelhante. A Tríplice Contingência é a representação gráfica sobre como determinados comportamentos estão relacionados. É a formula usada pela Análise do Comportamento para estudar e entender como estes comportamentos se relacionam com os estímulos que os antecedem e com os estímulos que a eles se seguem. Antes de qualquer coisa, é preciso diferenciar respostas (R) de comportamento (Sd – R – Sr+). 

    Resposta, quando falamos em comportamento operante, pode ser definida como tudo que é eliciado por um estímulo antecedente (que pode ser contextual, discriminativo, evocativo etc…) e produz uma conseqüência (reforço positivo, negativo, punição positiva ou negativa). 

    Comportamento é a relação entre estímulos antecedentes e conseqüentes a uma resposta É a relação entre todos os termos da contingência, inclusive a resposta. Segundo Souza (2001, p.85), “o enunciado de uma contingência é feito em forma de afirmações do tipo se…, então…” ou seja, podemos dizer como exemplo, se eu fizer X então vai acontecer Y.

    Uma metacontingência existe se o objeto de análise for uma prática cultural de um grupo de indivíduos, se as conseqüências desta pratica, para o grupo, e se seus antecedentes puderem ser identificados (Glenn, 1986). No estudo das contingências sociais usando a unidade de análise metacontingência, há uma distinção entre contingências de reforçamento (relações de contingências entre uma classe de respostas e uma conseqüência comum) e metacontingências (relações de contingência entre uma classe de operantes e uma conseqüência cultural e comum a longo prazo).Em Estados democráticos de direito, como o Brasil, as metacontingências que controlam a sociedade são deliberadas democraticamente por um Congresso eleito pela maioria da população. Alguns exemplos são: a Constituição, o Código Penal, o Código Civil e o Estatuto da Criança e do Adolescente.

     


ID
2478748
Banca
PUC-PR
Órgão
TJ-PR
Ano
2017
Provas
Disciplina
Psicologia
Assuntos

O abuso sexual praticado contra crianças e adolescentes se caracteriza por ações de conteúdo sexualizado impostas às vítimas.

Em alusão à temática, é CORRETO afirmar que

Alternativas
Comentários
  • a) são intrafamiliares

    b) mais frequentes;

    d) a primeira parte está correta, contudo essa reação de poder leva o abusdor a seduzir a vítima; (mito da criança sedutora)

    e) desperta a confiança;

     

    ------------------- 

    Gabarito: C

  • Um professor pode ter laços significativos com a criança e nem por isso ser parte da família. Questão pobre de sentido


ID
2478751
Banca
PUC-PR
Órgão
TJ-PR
Ano
2017
Provas
Disciplina
Psicologia
Assuntos

No que se refere à compreensão do desenvolvimento humano na perspectiva da Análise do Comportamento, assinale a alternativa CORRETA.

Alternativas
Comentários
  •  

     

    Para a análise do comportamento, o processo do desenvolvimento sofre influência tanto da maturação biológica quanto das experiências que o individuo vivencia. V

     

  • LETRA D


ID
2478754
Banca
PUC-PR
Órgão
TJ-PR
Ano
2017
Provas
Disciplina
Psicologia
Assuntos

Sobre os quadros de Transtorno de Personalidade Boderline, pode-se referir:

Alternativas
Comentários
  • TRANSTORNO DA PERSONALIDADE BORDERLINE (OU “EMOCIONALMENTE INSTÁVEL”) 1. Instabilidade emocional intensa 2. Sentimentos crônicos de vazio 3. Relacionamentos pessoais intensos, mas muito instáveis, oscilando em curtos períodos de uma grande “paixão” ou “amizade” para “ódio” e “rancor” profundos 4. Esforços excessivos para evitar abandono 5. Dificuldades sérias e instabilidade com relação à auto-imagem, aos objetivos e às preferências pessoais (inclusive a sexual) 6. Atos repetitivos de autolesão, envolvendo-se em atuações perigosas (como guiar muito embriagado e velozmente, intoxicar-se com substâncias, etc.) 7. Atos suicidas repetitivos

    [Psicopatologia e semiologia dos transtornos mentais - Paulo Dalgalarrondo]

     

     

  • A) correta.

    B) Um transtorno da personalidade é um padrão persistente de experiência interna e comportamento que se desvia acentuadamente das expectativas da cultura do indivíduo, é difuso e inflexível, começa na adolescência ou no início da fase adulta, é estável ao longo do tempo e leva a sofrimento ou prejuízo.

    C) Transtorno da personalidade dependente

    D) Errada.

    E) Transtorno da personalidade antissocial


ID
2478757
Banca
PUC-PR
Órgão
TJ-PR
Ano
2017
Provas
Disciplina
Psicologia
Assuntos

Uma avaliação psicológica pode apresentar diferentes objetivos, dependendo dos motivos alegados ou reais do encaminhamento. Sobre os diferentes objetivos de uma avaliação psicológica, assinale a alternativa CORRETA.

Alternativas
Comentários
  • RESPOSTA "C"

    Vejamos, 

    a) A avaliação realizada com fins de determinar o curso provável de um caso é a avaliação com objetivo diagnóstico. (falso)

    O Prognóstico determina o curso provável do caso e não a avaliação com objetivo diagnóstico.

     b) Na avaliação de classificação nosológica, objetiva-se determinar o nível de funcionamento da personalidade, em que são examinadas as funções do ego, em especial a de insight.(falso)

    Na Classificação Nosológica hipóteses iniciais são testadas, tomando como referência critérios diagnósticos.

    A avaliação que determina o nível do funcionamento da personalidade e que são examinadas as funções do ego, em especial a de insight, condições  do sistema de defesas, para facilicitar a indicação de recursos terapêuticos e prever a possivel resposta aos mesmos é a Avaliação Compreeensiva.

     c) Um exemplo de avaliação psicológica com objetivo de classificação simples é a avaliação do nível intelectual, em que o resultado quantitativo obtido pelo avaliando é comparado com os resultados de outros sujeitos da população. (verdadeiro)

     d) A avaliação com objetivo de diagnóstico diferencial é aquela em que o psicólogo é solicitado judicialmente a responder sobre questões relacionadas à insanidade.  (falso)

    A avaliação com objetivo de diagnóstico diferencial investiga irregularidades ou inconsistências do quadro sintomático, para diferenciar alternativas diagnósticas, níveis de funcionamento ou a natureza da patologia. 

     e) A perícia forense pode ser considerada um objetivo de avaliação. Nesse caso, o psicólogo precisa diagnosticar o sujeito de avaliação e se posicionar acerca da tomada de decisão judicial.(falso)

    A Pericia Forense fornece subsídios para questoes relacionadas com "insanidade" competência para o exercício das unções de cidadão, avaliação de incapacidades ou patologias que podem se associar com infrações da lei, etc. 

     

    Bibliografia:

    Cunha. Psicodiagnóstico V. 2000 pág. 27 "Quadro 2.1 Objetivos de uma avaliação psicológica clínica"

  • Classificação simples O exame compara a amostra do comportamento do examinando com os resultados de outros sujeitos da população geral ou de grupos específicos, com condições demográficas equivalentes; esses resultados são fornecidos em dados quantitativos, classificados sumariamente como em uma avaliação de nível intelectual.


    Descrição Ultrapassa a classificação simples, interpretando diferenças de escores, identificando forças e fraquezas e descrevendo o desempenho do paciente, como em uma avaliação de déficits neuropsicológicos.

     

    Classificação nosológica Hipóteses iniciais são testadas, tomando como referência critérios diagnósticos.

     

    Diagnóstico diferencial São investigadas irregularidades ou inconsistências do quadro sintomático, para diferenciar alternativas diagnósticas, níveis de funcionamento ou a natureza da patologia.

    Avaliação compreensiva É determinado o nível de funcionamento da personalidade, são examinadas as funções do ego, em especial a de insight, condições do sistema de defesas, para facilitar a indicação de recursos terapêuticos e prever a possível resposta aos mesmos.
    Entendimento dinâmico Ultrapassa o objetivo anterior, por pressupor um nível mais elevado de inferência clínica, havendo uma integração de dados com base teórica. Permite chegar a explicações de aspectos comportamentais nem sempre acessíveis na entrevista, à antecipação de fontes de dificuldades na terapia e à definição de focos terapêuticos, etc.
     

    Prevenção Procura identificar problemas precocemente, avaliar riscos, fazer uma estimativa de forças e fraquezas do ego, de sua capacidade para enfrentar situações novas, difíceis, estressantes.

     

    Prognóstico Determina o curso provável do caso.
    Perícia forense Fornece subsídios para questões relacionadas com “insanidade”, competência para o exercício das funções de cidadão, avaliação de incapacidades ou patologias que podem se associar com infrações da lei, etc.

     

    Visite nossa Facebook: Psicologia Pra Passar


ID
2478760
Banca
PUC-PR
Órgão
TJ-PR
Ano
2017
Provas
Disciplina
Psicologia
Assuntos

O indivíduo que apresenta Transtorno de Personalidade Narcisista tende a ser

Alternativas
Comentários
  • TRANSTORNO DA PERSONALIDADE NARCISISTA 1. O indivíduo apresenta senso grandioso (e irreal) da importância de sua pessoa. Julga ter talentos especiais, espera ser reconhecido como superior, sem que tenha feito algo concreto para isso 2. Muito voltado para fantasias de grande sucesso pessoal, de poder, brilho, beleza ou de um amor ideal 3. Acha-se excepcionalmente “especial” e “único”, acreditando que só pessoas ou instituições também excepcionalmente especiais ou únicas podem estar à sua altura 4. Requer admiração excessiva 5. Tende a ser “explorador” nas relações interpessoais, buscando vantagens sobre os outros para atingir o seu fim ou sucesso pessoal 6. Sem empatia pelas pessoas comuns 7. Freqüentemente invejoso dos outros ou do sucesso alheio; acha sempre que os outros têm inveja dele 8. Freqüentemente arrogante

    [Psicopatologia e semiologia dos transtornos mentais - Paulo Dalgalarrondo]

     

  • Um padrão difuso de grandiosidade (em fantasia ou comportamento), necessidade de admiração e falta de empatia que surge no início da vida adulta e está presente em vários contextos, conforme indicado por cinco (ou mais) dos seguintes:
    1. Tem uma sensação grandiosa da própria importância (p. ex., exagera conquistas e talentos, espera ser reconhecido como superior sem que tenha as conquistas correspondentes).
    2. É preocupado com fantasias de sucesso ilimitado, poder, brilho, beleza ou amor ideal.

    3. Acredita ser “especial” e único e que pode ser somente compreendido por, ou associado a, outras pessoas (ou instituições) especiais ou com condição elevada.
    4. Demanda admiração excessiva.
    5. Apresenta um sentimento de possuir direitos (i.e., expectativas irracionais de tratamento especialmente favorável ou que estejam automaticamente de acordo com as próprias expectativas).
    6. É explorador em relações interpessoais (i.e., tira vantagem de outros para atingir os próprios fins).
    7. Carece de empatia: reluta em reconhecer ou identificar-se com os sentimentos e as necessidades dos outros.
    8. É frequentemente invejoso em relação aos outros ou acredita que os outros o invejam.

    (DSM-V)

  • Questãozinha meio renga, mas...

    a) Acredita ser extremamente especial e incrível, mas acho que isso não quer dizer necessariamente alta autoestima e que se expor nas RS seja um comportamento notável...

    b) Sim, empatia pra quê se se considera o centro do mundo, e todos os outros não tem importância?!.

    c) Geralmente indivíduo que necessita tirar vantagem de outras pessoas, sem deixar de ser empático (?!)

    d) A primeira parte refere-se ao transtorno paranóide

    e) Essa eu fiquei na dúvida, mas acho que o que a torna errada é afirmar sobre necessidade de atenção, sendo que as descrição sobre o transtorno narcisista falam em uma "crença" ou "ideia" de ter atributos para ter essa atenção e não em uma necessidade

  • Letra E não poderia... palavra chave para transtorno histriônico é "sedutor". Tomar cuidado porque uma pessoa com este transtorno também é egocêntrica.

  • No Transtorno de Personalidade Narcisista sofre-se de baixa autoestima. O transtorno é uma defesa a baixa autoestima.


ID
2478763
Banca
PUC-PR
Órgão
TJ-PR
Ano
2017
Provas
Disciplina
Psicologia
Assuntos

A Resolução CFP nº 007/2003, a qual institui o Manual de Elaboração de Documentos Escritos produzidos pelo psicólogo, decorrentes de avaliação psicológica, estabelece alguns parâmetros para a construção de quatro modalidades de documentos. A esse respeito, avalie as proposições a seguir, considerando o texto da referida resolução.

I. Documento que certifica uma determinada situação ou estado psicológico, tendo como finalidade afirmar sobre as condições psicológicas de quem, por requerimento, o solicita.

II. Deve ser subsidiado em dados colhidos e analisados, à luz de instrumental técnico, consubstanciado em referencial técnico-filosófico e científico adotado pelo psicólogo.

III. O item Descrição da Demanda é destinado à narração das informações referentes à problemática apresentada e dos motivos, razões e expectativas que produziram o pedido do documento.

IV. Apresentar resposta esclarecedora, no campo do conhecimento psicológico, por meio de uma avaliação especializada, de uma “questão-problema”, visando a dirimir dúvidas que estão interferindo na decisão.

V. Deve conter narrativa detalhada e didática, com clareza, precisão e harmonia, tornando-se acessível e compreensível ao destinatário.

É CORRETO o que se afirma sobre o Relatório/Laudo Psicológico em

Alternativas
Comentários
  • O relatório psicológico é uma peça de natureza e valor científicos, devendo conter narrativa detalhada e didática, com clareza, precisão e harmonia, tornando-se acessível e compreensível ao destinatário. Os termos técnicos devem, portanto, estar acompanhados das explicações e/ou conceituação retiradas dos fundamentos teórico-filosóficos que os sustentam.

    Demanda: Esta parte é destinada à narração das informações referentes à problemática apresentada e dos motivos, razões e expectativas que produziram o pedido do documento. Nesta parte, deve-se apresentar a análise que se faz da demanda de forma a justificar o procedimento adotado.

  • Alternativa A

    I - Errado - Característica do Atestado Psicológico: 

    É um documento expedido pelo psicólogo que certifica uma determinada situação ou estado psicológico, tendo como finalidade afirmar sobre as condições psicológicas de quem, por requerimento, o solicita, com fins de: 

    a) Justificar faltas e/ou impedimentos do solicitante;
    b) Justificar estar apto ou não para atividades específicas, após realização de um processo de avaliação psicológica, dentro do rigor técnico e ético que subscreve esta Resolução;
    c) Solicitar afastamento e/ou dispensa do solicitante, subsidiado na afirmação atestada do fato, em acordo com o disposto na Resolução CFP no 015/96.

    II - Correto - O relatório ou laudo psicológico é uma apresentação descritiva acerca de situações e/ou condições psicológicas e suas determinações históricas, sociais, políticas e culturais, pesquisadas no processo de avaliação psicológica. Como todo DOCUMENTO, deve ser subsidiado em dados colhidos e analisados, à luz de um instrumental técnico (entrevistas, dinâmicas, testes psicológicos, observação, exame psíquico, intervenção verbal),consubstanciado em referencial técnico-filosófico e científico adotado pelo psicólogo.

    III - Correto - Esta parte é destinada à narração das informações referentes à problemática apresentada e dos motivos, razões e expectativas que produziram o pedido do documento. Nesta parte, deve-se apresentar a análise que se faz da demanda de forma a justificar o procedimento adotado.

    IV - Errado - Características do Parecer Psicológico. O parecer tem como finalidade apresentar resposta esclarecedora, no campo do conhecimento psicológico, através de uma avaliação especializada, de uma “questão-problema”, visando a dirimir dúvidas que estão interferindo na decisão, sendo, portanto, uma resposta a uma consulta, que exige de quem responde competência no assunto.

    V - Correto - O relatório psicológico é uma peça de natureza e valor científicos, devendo conter narrativa detalhada e didática, com clareza, precisão e harmonia, tornando-se acessível e compreensível ao destinatário. Os termos técnicos devem, portanto, estar acompanhados das
    explicações e/ou conceituação retiradas dos fundamentos teórico-filosóficos que os sustentam.

  • I. Documento que certifica uma determinada situação ou estado psicológico, tendo como finalidade afirmar sobre as condições psicológicas de quem, por requerimento, o solicita. ATESTADO PSICOLÓGICO

    II. Deve ser subsidiado em dados colhidos e analisados, à luz de instrumental técnico, consubstanciado em referencial técnico-filosófico e científico adotado pelo psicólogo. RELATÓRIO/LAUDO PSICOLÓGICO

    III. O item Descrição da Demanda é destinado à narração das informações referentes à problemática apresentada e dos motivos, razões e expectativas que produziram o pedido do documento. RELATÓRIO/LAUDO PSICOLÓGICO

    IV. Apresentar resposta esclarecedora, no campo do conhecimento psicológico, por meio de uma avaliação especializada, de uma “questão-problema”, visando a dirimir dúvidas que estão interferindo na decisão. PARECER PSICOLÓGICO

    V. Deve conter narrativa detalhada e didática, com clareza, precisão e harmonia, tornando-se acessível e compreensível ao destinatário. RELATÓRIO/LAUDO PSICOLÓGICO

     

    GABARITO - A

     

     

     

    https://www.instagram.com/diariodapsicologa/?hl=pt-br

  • RESOLUÇÃO CFP Nº 007/2003

     

    Somente as assertivas II, III e V se referem ao laudo/relatório. Vejamos:

     

    3.1 – O relatório ou laudo psicológico é uma apresentação descritiva acerca de situações e/ou condições psicológicas e suas determinações históricas, sociais, políticas e culturais, pesquisadas no processo de avaliação psicológica. Como todo documento, deve ser subsidiado em dados colhidos e analisados, à luz de um instrumental técnico (entrevistas, dinâmicas, testes psicológicos, observação, exame psíquico, intervenção verbal), consubstanciado em referencial técnico-filosófico e científico adotado pelo psicólogo; (II)

     

    3.2 – O relatório psicológico é uma peça de natureza e valor científicos, devendo conter narrativa detalhada e didática, com clareza, precisão e harmonia, tornando-se acessível e compreensível ao destinatário. Os termos técnicos devem, portanto, estar acompanhados das explicações e/ou conceituação retiradas dos fundamentos teórico-filosóficos que os sustentam; (V)

     

    3.2.2 –  Descrição da Demanda

    Esta parte é destinada à narração das informações referentes à problemática apresentada e dos motivos, razões e expectativas que produziram o pedido do documento. Nesta parte, deve-se apresentar a análise que se faz da demanda de forma a justificar o procedimento adotado; (III)

     

    Os erros das demais assertivas:

    I) trata-se do conceito de atestado psicológico;

    IV) trata-se do conceito de parecer

     

    Quem escolheu a busca não pode recusar a travessia - Guimarães Rosa

    ------------------- 

    Gabarito: A


ID
2478766
Banca
PUC-PR
Órgão
TJ-PR
Ano
2017
Provas
Disciplina
Psicologia
Assuntos

Compreender o conceito de Tendência Atualizante é fundamental para entender a base da Psicoterapia Humanista Centrada na Pessoa. Das alternativas a seguir, assinale a que melhor demonstra esse conceito.

Alternativas
Comentários
  •  "todo organismo é movido por uma tendência inerente para desenvolver todas as suas potencialidades e para desenvolvê-las de maneira a favorecer sua conservação e seu enriquecimento"

    considerava a tendência atualizante uma capacidade do indivíduo se reorganizar e se reestruturar, uma vez que certas condições facilitadoras deste processo sejam atendidas.

    a tendência à atualização é a mais fundamental do organismo em sua totalidade. Preside o exercício de todas as funções, tanto físicas quanto experienciais. E visa constantemente desenvolver as potencialidades do indivíduo para assegurar sua conservação e seu enriquecimento, levando-se em conta as possibilidades e limites do meio (p. 41).

    Portanto, percebemos que a tendência atualizante é inerente ao indivíduo, sendo uma disposição para que sejam desenvolvidas suas potencialidades, porém, parcialmente condicionadas às limitações do meio. 

    (...) existe em todo organismo, em qualquer nível, um fluxo subjacente de movimento para uma realização construtiva de suas possibilidades intrínsecas. Há no homem uma tendência natural para o desenvolvimento completo. O termo mais frequentemente usado para isso é o de tendência de realização, que está presente em todos os organismos vivos. Trata-se do fundamento sobre o qual está construída a abordagem centrada-na-pessoa (Rogers, 1977/1986, p. 17).

     

    http://pepsic.bvsalud.org/scielo.php?script=sci_arttext&pid=S1809-68672014000100003 

  • "todo organismo é movido por uma tendência inerente para desenvolver todas as suas potencialidades e para desenvolvê-las de maneira a favorecer sua conservação e seu enriquecimento" (p. 159). Desta forma, podemos considerar a tendência atualizante um fundamento propulsor da noção de homem no pensamento de Carl Rogers, questão que foi desenvolvida em estudos posteriores de outros promotores da ACP (Advíncula, 1991; Hipólito, 1999; Moreira, 2007, 2009). Rogers afirmou: "Descobrimos, dentro da pessoa, sob certas condições, uma capacidade para a reestruturação e reorganização do self, e, consequentemente, a reorganização do comportamento, o que tem profundas implicações sociais" (Rogers, 1947, p. 368).

     Rogers considerava a tendência atualizante uma capacidade do indivíduo se reorganizar e se reestruturar, uma vez que certas condições facilitadoras deste processo sejam atendidas(...) existe em todo organismo, em qualquer nível, um fluxo subjacente de movimento para uma realização construtiva de suas possibilidades intrínsecas. Há no homem uma tendência natural para o desenvolvimento completo. O termo mais frequentemente usado para isso é o de tendência de realização, que está presente em todos os organismos vivos. Trata-se do fundamento sobre o qual está construída a abordagem centrada-na-pessoa (Rogers, 1977/1986, p. 17).

  • TENDÊNCIA À AUTOATUALIZAÇÃO: postulado fundamental da teoria rogeriana;

    impulso inerente em direção a sermos tão competentes e capazes quanto estamos aptos a ser biologicamente;

    a pessoa é impelida a tornar-se total, completa e autoatualizada;

    conduz todo organismo a desenvolver-se, tornar-se autônomo, amadurecer, expressar-se, ser responsável por ativar todas as capacidades do organismo;

     

    Quem escolheu a busca não pode recusar a travessia - Guimarães Rosa

    ------------------- 

    Gabarito: E


ID
2478769
Banca
PUC-PR
Órgão
TJ-PR
Ano
2017
Provas
Disciplina
Psicologia
Assuntos

Sobre as propriedades psicométricas dos instrumentos de avaliação psicológica, assinale a alternativa CORRETA.

Alternativas
Comentários
  • Validade:

    A validade de um teste nada mais é do que a capacidade dele de medir/mensurar aquilo que ele propõe. Se um teste propõe avaliar a capacidade de leitura de uma pessoa então, necessariamente, deve ser capaz de avaliar a capacidade de leitura de uma pessoa.

    Para que possamos verificar a validade de um teste podemos fazer jus a um desses três métodos de validação: Validade de conteúdo, Validade de Critério ou Validade de Constructo.

    Validade de Conteúdo: Aqui avalia a escolha dos itens para saber se realmente são apropriados para a verificação proposta do teste. Exemplo, se um teste propõe avaliar "a fala" de uma pessoa, então o teste não seria valido se suas formas de avaliação fosse fazer a pessoa escrever. Este tipo de validade não requer qualquer tipo de tratamento estatístico, mas simi métodos racionas e lógicos.

    Validade de Critério: É a eficácia que um teste tem em predizer um determinado desempenho de um sujeito. O teste, dentro da validade de critério, pode ser um preditor presente ou futuro. Existem dois tipos de Validade de Critério, a Preditiva e a Concorrente:

    Validade Preditiva: O interesse está no desempenho futuro do sujeito. Exemplo: Teste vocacionais, classificação de pessoal.

    Validade Concorrente: Aqui a coleta de informações pelo teste a ser validado e a coleta de informações sobre o critéro são simultâneas.

    Validade de Constructo: É a capacidade de um teste medir um conceito abstrato. Este tipo de validade busca pesquisas as qualidades psicológicas que um determinado teste mede.

    Visite nosso Facebook: Psicologia Pra Passar

  • GABARITO C


ID
2478772
Banca
PUC-PR
Órgão
TJ-PR
Ano
2017
Provas
Disciplina
Psicologia
Assuntos

Sobre as Teorias do Desenvolvimento apresentadas nas proposições a seguir.

I. Psicogenética (Piaget).

II. Psicossocial (Erikson).

III. Histórico-cultural (Vygostky).

É CORRETO apenas o que se afirma em:

Alternativas
Comentários
  • Letra B

  • Gabarito letra B

     aAs teorias apontadas nas proposições I e III compreendem o desenvolvimento
    como estágios e/ou períodos que se sucedem. ERRADO: NA TEORIA DE VYGOSTKY NÃO
    HÁ ESTÁGIOS OU PERÍODOS DE DESENVOLVIMENTO QUE SE SUCEDEM.
     bSegundo a teoria apontada na proposição III, o nível de desenvolvimento real
    de uma criança define funções que já amadureceram, ou seja, os produtos finais
    do desenvolvimento. GABARITO
     cAs teorias apontadas nas proposições I e II apresentam como foco de estudo e
    pesquisa o desenvolvimento sócio-cognitivo. ERRADO: SOMENTE A TEORIA DE
    ERICKSON APRESENTA UM FOCO NO ESTUDO DO DESENVOLVIMENTO PSICOSSOCIAL. A TEORIA
    DE PIAGET TEM SEU FOCO NO DESENVOLVIMENTO COGNITIVO.
     dPara a teoria apontada na proposição III, o adulto experiente atua como um
    mediador do conhecimento, enquanto que para a teoria apontada na proposição
    II, o adulto deve ser uma figura de afeto. ERRADO: NA TEORIA DE ERICKSON NÃO É
    DITO SOBRE O ADULTO COMO FIGURA DE AFETO,SENDO QUE A TEORIA DO MESMO FALA QUE
    EM CADA FASE DO DESENVOLVIMENTO PSICOSSOCIAL É PRECISA REALIZAR AS RESOLUÇÕES
    DAS CRISES BÁSICAS.
     ePara a teoria apontada na proposição I, o ponto crítico do desenvolvimento é
    a relação entre as bases biológicas do comportamento e os acontecimentos
    sociais, sendo o conceito fundamental o de sistema funcional do
    aprendizado.ERRADO: O PONTO CRÍTICO É A RELAÇÃO ENTRE AQUELE QUE QUER CONHECER
    E AQUELE QUE SE PERMITE CONHECER, OU SEJA, A RELAÇÃO INTERDEPENDENTE ENTRE
    SUJEITO E OBJETO, COM UM FOCO MAIOR NA CONSTRUÇÃO DO CONHECIMENTO PELO SUJEITO
    (COGNIÇÃO). A TEORIA DA EQUILIBRAÇÃO É FUNDAMENTAL PARA ESTABELECER E
    CONSTRUIR ESSA RELAÇÃO. 
    O CONCEITO DE SISTEMA FUNCIONAL DE APRENDIZADO FOI INTRODUZIDO POR LURIA.


ID
2478775
Banca
PUC-PR
Órgão
TJ-PR
Ano
2017
Provas
Disciplina
Psicologia
Assuntos

Com base no modelo bioecológico do desenvolvimento humano, analise as alternativas a seguir.

I. O ambiente/contexto ecológico é constituído por um conjunto de sistemas interdependentes vistos como organização de encaixe de estruturas concêntricas.

II. Para Bronfenbrenner, na compreensão do desenvolvimento humano, a herança genética é fundamental, definirá o desenvolvimento e se sobrepõe ao fenótipo.

III. Para compreender o desenvolvimento humano, é necessário considerar o ambiente ecológico, o qual é composto por oito subsistemas.

IV. Desenvolvimento é o processo por meio do qual a pessoa se transforma e adquire uma concepção mais ampliada, diferenciada e válida do meio.

V. O desenvolvimento acontece na sucessão de estágios, sendo que em cada estágio o ego passa por uma crise. O desenvolvimento do indivíduo está relacionado ao seu contexto social, o qual influencia as crises.

Estão CORRETAS somente

Alternativas
Comentários
  • RESPOSTA "A"

    vejamos, 

    A abordagem ecológica desenvolvida por Bronfenbrenner (1977, 1989, 1996) privilegia estudos em desenvolvimento de forma contextualizada e em ambientes naturais, diferentemente de experiências em laboratório, visando apreender a realidade de forma abrangente, tal como é vivida e percebida pelo ser humano no contexto em que habita. Sua teoria ficou mais conhecida e, utilizada por vários pesquisadores no Brasil (HADDAD,1997; FLEURY, 1999 e YUNES, 2001) como um modelo que diferencia as várias camadas de ambientes. O ambiente ecológico de desenvolvimento humano não se limita apenas a um ambiente único e imediato, e deve ser “concebido topologicamente como uma organização de estruturas concêntricas, cada uma contida na seguinte” (BRONFENBRENNER, 1996 p.18). Esse conjunto de estruturas, que no dizer do autor parece lembrar um jogo de bonecas russas encaixadas uma dentro da outra, interferem mutuamente entre si e afetam conjuntamente o desenvolvimento da pessoa. Cada uma das estruturas é chamada pelo autor de: micro-, meso-, exo- e macrossistema.

    (...) microssistema que é definido como: “um padrão de atividades, papéis e relações interpessoais experienciados pela pessoa em desenvolvimento num dado ambiente com características físicas e materiais específicas” (p. 18). Portanto, ambientes tais como a casa, a creche ou a escola em que a pessoa é envolvida em interações face-a-face fazem parte do microssistema.

    O mesossistema diz respeito às inter-relações entre dois ou mais ambientes nos quais uma pessoa participa ativamente, podendo ser formado ou ampliado sempre que ela passe a fazer parte de novos ambientes. Em alguns casos, por exemplo, esse sistema inclui as relações que uma criança mantém em casa, na escola, no clube e com amigos da vizinhança; em outros, apenas as relações exclusivamente familiares e com membros da igreja da qual sua família faz parte.

    Num exosssistema, ao contrário do mesossistema, a criança ou pessoa em desenvolvimento não é participante ativa, mas aí podem ocorrer eventos que a afetem, ou ainda vice e versa, podem ser afetados por acontecimentos do ambiente imediato onde a criança se encontra. Estes tipos de ambientes que consistem em exosssistemas podem ser por exemplo: o local de trabalho dos pais, a escola do irmão ou a rede de amigos dos pais.

    Finalmente, o macrossistema envolve todos os outros ambientes, formando uma rede de interconexões que se diferenciam de uma cultura para outra. Neste caso, podemos dar, como exemplo, a estrutura política e cultural de uma família norte americana de classe média enquanto sistema, muito diferente de um grupo familiar de operários brasileiros.

    A abordagem ecológica de Urie Bronfenbrenner em estudos com famílias

    http://pepsic.bvsalud.org/scielo.php?script=sci_arttext&pid=S1808-42812004000100006

     


ID
2478778
Banca
PUC-PR
Órgão
TJ-PR
Ano
2017
Provas
Disciplina
Psicologia
Assuntos

Uma das formas de melhor compreender o contexto familiar onde ocorre o abuso sexual contra crianças e/ou adolescentes, é direcionar o foco da análise para os personagens principais dessa história: a mãe, o abusador e a vítima.

Considerando tal perspectiva, é CORRETO afirmar:

Alternativas
Comentários
  • Segundo Araújo (2002), no que diz respeito à experiência de atendimento a casos de abuso sexual intrafamiliar, o fato de muitas mães não acreditarem ou mesmo punirem suas filhas pelo acontecido pode ser visto como uma forma de suportar o desmantelamento da unidade familiar e conjugal. A negação das mães pode denunciar uma postura de cumplicidade silenciosa com o agressor, freqüentemente encontrada em casais com conflitos sexuais;

     

    Forward & Buck (1989) descreveram a figura materna como uma cúmplice silenciosa ou como uma pessoa ativamente envolvida;

     

    Fonte: http://www.scielo.br/pdf/estpsi/v25n4/a14v25n4.pdf

     

    ------------------- 

    Gabarito: E

  • Qual o erro da B?
  • Acredito que o erro da letra b é dizer que a criança é conivente com o ato, sendo que, na verdade, ela é vítima.

    Significado de Conivente

    Cúmplice; que esconde uma ação ilegal ou imoral cometida por outra pessoa; que não faz nada para impedir uma ação errada ou criminosa.Que não tenta evitar nem prevenir um crime; que expressa cumplicidade.

  • Não acredito que a B esteja errada, uma vez que muitas crianças se sentem coniventes com o abuso mediante presentes, doces e promessas em geral.


ID
2478781
Banca
PUC-PR
Órgão
TJ-PR
Ano
2017
Provas
Disciplina
Psicologia
Assuntos

“A personalidade pode ser definida como uma constelação única de traços psicológicos de um indivíduo, a qual é relativamente estável com o passar do tempo” (COHEN, 2014). Sobre a avaliação psicológica da personalidade, assinale a alternativa CORRETA.

Alternativas
Comentários
  • D - Atualmente, o Desenho da Figura Humana permite avaliar questões tanto cognitivas quanto de personalidade

    file:///C:/Users/karina/Downloads/pa-3510.pdf

     


ID
2478784
Banca
PUC-PR
Órgão
TJ-PR
Ano
2017
Provas
Disciplina
Psicologia
Assuntos

Diferentes estratégias podem ser utilizadas em um processo de avaliação psicológica. “O teste psicológico é considerado como um dispositivo ou procedimento de medida”(COHEN, 2014). Uma avaliação psicológica pode incluir testes de diferentes domínios. Sobre os testes psicológicos, assinale a alternativa CORRETA.

Alternativas
Comentários
  • RESPOSTA "B"

    Acerca da letra "E" que está incorreta

    As Escalas Beck são compostas pelo Inventário de Depressão (BDI), Inventário de Ansiedade (BAI), Escala de Desesperança (BHS) e Escala de Ideação Suicida (BSI). O BDI mede a intensidade da depressão, e o BAI, a intensidade da ansiedade. A BHS é uma medida de pessimismo e oferece indícios sugestivos de risco de suicídio em sujeitos deprimidos ou que tenham história de tentativa de suicídio. A BSI detecta a presença de ideação suicida, mede a extensão da motivação e planejamento de um comportamento suicida. 

    http://www.pearsonclinical.com.br/escala-beck.html

  • para saber os testes favoraveis ver tabela:http://satepsi.cfp.org.br/listaTeste.cfm

  • Eu fiquei com uma breve dúvida entre a "B" (correta) e a "D" (sobre as escalas WAIS e WISC). Eu sabia que o WISC-IV tinha saído, mas não tinha certeza se já tinham validado o WAIS-IV aqui. Consegui resolver porque lembrei que tem o WASI (a versão abreviada do WAIS) e isso invalidou a alternativa. :)

ID
2478787
Banca
PUC-PR
Órgão
TJ-PR
Ano
2017
Provas
Disciplina
Psicologia
Assuntos

Leia a citação a seguir.

Buosi (2012, p.54) ilustra:

A origem da Síndrome de Alienção Parental (SAP) ocorre exatamente no momento em que um dos genitores percebe o interesse do outro genitor em preservar a convivência afetiva com a criança, e a usa de forma vingativa perante ressentimentos advindos da época do relacionamento ou da separação, programando o filho a odiar e rejeitar o outro genitor sem nenhuma justificativa plausível.”

A respeito de Alienação Parental, é CORRETO afirmar sobre sua origem:

Alternativas
Comentários
  •  

    O genitor alienado (ERRO) geralmente se apresenta com um perfil de superprotetor, que não consegue ter consciência da raiva que está sentindo e, com intencionalidade de se vingar do outro, passa a emitir os comportamentos alienadores. 

    O discurso verbal do genitor alienador (ERRADO) é sempre no sentido de que está pensando no melhor para si, em seus interesses e em tudo o que possa fazer para sentir-se melhor. 

    O fato de um genitor alienar a criança contra o outro genitor se torna cada vez mais comum em disputas de guarda pelos filhos e separações conjugais, nas quais tal manipulação faz com que o genitor alienado ganhe força (ERRADO)

    A partir do momento em que as situações não estão resolvidas entre os genitores, eles se sentirão lesados e possivelmente alimentarão um desejo de vingança para com o outro, sendo, portanto, os filhos a forma mais acessível de atingir esse objetivo. 

    Os abusos psicológicos, que são realizados sutilmente e sem que os envolvidos percebam (ERRADO) são comumente aplicados pelos filhos alienados ao buscar desmoralizar um genitor (ERRADO)

     

    Indicar para comentário

  • De acordo com Buosi (2011) "A Síndrome da Alienação parental atinge principalmente crianças e adolescentes, filhos de casais separados que enfrentam uma disputa judicial sobre a guarda dessa criança, que passa a ser usada por um dos genitores como objeto de vingança do outro, que detém mágoas e sentimentos mal resolvidos da dissolução conjugal. A partir daí, um jogo de manipulações se instala, e uma série de consequências negativas atinge os envolvidos" (p. 9).

    Ainda segundo Buosi (2011), "a partir do momento em que as situações não estão resolvidas entre os ex-parceiros, eles se sentirão lesados e possivelmente alimentarão um desejo de vingança para com o outro, sendo, portanto, os filhos a forma mais acessível de atingi-lo. Assim, quem irá sofrer em função dos problemas do casal será a criança, o que não poderia ocorrer tendo em vista que os adultos deveriam separar os problemas relacionados à conjugalidade e parentalidade para resguardar os filhos" (p. 46).

    Fonte: Lei da Alienação Parental: O contexto sociojurídico da sua promulgação e uma análise dos seus efeitos

    Gabarito: D


ID
2478790
Banca
PUC-PR
Órgão
TJ-PR
Ano
2017
Provas
Disciplina
Psicologia
Assuntos

Analise as afirmativas abaixo que exemplificam a alienação parental.

I. Omitir deliberadamente ao genitor informações pessoais relevantes sobre a criança ou adolescente, inclusive escolares e médicas.

II. Dificultar o exercício do direito regulamentado de convivência social comunitária.

III. Realizar campanha de desqualificação da conduta do genitor no exercício da paternidade ou maternidade.

IV. Apresentar falsa denúncia contra genitor, contra familiares deste ou contra avós para obstar ou dificultar a convivência deles com a criança ou adolescente.

V. Mudar o domicílio para local distante, sem justificativa, visando a dificultar a convivência da criança ou adolescente com o outro genitor, familiares deste ou com avós.

É CORRETO o que se afirma nas alternativas.

Alternativas
Comentários
  • LEI Nº 12.318/10

    Art. 2º, § único São formas exemplificativas de alienação parental, além dos atos assim declarados pelo juiz ou constatados por perícia, praticados diretamente ou com auxílio de terceiros: 

    I - realizar campanha de desqualificação da conduta do genitor no exercício da paternidade ou maternidade; (III)

    II - dificultar o exercício da autoridade parental; 

    III - dificultar contato de criança ou adolescente com genitor; 

    IV - dificultar o exercício do direito regulamentado de convivência familiar; (II)

    V - omitir deliberadamente a genitor informações pessoais relevantes sobre a criança ou adolescente, inclusive escolares, médicas e alterações de endereço; (I)

    VI - apresentar falsa denúncia contra genitor, contra familiares deste ou contra avós, para obstar ou dificultar a convivência deles com a criança ou adolescente; (IV)

    VII - mudar o domicílio para local distante, sem justificativa, visando a dificultar a convivência da criança ou adolescente com o outro genitor, com familiares deste ou com avós. (V)

    Gabarito: Apenas II está incorreta, mas não há essa opção, por isso foi anulada.


ID
2478793
Banca
PUC-PR
Órgão
TJ-PR
Ano
2017
Provas
Disciplina
Psicologia
Assuntos

Os Transtornos de Ansiedade incluem os transtornos que compartilham características de medo e ansiedade excessivos e perturbações relacionadas. Sobre esse assunto, marque a alternativa CORRETA.

Alternativas
Comentários
  • a)O indivíduo apresenta preocupação excessiva com perda da realidade e alteração na função do ego.  ERRADA

     

     b)Os ataques de pânico ocorrem apenas diante de uma situação social temida.  ERRADA

     

     c)O ataque de pânico é um surto abrupto de medo intenso a nível de pensamento sem manifestações físicas.  ERRADA

     

     d)Crianças com transtorno de ansiedade de separação apresentam dificuldade em estar longe de um cuidador primário ou de figura de apego.CERTA

     

     e)As principais características dos transtornos de ansiedade são agitação motora e alteração da sensopercepção, incluindo alucinaçãoERRADA

     

     

    https://www.instagram.com/diariodapsicologa/?hl=pt-br

  • a) perda da realidade é característica das psicoses, não das neuroses;

    b) são as fobias que ocorrem apenas diante de uma situação temida, não necessariamente social;

    c) o ataque de pânico é um surto abrupto de medo intenso a níveis psíquico e físico;

    e) alucinação é característica, sobretudo, das psicoses;

    Quem escolheu a busca não pode recusar a travessia - Guimarães Rosa

    Gabarito: D


ID
2478796
Banca
PUC-PR
Órgão
TJ-PR
Ano
2017
Provas
Disciplina
Psicologia
Assuntos

O abuso sexual pode ser considerado um grave problema de saúde pública, pois há índices significativos de incidência e vários estudos sobre os impactos físicos e emocionais causados às vítimas.

Na perspectiva das consequências do abuso sexual, analise as afirmativas a seguir.

I. O abuso sexual pode trazer sérias consequências para o desenvolvimento cognitivo, afetivo e social da vítima, mas não para sua família.

II. Nenhum sintoma psiquiátrico específico resulta universalmente do abuso sexual.

III. Crianças e adolescentes vítimas de abuso sexual têm maior risco de apresentar problemas interpessoais e psicológicos.

IV. Transtorno de estresse pós-traumático e transtorno dissociativo são exemplos de possíveis sequelas psicológicas apresentadas pelas vítimas.

V. As experiências sexualmente abusivas podem comprometer o desenvolvimento humano, principalmente a saúde física das vítimas.

É CORRETO o que se afirma em

Alternativas

ID
2478799
Banca
PUC-PR
Órgão
TJ-PR
Ano
2017
Provas
Disciplina
Psicologia
Assuntos

Com base nos conhecimentos do Behaviorismo Radical, analise as afirmativas a seguir.

I. Todos os comportamentos são aprendidos durante a vida do indivíduo, principalmente através da interação com o meio.

II. Apesar de não negar os sentimentos e as emoções e embora não possam ser acessados, o Behaviorismo Radical optou por considerar como verdade apenas os eventos públicos os quais, podem ser observado igualmente por mais de um espectador.

III. Ações, pensamentos e sentimentos obedecem às mesmas leis, e têm as mesmas propriedades de qualquer outro comportamento, diferenciando-se apenas pelo grau de acessibilidade.

IV. O Behaviorismo Radical baseia-se no pragmatismo.

V. O comportamento operante refere-se ao comportamento que produz modificações no ambiente e é afetado por essas modificações.

Assinale a alternativa CORRETA.

Alternativas
Comentários
  •  a) III, IV e V. 

  • Qual o erro da I?

  • Natasha, comportamentos inatos não são aprendidos. Por exemplo, o dilatar das pupilas diante de pouca luz...

  • Qual o erro da II?

  • I. Todos os comportamentos são aprendidos durante a vida do indivíduo, principalmente através da interação com o meio. Comportamentos inatos não são aprendidos, como sucção em bebês e recolher o pé ferido.

    II. Apesar de não negar os sentimentos e as emoções e embora não possam ser acessados, o Behaviorismo Radical optou por considerar como verdade apenas os eventos públicos os quais, podem ser observado igualmente por mais de um espectador. Considera muitos que não podem ser observados por mais de um nem são públicos, como por exemplo ir ao banheiro após adulto.

    III. Ações, pensamentos e sentimentos obedecem às mesmas leis, e têm as mesmas propriedades de qualquer outro comportamento, diferenciando-se apenas pelo grau de acessibilidade.

    IV. O Behaviorismo Radical baseia-se no pragmatismo.

    V. O comportamento operante refere-se ao comportamento que produz modificações no ambiente e é afetado por essas modificações.

  • Ótima questão! Eu errei, mas reconheço que ela é muito boa. kkkk

    GAB. A


ID
2478802
Banca
PUC-PR
Órgão
TJ-PR
Ano
2017
Provas
Disciplina
Psicologia
Assuntos

Leia o texto a seguir.

“A aproximação entre a Psicologia e o Direito ocorreu a partir da preocupação com a conduta humana. Apesar de atividades de intervenção, orientação e acompanhamento serem igualmente importantes, observa-se que a avaliação psicológica ainda é considerada a principal demanda dos operadores do Direito” (LAGO et al., 2009).

Sobre a avaliação psicológica, no contexto jurídico, assinale a alternativa CORRETA.

Alternativas
Comentários
  • GABARITO: E

    Após a conclusão do processo de avaliação psicológica (QUE É FEITA PELO PSICÓLOGO E EXCLUSIVAMENTE POR ELE), o psicólogo pode apresentar recomendações para as situações de conflito identificadas (INDICAR TRATAMENTO TERAPÊUTICO OU DE OUTRAS ARÉAS), entretanto não pode determinar os procedimentos jurídicos a serem adotados (CLARO NÃO! OS PROCEDIMENTOS JURIDICOS SÃO DE COMPETÊNCIA DOS JUIZES E ADVOGADOS E OUTROS QUE TEM COMO INSTRUMENTO DE TRABALHO A LEI). 

    PSICOLOGIA PRA PASSAR - OSLEM KLESIANO

  • Em casos de suspeita de abuso sexual de crianças e adolescentes, é recomendando que se evite a realização de entrevistas em conjunto com a vítima e seus responsáveis, visto que a observação da dinâmica familiar não deve ser considerada nessas situações. errado

    Os peritos são profissionais reconhecidos na comunidade científica e indicados por uma das partes envolvidas no processo pericial. errado

    O emprego de testes psicológicos em situações de abuso sexual não é usual, sendo as entrevistas e as sessões lúdicas as melhores estratégias adotadas. 

    A competência para ser julgado está relacionada com a capacidade de um réu em entender as acusações contra ele e de ajudar em sua própria defesa. Assim, pessoas com potencial intelectual rebaixado, com transtornos mentais ou com doenças neurológicas são consideradas incompetentes para serem julgadas.  errado

    Após a conclusão do processo de avaliação psicológica, o psicólogo pode apresentar recomendações para as situações de conflito identificadas, entretanto não pode determinar os procedimentos jurídicos a serem adotados. CERTO

  • Art. 7º - Em seu relatório, o psicólogo perito apresentará indicativos
    pertinentes à sua investigação que possam diretamente subsidiar o Juiz na solicitação
    realizada, reconhecendo os limites legais de sua atuação profissional, sem adentrar
    nas decisões, que são exclusivas às atribuições dos magistrados.

     

    RESOLUÇÃO CFP Nº 008/2010
    Dispõe sobre a atuação do psicólogo
    como perito e assistente técnico no Poder
    Judiciário.

  • Resolução do CFP Nº 008/2010

    • Art. 7º - Em seu relatório, o psicólogo perito apresentará indicativos pertinentes à sua investigação que possam diretamente subsidiar o Juiz na solicitação realizada, reconhecendo os limites legais de sua atuação profissional, sem adentrar nas decisões, que são exclusivas às atribuições dos magistrados.

    Além disso, segundo a mesma resolução, "o psicólogo perito é profissional designado para assessorar a Justiça no limite de suas atribuições."

    Gabarito: E


ID
2478805
Banca
PUC-PR
Órgão
TJ-PR
Ano
2017
Provas
Disciplina
Psicologia
Assuntos

Sobre a psicologia analítica, analise as afirmativas a seguir.

I. A psicologia analítica enfatiza a dinâmica dos processos inconscientes, considerando o potencial mitopoiético da psique.

II. A investigação psicológica na psicologia analítica considera os fenômenos em seu âmbito individual (sonhos, fantasias, experiências pessoais) e coletivo (mitos, contos de fadas, obras de arte, acontecimentos sociais e políticos), desde que revestidos de valor simbólico.

III. O sonho pode ser considerado uma ponte epistemológica para efeitos do processo do conhecer, segundo os pressupostos da psicologia analítica.

IV. Os arquétipos, correlatos aos instintos, são experimentados subjetivamente pelo sujeito como impulsos básicos. Eles são princípios de organização psíquica, em torno de temas humanos fundamentais, e seu conteúdo (imagens) é herdado, configurando um acervo de impressões acumuladas pela experiência do homem ao longo dos milênios.

Assinale a alternativa CORRETA.

Alternativas
Comentários
  • Gabarito: C   (Estão corretas apenas as alternativas I, II e III)

     

  • Porque a IV está errada? Obrigada.

  • O erro da IV está na segunda frase. "Considera-se que os arquétipos são padrões básicos de organização psíquica, sem conteúdo a priori, presentes desde o nascimento, que mediam a experiência da criança em torno de temas fundamentais da vida humana e são preenchidos por conteúdos advindos da vivência pessoal." - Michel Alexandre Fillus - O desenvolvimento do ego infatil expresso nas imagens oníricas das crianças (2013)

  • Arquétipo, na Psicologia Analítica, significa a forma imaterial à qual os fenômenos psíquicos tendem a se moldar.[1] C.G.Jung usou o termo para se referir a estruturas inatas que servem de matriz para a expressão e desenvolvimento da psique.

  • Arquétipos

    Um arquétipo é uma forma de pensamento universal (ideia) que contém uma grande parte de emoção. Essa forma de pensamento cria imagens ou visões que correspondem, na vida normal de vigília, a alguns aspectos da situação consciente. Por exemplo, o arquétipo da mãe produz uma imagem da figura materna, que está identificada com a mãe atual. (...)

    Como se origina um arquétipo? É um depósito permamente de uma experiência que foi constantemente repetida durante muitas gerações. (...) Os arquétipos funcionam como centros autônomos, altamente carregados de energia, que tendem a produzir, em cada geração, a repetição e a elaboração dessas mesmas experiências.

    Os arquétipos não estão obrigatoriamente isolados uns dos outros. Eles se interpenetram e se misturam.

     

    fonte: Teorias da Personalidade (Hall e Lindzey)


ID
2478808
Banca
PUC-PR
Órgão
TJ-PR
Ano
2017
Provas
Disciplina
Psicologia
Assuntos

A característica essencial de um transtorno decorrente do uso de substâncias consiste na presença de um agrupamento de sintomas cognitivos, comportamentais e fisiológicos. Sobre esse assunto, é CORRETO afirmar que:

Alternativas
Comentários
  • b

    o uso de dosagens elevadas de substâncias estimulantes pode produzir ansiedades temporária intensas que lembram Transtornos de pânico ou Transtornos de ansiedade generalizadas, bem como ideação paranoide. 


ID
2478811
Banca
PUC-PR
Órgão
TJ-PR
Ano
2017
Provas
Disciplina
Psicologia
Assuntos

Para Piaget “Cada estágio é caracterizado pela aparição de estruturas originais, cuja construção o distingue dos estágios anteriores.” (Piaget, 1964/1987, p. 13).

Durante o banho João diz que o sabonete está cansado, pois mergulhou na banheira. Quando os pais estão conversando, João chora e cria situações para que interrompam a conversa e deem total atenção para ele. Pedro, por sua vez, espera a sua vez para falar, presta atenção ao diálogo dos pais, mas não consegue compreender a discussão dos pais sobre o futuro do país de como será a economia e as possibilidades de reformas políticas. O estágio do desenvolvimento cognitivo em que João e Pedro se encontram é, respectivamente,

Alternativas
Comentários
  • João está no estágio pré-operatório (entre 2 e 6 anos) que tem como uma de suas características o egocentrismo (que pode ser observado no comportamento de chorar e criar situações para conseguir a atenção dos pais). 

    Pedro está no estágio operatório concreto (entre 7 e 11 anos) que tem como caracteristicas o conhecimento de regras e estratégias, uso da lógica indutiva e aumento do conhecimento lógico formal. Como ele ainda não consegue entender muito bem as discussões sobre a situação do país, mostra que ele ainda não está na fase operatorio formal.

    Assim:

    Alternativa correta é a letra A

  • Uma das dicas também para identificar o estágio em que João está, é o fato dele atribuir caractarísticas humanas a seres inanimados. Ex: "João diz que o sabonete está cansado, pois mergulhou na banheira." Que é uma das caracterísiticas do estágio Pré operatório, no qual se denomina Animismo


ID
2478814
Banca
PUC-PR
Órgão
TJ-PR
Ano
2017
Provas
Disciplina
Psicologia
Assuntos

Diversos pesquisadores formularam teorias para compreender de que maneira ocorre o desenvolvimento infantil. Dentre eles, Lev Vygotsky e Jean Piaget apresentaram seus estudos que são considerados referências para diversas pesquisas científicas nessa temática. Sobre o assunto, assinale a única alternativa CORRETA.

Alternativas
Comentários
  • d) Vygotsky considera que aquilo que é zona de desenvolvimento proximal hoje será o nível de desenvolvimento real amanhã, ou seja, aquilo que uma criança pode fazer com assistência hoje, ela será capaz de fazer sozinha amanhã. 

  • erro da questão A:

    No estágio das operações concretas, Piaget estabelece que já se verifica um pensamento lógico bem estruturado e que as operações já repousam sobre proposições de enunciados verbais.( ISSO SÓ ACONTECE NO ESTAGIO DAS OPERAÇÕES FORMAIS).


ID
2478817
Banca
PUC-PR
Órgão
TJ-PR
Ano
2017
Provas
Disciplina
Psicologia
Assuntos

Os quadros de intoxicação, abuso e dependência de álcool e outras substâncias psicoativas se caracterizam por uma forma particular de relação do indivíduo com substância que produzem alterações no sistema nervoso central. Sobre esse assunto, é CORRETO afirmar que

Alternativas
Comentários
  • a síndrome de abstinência de álcool se desenvolve no período de várias horas a alguns dias após a cessação (ou redução) do uso prolongado de álcool e inclui sintomas como hiperatividade autonômica, ansiedade, tremor, insônia entre outros.  

  • a) a dependência é o termo que se dá ao uso mal adaptativo de álcool com mudanças comportamentais e psicológicas;

    b) o consumo de substâncias psicoativas causa  sintomas cognitivos, psicológicos, comportamentais e fisiológicos;

    c) os sintomas de abstinência ajudam a suspender o comportamento de restrição, contribuindo como  facilitador de recaídas;

    d) a intoxicação é definida como uma síndrome reversível com alterações comportamentais, prejuízos do nível de CS e alteração de humor;

     

    Quem escolheu a busca não pode recusar a travessia - Guimarães Rosa

    ------------------- 

    Gabarito: E


ID
2478820
Banca
PUC-PR
Órgão
TJ-PR
Ano
2017
Provas
Disciplina
Psicologia
Assuntos

Sobre o abuso de substâncias psicoativas e sua relação com a psicopatologia, é CORRETO afirmar que

Alternativas

ID
2478823
Banca
PUC-PR
Órgão
TJ-PR
Ano
2017
Provas
Disciplina
Psicologia
Assuntos

A alienação parental é uma violação de direitos fundamentais da criança e do adolescente e constitui um abuso moral com penalização prevista em lei. Havendo indício da prática de ato de alienação parental, em ação autônoma ou incidental, o juiz, se necessário, determinará perícia psicológica ou biopsicossocial. Sobre o assunto, assinale alternativa CORRETA.

Alternativas
Comentários
  • RESPOSTA "C"

    LEI Nº 12.318, DE 26 DE AGOSTO DE 2010.

    Art. 2o  Considera-se ato de alienação parental a interferência na formação psicológica da criança ou do adolescente promovida ou induzida por um dos genitores, pelos avós ou pelos que tenham a criança ou adolescente sob a sua autoridade, guarda ou vigilância para que repudie genitor ou que cause prejuízo ao estabelecimento ou à manutenção de vínculos com este. 

    Parágrafo único.  São formas exemplificativas de alienação parental, além dos atos assim declarados pelo juiz ou constatados por perícia, praticados diretamente ou com auxílio de terceiros:  

    I - realizar campanha de desqualificação da conduta do genitor no exercício da paternidade ou maternidade; 

    II - dificultar o exercício da autoridade parental; 

    III - dificultar contato de criança ou adolescente com genitor; 

    IV - dificultar o exercício do direito regulamentado de convivência familiar; 

    V - omitir deliberadamente a genitor informações pessoais relevantes sobre a criança ou adolescente, inclusive escolares, médicas e alterações de endereço; 

    VI - apresentar falsa denúncia contra genitor, contra familiares deste ou contra avós, para obstar ou dificultar a convivência deles com a criança ou adolescente; 

    VII - mudar o domicílio para local distante, sem justificativa, visando a dificultar a convivência da criança ou adolescente com o outro genitor, com familiares deste ou com avós. 

     

    AINDA NO ARTIGO 6º:

     

    Art. 6o  Caracterizados atos típicos de alienação parental ou qualquer conduta que dificulte a convivência de criança ou adolescente com genitor, em ação autônoma ou incidental, o juiz poderá, cumulativamente ou não, sem prejuízo da decorrente responsabilidade civil ou criminal e da ampla utilização de instrumentos processuais aptos a inibir ou atenuar seus efeitos, segundo a gravidade do caso: 

    I - declarar a ocorrência de alienação parental e advertir o alienador; 

    II - ampliar o regime de convivência familiar em favor do genitor alienado; 

    III - estipular multa ao alienador; 

    IV - determinar acompanhamento psicológico e/ou biopsicossocial; 

    V - determinar a alteração da guarda para guarda compartilhada ou sua inversão; 

    VI - determinar a fixação cautelar do domicílio da criança ou adolescente; 

    VII - declarar a suspensão da autoridade parental. 

    http://www.planalto.gov.br/ccivil_03/_ato2007-2010/2010/lei/l12318.htm

  •  a)Em razão do caráter de urgência de apreciação de situações de indícios de alienação parental, o Ministério Público não se manifestará nos autos, salvo se houver pedido expresso dessa manifestação feito por uma ou por ambas as partes. Ministério Público, o fiscal da lei, sempre se manifestará nos autos

     

     b)Dada a complexidade de realizar a perícia relativa à avaliação psicológica ou biopsicossocial, conforme for o caso, o perito ou a equipe multidisciplinar terá o prazo de 45 dias para apresentar o laudo, prazo esse prorrogado uma única vez, por igual período, mediante autorização judicial justificada. Prazo de 90 dias

     

     c)Verificada a prática de alienação parental, a autoridade judiciária poderá inverter a guarda ou mesmo converter a guarda para guarda compartilhada da criança ou adolescente. CORRETA

     

     d)A alienação parental somente pode ser praticada pelos genitores, não podendo ser caracterizada por atos praticados pelos avós de crianças e adolescentes.  Pode ser caracterizada por atos praticados pelos avó, ou por quem detenha a guarda.

     

     e)A mudança de domicílio do genitor detentor da guarda de criança para local distante que dificulta a convivência da criança com outro genitor, ainda que ocorrida por motivo justificável, caracteriza a alienação parental. Sem motivo justificável

     

    "Daqui a um ano, você vai desejar ter começado hoje" Karen Lamb    
     

  • LEI Nº 12.318/2010

    Art. 6º - Caracterizados atos típicos de alienação parental ou qualquer conduta que dificulte a convivência de criança ou adolescente com genitor, em ação autônoma ou incidental, o juiz poderá, cumulativamente ou não, sem prejuízo da decorrente responsabilidade civil ou criminal e da ampla utilização de instrumentos processuais aptos a inibir ou atenuar seus efeitos, segundo a gravidade do caso: 

    •  V - determinar a alteração da guarda para guarda compartilhada ou sua inversão;  

    Vejamos a correção das demais assertivas:

    • a) ouvido o Ministério Público;
    • b) terá prazo de 90 dias para apresentação do laudo;
    • d) inclusive pelos avós ou pelos que tenham sob a sua autoridade, guarda ou vigilância;
    • e) caracteriza alienação parental apenas a mudança sem justificativa;

    Gabarito: C

  • a) ERRADO - art 4º, Lei 12.318/2010 - O MP será ouvido.

    b) ERRADO - art 5º, § 3, Lei 12.318/2010 - O perito ou equipe multidisciplinar designada para verificar a ocorrência de alienação parental terá prazo de 90 (noventa) dias para apresentação do laudo, prorrogável exclusivamente por autorização judicial baseada em justificativa circunstanciada.

    c) CORRETO - art 6º, Lei 12.318/2010.

    d) ERRADO - art 2º, Lei 12.318/2010.

    e) ERRADO - art 2º, Parágrafo Único, inciso VII.

    GABARITO: LETRA C

  • Alienação parental tem servido para condenar crianças e mães à violência de homens.


ID
2478826
Banca
PUC-PR
Órgão
TJ-PR
Ano
2017
Provas
Disciplina
Psicologia
Assuntos

Segundo a Organização Mundial de Saúde, a depressão é hoje uma das principais causas de afastamentos do trabalho e das relações sociais. Sobre os transtornos depressivos, é CORRETO afirmar que

Alternativas
Comentários
  • A característica comum desses transtornos é a presença de humor triste, vazio ou irritável, acompanhado de alterações somáticas e cognitivas que afetam significativamente a capacidade de funcionamento do indivíduo. O que difere entre eles são os aspectos de duração, momento ou etiologia presumida.

    Fonte: DSM-V (pg.155)

  • b) hipomania é sintoma do transtorno bipolar;

    c) episódios distintos envolvendo alteração nítidas do afeto são características do transtorno bipolar;

    d) o processo de luto difere-se do transtorno depressivo maior e nem sempre o desencadeia;

    e) transtorno depressivo disfórico;

    Quem escolheu a busca não pode recusar a travessia - Guimarães Rosa

    Gabarito: A


ID
2478829
Banca
PUC-PR
Órgão
TJ-PR
Ano
2017
Provas
Disciplina
Psicologia
Assuntos

Leia a situação a seguir para responder a questão .

Rafael tem 20 anos de idade e está em uma clínica psiquiátrica há uma semana. De acordo com o relato de sua mãe, ele apresentou mudanças de comportamento após seu pai ter saído de casa, em virtude do rompimento do casamento dos seus pais. Após esse fato, os pais ficaram preocupados porque Rafael deixou de ir às aulas na faculdade de direito, abandonou os cuidados com a higiene e parou de comer. Passava muito tempo em seu quarto escrevendo. Os pais resolveram interná-lo quando observaram Rafael tirando os aparelhos eletrônicos de dentro da casa. Ele falou que havia escutas nesses objetos afirmando que “estão querendo fazer mal à nossa família”. De acordo com a mãe, o filho relatava que seres do “mal” queriam destruí-los.

Considerando as informações contidas na situação hipotética descrita no texto anterior, é CORRETO afirmar:

Alternativas
Comentários
  • e) Apresenta quadro psicótico, pois os sintomas enfatizam a perda do contato com a realidade.

  • " ... tirando os aparelhos eletrônicos de dentro da casa."

    " ... havia escutas nesses objetos afirmando que 'estão querendo fazer mal à nossa família'."

    " ... seres do 'mal' queriam destruí-los."

    Os referidos trechos trazidos pela questão nos dão a ideia de um delírio, mais especificamente persecutório, típico de quadros psicóticos tais como o transtorno delirante ou esquizofrenia paranoide.

    Quem escolheu a busca não pode recusar a travessia - Guimarães Rosa

    Gabarito: E


ID
2478832
Banca
PUC-PR
Órgão
TJ-PR
Ano
2017
Provas
Disciplina
Psicologia
Assuntos

A respeito da Resolução CFP nº 007/2003, a qual institui o Manual de Elaboração de Documentos Escritos produzidos pelo psicólogo, decorrentes de avaliação psicológica, destaca-se que:

O referido Manual dispõe sobre os seguintes itens: Princípios Norteadores; Modalidades de Documentos; Conceito/Finalidade/Estrutura; Validade dos documentos; Guarda dos documentos.

Considerando esses aspectos abordados na normativa, assinale a alternativa CORRETA.

Alternativas
Comentários
  • GABARITO:  b) Ao definir o prazo de validade do conteúdo dos documentos, o psicólogo deve dispor dos fundamentos para tal indicação, devendo apresentá-los sempre que solicitado.  

    V – VALIDADE DOS CONTEÚDOS DOS DOCUMENTOS O prazo de validade do conteúdo dos documentos escritos, decorrentes das avaliações psicológicas, deverá considerar a legislação vigente nos casos já definidos. Não havendo definição legal, o psicólogo, onde for possível, indicará o prazo de validade do conteúdo emitido no documento em função das características avaliadas, das informações obtidas e dos objetivos da avaliação. Ao definir o prazo, o psicólogo deve dispor dos fundamentos para a indicação, devendo apresentá-los sempre que solicitado.

  •  a) O parecer psicológico é um documento que visa a informar a ocorrência de fatos ou situações objetivas relacionadas ao atendimento psicológico realizado. ERRADA - DECLARAÇÃO

     b) Ao definir o prazo de validade do conteúdo dos documentos, o psicólogo deve dispor dos fundamentos para tal indicação, devendo apresentá-los sempre que solicitado.  CORRETA

     c) Os documentos escritos decorrentes da avaliação psicológica, bem como todo o material que os fundamentou, deverão ser guardados pelo prazo máximo de 5 anos, observando-se a responsabilidade por eles tanto do psicólogo quanto da instituição em que ocorreu a avaliação psicológica. ERRADA - "deverão ser guardados pelo prazo mínimo de 5 anos"

     d) O documento produzido pelo psicólogo deve apresentar redação bem estruturada, compatível com as expressões próprias da linguagem coloquial. ERRADA - "emprego de frases e termos deve ser compatível com as expressões próprias da linguagem profissional"

     e) Justificar estar apto ou não para atividades específicas, após realização de um processo de avaliação psicológica, é uma das finalidades do relatório psicológico. ERRADA - ATESTADO PSICOLÓGICO

     

    GABARITO B

     

     

     

    https://www.instagram.com/diariodapsicologa/?hl=pt-br

  • RESOLUÇÃO CFP Nº 007/2003

     

    V – O prazo de validade do conteúdo dos documentos escritos, decorrentes das avaliações psicológicas, deverá considerar a legislação vigente nos casos já definidos. Não havendo definição legal, o psicólogo, onde for possível, indicará o prazo de validade do conteúdo emitido no documento em função das características avaliadas, das informações obtidas e dos objetivos da avaliação. Ao definir o prazo, o psicólogo deve dispor dos fundamentos para a indicação, devendo apresentá-los sempre que solicitado;

     

    Os erros das demais assertivas:

     

    a) o conceito é o da declaração;

    c) deverão ser guardados pelo prazo mínimo de 5 anos;

    d) o emprego de frases e termos deve ser compatível com as expressões próprias da linguagem profissional;

    e) é uma das finalidades do atestado psicológico;

     

    Quem escolheu a busca não pode recusar a travessia - Guimarães Rosa

    ------------------- 

    Gabarito: B


ID
2478835
Banca
PUC-PR
Órgão
TJ-PR
Ano
2017
Provas
Disciplina
Psicologia
Assuntos

A Resolução CFP nº 08/2010 dispõe sobre a atuação do psicólogo como perito e assistente técnico no Poder Judiciário, considerando, em seu teor, que:

O psicólogo perito é profissional designado para assessorar a justiça no limite de suas atribuições e, portanto, deve exercer tal função com isenção em relação às partes envolvidas.

E também estabelece que:

Os assistentes técnicos são de confiança da parte para assessorá-la e garantir o direito ao contraditório, não sujeitos a impedimento ou suspeição legais.

Sendo assim, quanto ao contexto de atuação profissional, previsto na referida Resolução, é CORRETO afirmar que

Alternativas
Comentários
  • os psicólogos peritos e assistentes técnicos deverão fundamentar sua intervenção em referencial teórico, técnico e metodológico respaldados na Ciência Psicológica, no Direito, (errado) na ética e na legislação profissional. 

     

    o psicólogo perito deverá elaborar quesitos e endereçá-los ao(s) assistente(s) técnico(s), a fim de elucidar dúvidas surgidas durante a perícia.  errado

     

    o psicólogo perito e o(s) assistente(s) técnico(s) realizarão seu trabalho a fim de responder diretamente aos quesitos elaborados pelo juiz.  

    errado

    o psicólogo que atua como psicoterapeuta das partes envolvidas em um litígio poderá atuar como perito ou assistente técnico, somente mediante autorização dos envolvidos.  errado

     

  • CONSIDERANDO que, quando a prova do fato depender de conhecimento
    técnico ou científico, o juiz será assistido por perito, por ele nomeado;

     

     

  • RESOLUÇÃO CFP Nº 008/2010

    CONSIDERANDO que, quando a prova do fato depender de conhecimento técnico ou científico, o juiz será assistido por perito, por ele nomeado; (A)

    CONSIDERANDO que os psicólogos peritos e assistentes técnicos deverão fundamentar sua intervenção em referencial teórico, técnico e metodológico respaldados na ciência Psicológica, na ética e na legislação profissional, garantindo como princípio fundamental o bem-estar de todos os sujeitos envolvidos (B)

    • Art. 2º § único A relação entre os profissionais deve se pautar no respeito e colaboração, cada qual exercendo suas competências, podendo o assistente técnico formular quesitos ao psicólogo perito (C e D)
    • Art. 10,inciso I É vedado ao psicólogo que esteja atuando como psicoterapeuta das partes envolvidas em um litígio atuar como perito ou assistente técnico de pessoas atendidas por ele e/ou de terceiros envolvidos na mesma situação litigiosa (E)

    Gabarito: A


ID
2478838
Banca
PUC-PR
Órgão
TJ-PR
Ano
2017
Provas
Disciplina
Psicologia
Assuntos

Leia as afirmativas a seguir sobre Terapia Cognitiva.

I. O objetivo da Terapia Cognitiva é reestruturar e corrigir os pensamentos distorcidos utilizando as abordagens colaborativa e psicoeducativa para desenvolver soluções pragmáticas para produzir mudanças e melhorar os transtornos emocionais.

II. A maneira com que os indivíduos percebem e processam a realidade tem influência direta na maneira como cada pessoa se comporta e se sente.

III. Alguns pensamentos, conhecidos como pensamentos automáticos, ocorrem espontaneamente nas fronteiras da consciência, por esse motivo não é possível percebê-los e avaliá-los.

IV. Não há uma relação recíproca entre afeto e cognição, pois a cognição depende de suas estruturas internas, o afeto, por sua vez, está ligado com as experiências derivadas de comportamentos do indivíduo.

V. Os esquemas de indivíduos mal ajustados podem levar à distorção da realidade e com isso gerar um transtorno psicológico.

Assinale a alternativa CORRETA.

Alternativas
Comentários
  • Terapia Cognitiva (Psicoterapias: Abordagens Atuais)

    Resumo para melhor entendimento da questão!

    Seu foco de atenção é a atividade mental consciente ou pré-consciente (pensamentos automáticos, crenças subjacentes e suas conseqüências: emoções, comportamentos ou reações físicas).
    Fundamenta-se nas teorias do processamento patológico das informações
    De acordo com o modelo, existem erros (de lógica) no processamento da informação sob a forma de pensamentos disfuncionais e distorções cognitivas típicas: na depressão, nos transtornos de ansiedade, nos transtornos de personalidade, nos transtornos alimentares, entre outros.
    Há ainda esquemas disfuncionais nos transtornos de personalidade, nas relações conjugais e familiares. Essas distorções cognitivas, associadas
    a erros de lógica, como avaliações e interpretações distorcidas, provocam alterações no humor, reações físicas e comportamento desadaptativo, que acabam criando e perpetuando um círculo vicioso.
     

    Alternativa C


ID
2478841
Banca
PUC-PR
Órgão
TJ-PR
Ano
2017
Provas
Disciplina
Psicologia
Assuntos

Sobre a Abordagem Centrada na Pessoa (ACP), marque a alternativa CORRETA.

Alternativas
Comentários
  • RESPOSTA "E"

    A letra "B" está errada, pois

    " Rogers declarou, explicitamente, não ter estudado a filosofia existencial, com a qual tomou contato tardiamente a partir da leitura de Buber e de Kierkegaard, em resposta à insistência dos seus alunos. A partir do trabalho com Gendlin (1970), no Center for Studies of the Person, em La Jolla, na Califórnia, Rogers sentiu-se atraído pela ênfase na experiência como conceito fenomenológico-existencial, que, posteriormente, derivou na abordagem experiencial, uma vertente atual da Abordagem Centrada na Pessoa, fundada por Gendlin. Seu contato com Gendlin, possivelmente, contribuiu para que Rogers passasse do positivismo lógico para uma orientação existencialista, na sua fase experiencial; no entanto, considerar toda a psicologia rogeriana como fenomenológica seria um exagero evidente, haja vista que Rogers adotou tal denominação tardia e incidentalmente e nunca tentou praticar, intencionalmente, uma abordagem fenomenológica (Spiegelberg, 1972; Cury, 1987; Moreira, 2001, 2007)."

    Acerca da letra "E",

    " Rogers sofreu, durante toda a sua vida, com o difícil dilema entre pessoa e ciência (Rogers, 1976), haja vista que, para ele, a experiência subjetiva o levara a criar uma proposta inovadora de psicoterapia, quando, como cientista, se angustiava, dado que seu modelo de ciência era eminentemente positivista, presa da objetividade, entendida como a ciência de sua época e de seu momento histórico cultural. Provavelmente seu estilo pragmático, tipicamente norte-americano, aliado ao seu contexto histórico, onde metodologias qualitativas não eram consideradas cientificas, o impediu de buscar metodologias qualitativas mais apropriadas para seus estudos clínicos. Não contou com o método fenomenológico ou outro método qualitativo em suas pesquisas pioneiras em psicoterapia, que, na época, para serem reconhecidas como “científicas”, precisaram ser desenvolvidas como investigações positivistas, ou seja, apenas quantitativas."

    A Gestalt-terapia e a Abordagem Centrada na Pessoa são enfoques fenomenológicos?

    http://pepsic.bvsalud.org/scielo.php?script=sci_arttext&pid=S1809-68672009000100002

  • A Abordagem Centrada na Pessoa é uma abordagem das relações interpessoais desenvolvida pelo psicólogo americano Carl Rogers (1902-1987), um dos mais renomados psicólogos do século XX e um pioneiro na pesquisa científica em psicoterapia.

    fonte: https://apacp.org.br/o-que-e-a-acp/

  • O Gabarito é a letra "E". E as respostas estão fundamentadas neste artigo: http://pepsic.bvsalud.org/pdf/rnufen/v4n2/a04.pdf


ID
2478844
Banca
PUC-PR
Órgão
TJ-PR
Ano
2017
Provas
Disciplina
Psicologia
Assuntos

A Resolução CFP nº 010/2005, aprova o Código de Ética Profissional do Psicólogo (CEPP), sendo possível encontrar:

Toda profissão define-se a partir de um corpo de práticas que busca atender demandas sociais, norteado por elevados padrões técnicos e pela existência de normas éticas que garantam a adequada relação de cada profissional com seus pares e com a sociedade como um todo.”

Analise as afirmativas a seguir.

I. No atendimento à criança, ao adolescente ou ao interdito, deve ser comunicado aos responsáveis o estritamente essencial para se promoverem medidas em seu benefício.

II. O psicólogo, no relacionamento com profissionais não psicólogos, compartilhará somente informações relevantes para qualificar o serviço prestado, resguardando o caráter confidencial das comunicações.

III. Quando requisitado a depor em juízo, o psicólogo não poderá prestar informações, considerando a obrigatoriedade de manutenção do sigilo.

IV. A utilização de quaisquer meios de registro e observação da prática psicológica obedecerá às normas vigentes, devendo o usuário ou beneficiário, desde o início, ser informado.

V. Nos documentos que embasam as atividades em equipe multiprofissional, o psicólogo registrará todas as informações referentes ao trabalho desenvolvido.

Assinale a alternativa que corresponde às afirmativas CORRETAS.

Alternativas
Comentários
  • Gabarito: B) I, II e IV

  • I. Correto! Art 13.

    II. Correto! Art 6.

    III. Incorreto. 

    Art. 11 – Quando requisitado a depor em juízo, o psicólogo poderá prestar informações, considerando o previsto neste Código.

    IV. Correto. Art 14

    V. Incorreto!

    Art. 12 – Nos documentos que embasam as atividades em equipe multiprofissional, o psicólogo registrará apenas as informações necessárias para o cumprimento dos objetivos do trabalho.

     

  • Art. 13 – No atendimento à criança, ao adolescente ou ao interdito, deve ser comunicado aos responsáveis o estritamente essencial para se promoverem medidas em seu benefício

    Art. 6°

    b) Compartilhará somente informações relevantes para qualificar o serviço prestado, resguardando o caráter confidencial das comunicações, assinalando a responsabilidade, de quem as receber, de preservar o sigilo.

    Art. 14 – A utilização de quaisquer meios de registro e observação da prática psicológica obedecerá às normas deste Código e a legislação profissional vigente, devendo o usuário ou beneficiário, desde o início, ser informado.

  • RESOLUÇÃO CFP Nº 010/05

     

    Somente as assertivas I, II e IV estão corretas. Vejamos os erros das demais assertivas:

     

    III) Quando requisitado a depor em juízo, o psicólogo poderá prestar informações, considerando o previsto neste Código;

     

    V) Nos documentos que embasam as atividades em equipe multiprofissional, o psicólogo registrará apenas as informações necessárias para o cumprimento dos objetivos do trabalho;

     

    Quem escolheu a busca não pode recusar a travessia - Guimarães Rosa

    ------------------- 

    Gabarito: B


ID
2478847
Banca
PUC-PR
Órgão
TJ-PR
Ano
2017
Provas
Disciplina
Psicologia
Assuntos

Para o tratamento de pacientes que abusam de drogas, a principal estratégia do psicólogo deve ser

Alternativas
Comentários
  • e

    o tratamento com foco na autonomia do indivíduo que pode, assim, resgatar sua condição de sujeito social e autônomo.

  • Poderia ser menos óbvia


ID
2478850
Banca
PUC-PR
Órgão
TJ-PR
Ano
2017
Provas
Disciplina
Psicologia
Assuntos

A Lei 12.010/2009, em seu Art. 39, Parágrafo 1, estabelece:

“A adoção é medida excepcional e irrevogável, à qual se deve recorrer apenas quando esgotados os recursos de manutenção da criança ou adolescente na família natural ou extensa (...)

Sobre o tema, avalie as afirmações a seguir.

I. O estágio de convivência poderá ser dispensado se o adotando já estiver sob a tutela ou guarda legal do adotante durante tempo suficiente para que seja possível avaliar a conveniência da constituição do vínculo.

II. Para adoção conjunta, é indispensável que os adotantes sejam casados civilmente ou mantenham união estável, comprovado o exercício de alguma prática religiosa e a estabilidade da família.

III. A inscrição de postulantes à adoção será precedida de um período de preparação psicossocial e jurídica, orientado pela equipe técnica da Justiça da Infância e da Juventude.

IV. A adoção será indeferida ao adotante que, após inequívoca manifestação de vontade, vier a falecer no curso do procedimento, antes de prolatada a sentença judicial.

V. Serão criados e implementados cadastros estaduais e nacional de crianças e adolescentes em condições de serem adotados e de pessoas ou casais em processo de habilitação à adoção.

É CORRETO o que se afirma apenas em

Alternativas
Comentários
  • Gab. D

    Lei 12.010/2009:

    II) art. 42-§ 2o  Para adoção conjunta, é indispensável que os adotantes sejam casados civilmente ou mantenham união estável, comprovada a estabilidade da família.

    IV) art. 42- § 6o  A adoção poderá ser deferida ao adotante que, após inequívoca manifestação de vontade, vier a falecer no curso do procedimento, antes de prolatada a sentença.

    V) art.50- § 5o  Serão criados e implementados cadastros estaduais e nacional de crianças e adolescentes em condições de serem adotados e de pessoas ou casais habilitados à adoção.  

  • § 5o  Serão criados e implementados cadastros estaduais e nacional de crianças e adolescentes em condições de serem adotados e de pessoas ou casais habilitados à adoção. 

  • LEI Nº 8.069/1990.

    Os erros das demais asseetivas:

    II) comprovada a estabilidade da família apenas; a prática religiosa não é requisito (Art. 42 §2º);

    IV) a adoção poderá ser deferida (Art. 42 §6º);

    V) serão criados e implementados cadastros estaduais e nacional de crianças e adolescentes em condições de serem adotados e de pessoas ou casais habilitados à adoção (Art. 50 §5º);

    Gabarito: D